Вы находитесь на странице: 1из 240

A.

RATIONAL NUMBERS: THE SET Q


1. Understanding Rational Numbers
Definition

The ratio of an integer to a non-zero integer is called a rational number. The set of rational
a
numbers is denoted by Q. Q = { | a, b Z, b 0}
b

2. The Set of Positive Rational Numbers


If a rational number represents a point on the number line on the right side of zero, then it
is called a positive rational number.
a
is a positive rational number if a and b are both positive integers or both negab
tive integers.
2
2
2
and
For example,
are positive rational numbers, and denoted by
.
7
7
7

In short,

Definition

The set of positive rational numbers is denoted by Q+.


Q+ = {

a a
| 0 and a, b , b 0}
b b

3. The Set of Negative Rational Numbers


If a rational number represents a point on the number line on the left side of zero, then it is
called a negative rational number.
a
In short,
is a negative rational number if a is a positive integer and b is a negative integer,
b
or if a is a negative integer and b is a positive integer.
5
5
are negative rational numbers. We can write negative rational
and
4
4
5 5
5
numbers in three ways:

.
4
4
4

For example,

Definition

The set of negative rational numbers is denoted by Q.


a a
Q = { | 0 and a, b , b 0}
b b
10

Algebra 8

A. THE SET OF REAL NUMBERS


1. Understanding Real Numbers
In algebra we use many different sets of numbers. For example, we use the natural numbers
to express quantities of whole objects that we can count, such as the number of students in
a class, or the number of books on a shelf.
The set of natural numbers is denoted by N.
N = {1, 2, 3, 4, 5, ...}
-6

-5

-4

-3

-2

-1

Natural numbers

The set of whole numbers is the set of natural numbers together with zero. It is denoted by
W.
W = {0, 1, 2, 3, 4, 5, ...}
-6

-5

-4

-3

-2

-1

Whole numbers

The set of integers is the set of natural numbers, together with zero and the negatives of the
natural numbers. It is denoted by Z.
Z = {..., 5, 4, 3, 2, 1, 0, 1, 2, 3, 4, 5, ...}
-6

-5

-4

-3

-2

-1

Integers

We use integers to express temperatures below zero, distances above and below sea level, and
increases and decreases in stock prices, etc. For example, we can write ten degrees Celsius
below zero as 10C.
To express ratios between numbers, and parts of wholes, we use rational numbers.
8 2
3 0
17
For example, , , , , and
are rational numbers.
3 5
7 7
1
The set of rational numbers is the set of numbers that can be written as the quotient of two
integers. It is denoted by Q.
Q={

a
| a, b and b 0}
b
7
2
-7

-6

-5

-4

1
2
-3

-2

-1

2
3
0

5
4
1

13
2
3

Some rational numbers

Radicals

11

We can write every rational number as a repeating or terminating decimal. Conversely, we


can write any repeating or terminating decimal as a rational number.
3
321
0.6, and
0.324 = 0.324242424...
5
990

0.6 is a terminating decimal, and 0.324 is a repeating decimal.

For example,

There are some decimals which do not repeat or terminate.


For example, the decimals

0.1012001230001234000 ...
3.141592653 ... =

R = R+ {0} R
R+ is the set of positive
real numbers
R is the set of negative
real numbers

2.71828 ...

=e

1.4142135 ...

= 2

do not terminate and do not repeat. Therefore, we cannot write these decimals as rational
numbers. We say that they are irrational.

Definition

A number whose decimal form does not repeat or terminate is called an irrational number.
The set of irrational numbers is denoted by Q or I.
Definition

The union of the set of rational numbers and the set of irrational numbers forms the set of
all decimals. This union is called the set of real numbers.
The set of real numbers is denoted by R.
R = Q Q
For every real number there is a point on the number line. In
other words, there is a one-to-one correspondence between
the real numbers and the points on the number line.

Real Numbers

-2.35
-5

-4

-3

-2

-0.5

0.6

-1
0
1
2
Some rational numbers

p
3

The real numbers fill up the number line.

NWZQR
Q R

12

We can summarize the relationship between the different sets


of numbers that we have described in a diagram. As we know,
the set of natural numbers is a subset of the set of whole
numbers, the set of whole numbers is a subset of the set of
integers, the set of integers is a subset of the set of rational
numbers, and the set of rational numbers is a subset of the
set of real numbers. This relationship is shown by the diagram on the left.
Algebra 8

1. Understanding Square Roots


Remember that we can write a a as a2. We call a2 the square of a, and multiplying a number
by itself is called squaring the number. The inverse operation of squaring a number is called
finding the square root of the number.
Objectives

After studying this section you will be able to:


1. Understand the concepts of square root and radical number.
2. Use the properties of square roots to simplify expressions.
3. Find the product of square roots.
4. Rationalize the denominator of a fraction containing square roots.
Definition

If a2 = b then a is the square root of b (a 0, b 0).


We use the symbol to denote the square root of a number. b is read as the square root of
b. So if a2 = b then a = b (a b, b 0).
Here are the square roots of all the perfect squares from 1 to 100.

1 = 1

2 =4

4 = 2

32 = 9

9 = 3

16 = 4

25 = 5

12 = 1
2

4 = 16
5 = 25

36 = 6

7 = 49

49 = 7

82 = 64

64 = 8

81 = 9

100 = 10

62 = 36
2

9 = 81
2

10 = 100

The equation x = 9 can be stated as the question, What number multiplied itself is 9?
There are two such numbers, 3 and 3.
Rule

If x R then

x if x 0.
x2 | x |
x if x 0.

In other words, if x is a non-negative real number, then


if x is a negative real number, then
Radicals

x2 x, and

x2 x.
13

For example,

32 3, ( 32 9 3), and
(3)2 (3) 3 ( (3) 2 9 3).

We can conclude that the square root of any real number will always be greater than or equal
to zero. 9 is undefined. Negative numbers have no square root because the square of any
real number cannot be negative.
9 3, since 32 is 9, not (9).
9 3, since (3)2 is 9, not (9).

Note
x = 9 and x2 = 9 have different meanings in the set of all real numbers.
9 = 32 = |3| = 3
If x2 = 9 then x = 3 or x = 3.

EXAMPLE

Solution

EXAMPLE

Solution

Evaluate each square root.


a. 81

b. 1

g. 100

h. 0.09 i. 4

d. 64

b. 1 = 1

d. 64 = 8

e.

g. 100 = 10

h. 0.09 = 0.3

j.

k.

(4)2 16 4

e. 9

(4)2

j.

a. 81 = 9

k.

f. 0.64

4 2

c. 0 = 0

4 2

9 3

f. 0.64 = 0.8
i. 4 is undefined

42 16 is undefined

Evaluate each square root.


a. 100

b. 121

c. 144

d. 169

g. 400

h. 625

i.

j.

1225

e. 225

f. 361

10000

a. 100 = 10

b. 121 = 11

c. 144 = 12

d. 169 = 13

e. 225 = 15

f. 361 = 19

g. 400 = 20

h. 625 = 25

i.

j.
14

c. 0

1225 35

10000 100
Algebra 8

2. Properties of Square Roots


Property

For any real number a and b, where a 0, and b 0,


ab = ab.
For example,

25 16 25 16 5 4 20,
3 27 3 27 81 9,
36 a2 36 a2 6 a ( a 0), and
5 5 5 5 25 5.

Note

b0

If a 0 then

a0

a a = a a =

a . 6 =

a . 6

a2 a.

Mathematics is a universal language.

Simplify each of the following.

Solution

a. 28 = 28 = 16 = 4

b.

7 7 7 7 49 7

c. 502 = 502 = 100 = 10

d.

25 1 25 1 25 5

EXAMPLE

a. 2 8

b. 7 7 c. 502

d. 251

e.

576 36 16 36 16 6 4 24

f.

10 90 10 90 900 30

e. 576

f. 1090

Property

For any real numbers a and b, where a 0, and b > 0,


a
b

If a > 0 then
a
a

a
a

Radicals

= 1 = 1.

For example,

24
6

49

a
.
b

24
4 2, and
6
1
49

1
.
7
15

EXAMPLE

Simplify the expressions.


a.

g.

Solution

25
9

b.

24a 3

h.

6a

50

c.

2
a5 b6
ab

1
64

d.

e.

1
100

f.

x3 y3

a.

25
25 5
=
=
9
3
9

b.

c.

16
16 4

49
49 7

d.

64

f.

625
625 25

144
144 12

e.
g.

h.

i.

1
1
1

100
10
100
24a3

a5 b6
ab2

xy
3

x y

50
2

50
25 5
2

1
64

1
8

24 a3
4 a2 4 a2 2a
6a

6a

625
144

xy

i.

16
49

a5 b6
a4 b4 ( a2 b2 )2 a2 b2
ab2

xy

x3 y3

x2 y2

1
2

x y

1
xy

Property

For any real number a and n Z, ( a )n an

(a 0).

Proof
( a )n a a a ...

n factors of a

a a a ... a

an

n factors of a

For example, ( a )2 a2 a,
( 5)3 53 125, and
( 2 )8 2 8 256 16.
16

Algebra 8

EXAMPLE

Solution

Evaluate
(2)4 + (5)4 (5)2 (2)6.
( 2 )2 ( 5)4 ( 5)2 ( 2 ) 6 2 4 5 4 5 2 2 6
(22 )2 (52 )2 52 (23 )2
22 52 5 2 3 4 25 5 8 16

3. Working with Pure and Mixed Radicals


Definition

A radical expression is an expression of the form


index

a.
radical sign

a
n

radicand

Square roots have index 2. However, we usually write square roots in their shorter form, a:
2

a a

Definition

A mixed radical is a radical of the form


x n a (x Q, x {1, 0, 1})

For example, 32, 67, and 9115 are mixed radicals.


55, 99, and 27 are not mixed radicals. We say that they are pure radicals.
We can convert between mixed and pure radical numbers to simplify radical expressions.
Property

For any real numbers a and b, where a 0 and b 0,


a2 b a b and a b

For example,

a 2 b.

8 4 2 2 2 2 2 2 2 2 2,
27 9 3 32 3 32 3 3 3,
32 16 2 4 2 2 4 2 2 4 2, and
50 25 2 5 2 2 5 2 2 5 2.

Radicals

17

EXAMPLE

Simplify the expressions.


a. 8 + 232 18 + 72 98
b. 248 + 327 108 + 243

Solution

a.

8 = 22 2 = 2 2
2 32 = 2 4 2 2 = 8
18 = 3 2 = 3 2
2

72 = 6 2 = 6 2
98 = 7 2 = 7 2
2

8 2 32 18 72 98
2 2 8 2 3 2 6 2 7 2
2 (2 8 3 6 7) 6 2

b. 2 48 3 27 108 243 2 4 2 3 3 3 2 3

6 2 3 9 2 3

8 3 9 3 6 3 9 3
(8 9 6 9) 3 20 3
EXAMPLE

Solution

Write the numbers as pure radicals.


a. 22

b. 35

c. 53

a. 2 2 2 2 2 2 2 2
b. 3 5 32 5 9 5
c. 5 3 52 3

25 3

4 2

d. 1010

e. xy

45
75

d. 10 10 10 2 10 100 10 1000
e. x y x2 y
Property

For any non-zero real numbers a, b, c, and x,


ax + bx cx = (a + b c)x .

Note
a + b a+b
For example,
9 + 16 = 3 + 4 = 7, but 9 + 16 = 25 = 5.
18

Algebra 8

EXAMPLE

Perform the operations.


a. 3 + 3

b. 25 + 5

c. 36 + 46

e. 50 + 98 + 162
Solution

d. 105 35

f. 5x 9x + 64x

a. 3 + 3 = (1 + 1)3 = 23
b. 25 + 5 = (2 + 1)5 = 35
c. 36 + 46 = (3 + 4)6 = 76
d. 105 35 = (10 3)5 = 75
e. 50 + 98 + 162 =

252 + 492 + 812 = 52 + 72 + 92

= (5 + 7 + 9)2 = 212
f. 5x 9x + 64x = 5x 3x + 8x = (5 3 + 8)x = 10x
EXAMPLE

Solution

Compare the following numbers.


a. 7 ... 3

b. 35 ... 210

a.

b. 3 5...2 10

7...3

c. 27 ... 33

d. 23 ... 32

c. 2 7...3 3

d. 2 3... 3 2

7... 9

32 5... 2 2 10

22 7... 32 3

22 3... 32 2

7 9

45 40

28 27

12 18

2 7 3 3

2 3 3 2

7 3

3 5 2 10

Property

Let a, b, m, and n be four real numbers, satisfying a = m + n and b = m n. Then,


1.

2.

m n a2 b

m n a2 b

( m n)

Proof
1. In order to verify these expressions, suppose that t = m + n.
t2 = ( m + n)2 = ( m + n) ( m + n)
= ( m n) + (m n) + (n m ) + (n n)
(by the distributive property)
= m + ( m n) + (n m ) + n
= m + n + 2mn
a

(by the commutative property)

t = a + 2b t = a 2 b
2

2. We can prove the second part in the same way. Try it yourself.
Radicals

19

EXAMPLE

10

Solution

Simplify the expressions. Use the property to help you.


a.

32 2

b.

52 6

e.

5 21

f.

2 3

a.

3 2 2 2 1 2 1
2+1

c.

c.

6 32

b.

52 6 3 2

21

3+2

d.

64 2

32

6 32 6 2 8 4 2 2 2
4+2

d.

42

6 4 2 6 2 2 2 6 2 2 2 2 6 2 8 4 2 2 2
4+2

42

e. We need a 2 in front of 21 before we can use the property. Therefore, let us multi2

ply the expression by

.
7+3

5 21

2
2

5 21
3+1

f.

2 3

2 (5 21)
2

73

10 2 21
2

7 3
2

7
3

2
2

31

42 3
2

3 1
2

3
1

2
2

Check Yourself 1
1. Simplify the expressions.
a. 2 2
e.

32

b. 8 32
f.

12

2
2. Evaluate the following.

a. (3)2 + (4)4 (5)2 (2)4

c. 3x 12x
g.

a b
3

a b

d. 2 18
h.

1
49

b. (a)4 + (b)2 (c)6

3. Simplify the expressions.


a. 18

b. 50

f. 22 + 32 42
20

c. 48

d. 20

g. 50 18 32

e. 53 23 + 3
h. 12x + 27x 48x
Algebra 8

4. Write each number as a pure radical.


a. 53

b. 35

c. 42

d. 25

e. ab

5. Perform the operations.


a. 6

10
15
2
3
2

b. 52 8

c. 27 48

d.

27
75
12

4
4
4

6. Compare the numbers.


a. 35 and 210

b.

1
and
2

1
3

c. 25 and 33

7. Write each expression in its simplest form.


a.

32 2

b.

c.

62 8

g. ( 6 2 ) ( 8 2 12 )

7 2 10

d.

3 8 e.

h. ( 7 1) ( 8 28 )

i.

f.

94 5

7 48

3 5 3 5

Answers
1. a. 2 b. 16 c. 6x d. 6 e. 4 f. 2 g.
c. 43 d. 25 e. 43 f. 2

1
ab

h.

1
7

2. a. 10 b. a2 + b c3 3. a. 32 b. 52

g. 22 h. 3 x 4. a. 75 b. 45 c. 32 d. 20 e. a2 b

1
1
c. 25 > 33 7. a. 2 1

2
3
b. 2 + 2 c. 5 2 d. 2 + 1 e. 5 2 f. 2 3 g. 4 h. 6 i. 2

5. a. 330 b. 32 c. 3 d. 33 6. a. 35 > 210 b.

EXAMPLE

11

Simplify the following.


a.

Solution

4 21 13 9

b.

6 6 72

1
16
16

c.

1 1

9
16

Start from the radical on the inside of the expression and move outwards.
a. Start with 9, on the inside, and work outwards.
4 21 13 9 4 21 13 3 4 21 16
4 21 4 4 25 4 5 9 3

b.

6 6 72

1
4
1
6 6 72
16 6 6 72
16
16
4
6 6 72

1
6 6 36
2

6 6 6 6 36 6 6 36 6

c.
Radicals

1 1

9
25
5
9 3
1
1

16
16
4
4 2
21

EXAMPLE

12

a. Evaluate
c.

Solution

a a a a ... 7. Find a.

b.

2 2 2 2 ...

x x x ... 5. Find x.

a. Let x 2 2 2 2 ... .
x2 ( 2 2 2 2 ... )2

(take the square of both sides)

x2 2 2 2 2 2 ...

(remove a square root)

x 2x

( 2 2 2 ... x)

x x 2x

x
x

(simplify)

x 2. Therefore,

b.

2 2 2 2 ... 2.

a a a a ... 7
( a a a a ... )2 7 2

( x x x ... )2 5 2

c.

x x x x ... 25

a a a a ... 49

a 7 49

x 5 25
x 20

a7

4. Multiplying Square Roots


To multiply expressions containing square roots, we used the product property of square
roots: a b = a b. We can also use the distributive property of multiplication over
addition and subtraction to simplify the products of expressions that contain radicals.
For example,
28 32 = 2 3 8 2
= 616
= 64

Multiply the rational part by the rational


part and the radical part by the radical
part.

= 24
2 (3 + 22) = 2 3 + 2 2 2
= 6 + 2 2 2
= 6 + 2 2
= 6 + 4
22

Algebra 8

EXAMPLE

13

Solution

Perform the operations.


a. 2(5 + 3)

b. 3(33 + 22)

c. 25(3 + 2 + 25 7)

a. 2(5 + 3) = 2 5 + 2 3 = 2 5 + 2 3 = 10 + 6
b. 3(33 + 22)= 3 33 + 3 22 = 3 3 3 + 2 3 2
= 3 3 + 2 6 = 9 + 26
c. 25(3 + 2 + 25 7)= 25 3 + 25 2 + 25 25 25 7
= 215 + 210 + 425 235
= 215 + 210 + 20 235

EXAMPLE

14

Solution

Multiply and simplify.


a. (2 + 3) (2 + 3)

b. (5 + 5) (5 + 5)

a. (2 + 3) (2 + 3) = 2 2 + 2 3 + 3 2 + 3 3
= 4 + 6 + 6 + 9 = 2 + 26 + 3 = 5 + 26
b. (5 + 5) (5 + 5)= 52 + 2 5 5 + (5)2
= 25 + 105 + 5 = 30 + 105

EXAMPLE

15

Solution

Multiply and simplify.


a. (2 + 1) (2 1)

b. (5 + 3) (5 3)

d. (a + 1) (a 1)

e. (a + b) (a b)

c. (1 22) (1 + 22)

a. (2 + 1) (2 1) = 2 2 2 1+1 2 1 1 = (2)2 12 = 2 1 = 1
b. (5 + 3) (5 3) = (5)2 (3)2 = 5 3 = 2
c. (1 22) (1 + 22) = 12 (22)2 = 1 4 2 = 1 8 = 7
d. (a + 1) (a 1) = (a)2 12 = a 1 (a 0)
e. (a + b) (a b) = (a)2 (b)2 = a b (a, b 0)

EXAMPLE

16

Solution

Radicals

Multiply and simplify.


a.

3 5 3 5

b.

a.

3 5 3 5 (3 5) (3 5) 3 2 ( 5) 2 9 5 4 2

b.

2 2 2 2 (2 2 ) (2 2 ) 2 2 ( 2 ) 2 4 2 2

c.

a b a b

2 2 2 2

c.

a b a b

a2 b
23

EXAMPLE

17

Solution

Multiply and simplify.


a. (3 + 2) (5 1)

b. (5 + 3) (7 + 2)

c. (23 + 1) (5 + 1)

d. (32 2) (5 3)

a. (3 + 2) (5 1)= (3 5) (3 1) + (2 5) (2 1)
= 15 3 + 10 2
b. (5 + 3) (7 + 2) = (5 7) + (5 2) + (3 7) + (3 2)
= 35 + 10 + 21 + 6
c. (23 + 1) (5 + 1) = (23 5) + (23 1) + (1 5) + 1
= 215 + 23 + 5 + 1
d. (32 2) (5 3)= (32 5) (32 3) (25 + 23)
= 310 36 25 + 23

5. Rationalizing Denominators
1

Look at the numbers

10

19

. They are all fractions, and each fraction


5
2
12
13
has an irrational number as the denominator. In math, it is easier to work with fractions that
,

, and

have a rational number as the denominator.


Definition

Changing the denominator of a fraction from an irrational number to a rational number is


called rationalizing the denominator of the fraction. Rationalizing the denominator does not
change the value of the original fraction.
To rationalize the denominator, we multiply the numerator and denominator of the fraction
a
by a suitable factor. For example, if the fraction is in the form
, we multiply both the
b
numerator and the denominator by b.
a

So,

ab

bb

ab
. Note that
b

and

ab
have the same value: they are
b

equivalent fractions.
Look at some more examples:
3
2
3
3

2 2

3 5

24

3 5
2 2

2
3
3

2
2

3 2
2 2

3 3
3 3

3 2
2 2

6
4

6
,
2

3 3
3, and
3

3 52
2 2 2

3 10 3 10
.

22
4
Algebra 8

Definition

An expression with exactly two terms is called a binomial expression. Two binomial expressions
whose first terms are equal and last terms are opposite are called conjugates, i.e. a + b and
a b are conjugates.
If a 0 and b 0, then the binomials xa + yb and xa yb are conjugates. We can use
conjugates to rationalize denominators that contain radical expressions.
1
For example, let us rationalize
. 3 2 is the conjugate of 3 + 2.
3 2
Therefore, we multiply the numerator and the denominator by 3 2 to rationalize the
denominator.
1
3 2

3 2
3 2

1 ( 3 2)
( 3 2 ) ( 3 2 )

3 2
2

( 3) ( 2 )

3 2
3 2

3 2
32
1

Remark

(a + b)(a b) = a2 b2
(a + b)(a b) = a b where a 0 and b 0.
EXAMPLE

18

Rationalize the denominators.


a.

Solution

a.

b.

5
32 2
5
32 2

3 2
2 2 1

6 2
1 3

Radicals

3 2 2
52 5

32 2

2 2 1

32 2
3 2 2

6 2

d.

1 3

5 (3 2 2 )
(3 2 2 )(3 2 2 )

3 2 2
52 5

5 3 5 2 2
3 2 (2 2 ) 2

3 5 2 10 3 5 2 10

3 5 2 10
98
1

( 3 2 ) (2 2 1)
3 2 2 3 1 2 2 2 2 1

(2 2 1) (2 2 1)
(2 2 ) 2 12

d.

c.

c.

3 2

b.

2 6 32 2 2 2
8 1

2 6 34 2
7

( 6 2 ) (1 3)
6 6 3 2 1 2 3

(1 3) (1 3)
12 ( 3) 2
6 18 2 6
6 3 2 2 6 2 6 4 2

6 2 2
1 3
2
2

(3 2 2) (5 2 5) 3 2 5 3 2 2 5 2 5 2 2 5

(5 2 5) (5 2 5)
5 2 (2 5) 2

15 2 6 10 10 4 5 15 2 6 10 10 4 5

25 20
5
25

EXAMPLE

19

Rationalize the denominators to find the sum.


3

3 2 2
3

Solution

3 2 2

2
32 2

3
32 2
2
3 2 2

3 2 2 3 2 2
32 2 32 2
3 2 2
2

( 3 ( 3 2 2 )) ( 2 ( 3 2 2 ))
( 3 2 2) ( 3 2 2)
( 3 3) ( 3 2 2 ) ( 2 3) ( 2 2 2 )
( 3)2 (2 2 )2
32 6 6 4 7 6
6 7

38
5
5

Check Yourself 2
1. Rationalize the denominators and simplify.
a.

g.

3
7

b.

a b

h.

2 10

c. 2

d. 5

2 x y

i.

a3

1
2

e.

3 5

3 5

f.

2 6

a 3 b4

j.

x3 y3

a b2

2. Rationalize the denominators and simplify.


a.

g.

b.

2 1
2 52

3 2
2 5 7

h.

2 10 6

c.

i.

7 5

d.

5 1
a

6 2

a b

j.

a b

a b

5 1

e.

f.

5 1

3 2
2 3 2

k.

3 32 7

3. Rationalize the denominators and simplify.


a.
d.

1
5 2
2
2 1

b.

5 2
2
2 1

e.

2 2

2
2 2
3

2
2 2

3 1 1 3

c.

3 3

1
3 3

3
3 3

Answers
1. a.

15
21
b.
3
7

b. 6 2 c.
i.
26

30
4

c. 2 d. 15 e. 15 f.

5 1
3 6 6
d.
2
2

e.

3 5
2

f.

g.

4 6
10

10
20

h.

b
a

i.

2
xy

j. ab 2. a. 2 + 1

g. 52 10 + 3 5 3 h.

6
16 3 2
a ab
a b 2 ab
2 5
j.
k. 93 67 3. a.
b. 2 c.
d.
6
4
3
ab
ab

35 3
2

e. 17 3 3
6
Algebra 8

EXERCISES

1 .1

1. Evaluate the square roots.


a. 36
16x2

d.

5. Perform the operations.

b. 100
e.

c. 121

25 y2

f.

121 a4

a. 33 + 23

b. 65 + 5

c. 5x + 5x

d. 6 36

e. 318 + 272

f. 80 125 + 45

g. 75 + 108 48 + 27

2. Simplify the expressions.


a. 3 3

b. 5 5

c. 3 12

d. 3 27

e. 2x 8x

f.

g.

h. 3 2 x 4 18 xy2

3a 5a

6 xy 24xy

h.

9x3 16 x3 4 x 25 x

i.

2.25 2.89 1.44

j.

0.9 2.7 1.7 0.4

3. Simplify the expressions.


50

a.

d.

72 x3
2 2x

b.

e.

72
8
32 x3 y2
24 x

c.

f.

108

6. Write each expression in its simplest form.

27

3xy3

a.

32 2

b.

52 6

c.

8 2 12

d.

11 96

e.

82 7

f.

5 24

g.

74 3

h.

3 8 3 8

i.

4 15 4 15

f.

a 2 8a a

k.

2 2 4 12

12 x y

4. Write each number as a mixed radical.


a. 8

b. 72

c. 243

d.

e. 125

f.

Radicals

1000

x3 y2

27

7. Simplify the expressions.


a.

32 21 23 4

b.

7
2
6 9
3
9

c.

d.

8 16

10. Rationalize the denominators.


a.

3
3

d. 1 2
4 3

1
9 16
96

g.
j.

13 6 6 9

3 2

b.

e.

3 1
3 1
3

4
3 3 2

2 2

k.

2 3

f.

1 2

h.

c.

3 5

i.
l.

1
11
2
3 2

10
2 7 5
10 2 21
7 3

8. Find x in each equation.


b. 3 3 3 3 ... x

a.

2 2 2 2 ... x

c.

3x 3x 3x ... 9

9. Find the products.


a. 5 (2 + 3)
b. 7 (1 + 7)
c. 2 (3 8 + 1)

11. Perform the operations.


a.
b.
c.
d.

d. 2 (8 + 32)

1
3

3
3 1

3 1

5
2 3 11

4 2
3 2 2

2
2 3 11

2 3

32 2

e. 6 (23 + 32)
f. (3 + 5) (3 5)
g. (22 3) (22 + 3)
h. (23+2) (23 2)
i. (12 + 8) (3 2)
j. (12 + 22) (2 + 3)
k.
l.
m.
28

2 1

2 1

12. Perform the operations.


a.

52

5 1

1 2

b.

2 11 72

2 33 2 33
5 2 3 2 3 16 9 3

5 2

c.

1
2 1

1
3 2

1
4 3

...

1
100 99
Algebra 8

Objectives

After studying this section you will be able to:


1. Understand the concepts of nth root and rational exponent.
2. Write numbers in radical or rational exponent form.
3. Understand the properties of expressions with rational exponents.
4. Use the properties of rational exponents to solve problems.

A. RATIONAL EXPONENTS
1. nth Roots
In section 8.2 we studied exponential equations.
For example, 2n 2n = 2 is an exponential equation. Let us solve it.
2n 2n = 22n = 2

(an am = an+m)

22n = 21
2n = 1, n =

1
1
for n in the original equation we will get 2 2 2 2 2,
. If we substitute
2
2
1

but we know that 2 2 = 2. Therefore, 2 2 2 2 2.


Remark

Let x R {1, 0, 1}.


If xm = xn then m = n.
Definition

For any natural number n and a, b R.


1

n
If an = b then a = b n b. a is called the nth root of b. It is denoted by b.
n

In the expression a, a is called the radicand and n is called the index.


n

Remember that we do not usually write the index for square roots:
a = a.

Look at some examples of different roots:


52 = 25 and 5 = 25

the square root of 25 is 5,

2 = 8 and 2 = 8

the cube root of 8 is 2,

3 = 27 and 3 = 27

the cube root of 27 is 3, and

2 = 16 and 2 = 16

the fourth root of 16 is 2.

3
3
4

Radicals

3
4

29

2. Rational Exponents
Definition

If m and n are positive integers (n > 1), and b is a non-negative real number,
n

m
is called a rational exponent.
n
2

bm b n .

For example, the numbers 8 3 , 4 2 , and 2 2 have rational exponents.


EXAMPLE

20

Write the expressions in radical form.

Solution

b. 32 5

Solution

f.

( a 3 )2

c. 5 3 3 52 3 25

b. 32 5 5 32

21

e. ( x4 ) 4

a. 3 2 2 3 3

d. x 4

c. 5 3

d. x 4 4 x3
EXAMPLE

a. 3 2

e. ( x4 )4 x4

f.

( a 3 )2

Write the expressions with radical exponents.


3

a.

35

a.

35 3 5 31 3

d.

a2 a 3

b.

c.

64

d.

x14

a2

b.

64 43 4 3 41 4

e.

ab2 ( ab 2 )5 a 5 b 5

e.

ab2
7

c.

14

x14 x 7 x2

Rule

If b is any real number and n is a positive integer (n > 1):


1. If b > 0 then b is a positive real number.
n

2. If b = 0 then b is zero.
n

3. If b < 0 and n is odd, then b is a negative real number.


n

4. If b < 0 and n is even, then b is not a real number.


n

EXAMPLE

22

Solution

30

Simplify the following.


b.

a.

a.

8 23 2 3 2

c.

81

(8 > 0)
3
3

27

d.
b.

e.

16
4

81 34 3 4 3

c.

27 3 (3)3 (3) 3

(27 < 0 and 3 is odd)

d.

16 is not a real number

(16 < 0 and 4 is even)

e.

0 =0

(81 > 0)

Algebra 8

Check Yourself 3
1. Write the expressions in radical form.
1

a. 2 2

b. a 3

c. a 3

d. x 5

e. ( x2 y3 )6

g. ( a2 b2 ) 2

( x2 )3

f.

h. (( x y) x2 ) 3

2. Write the expressions with rational exponents.


3

a.

b.

c.

a2

d.

x y2

e.

625 e.

x4 f.

a8

g.

512

g.

x2 y4 h.

12

x6 y4

3. Simplify the expressions.


a. 16

b.

27 c.

64

d.

64 f.

x2 y4 h. 4 16 a4 b8

Answers
1. a. 2 b. 3 a c. 3 a2 d. 5 x3 e. 6 x2 y3
1

b. 2 2

c. a 5

d. ( xy2 )3

f. 6 x g. a2 b2
1

h. 3 ( x y)x2

e. x 3 f. a2 g. ( x2 y4 )3 h. ( x3 y2 )6

2. a. 2 3

3. a. 4 b. 3 c. 4 d. 5 e. 2

f. 2 g. xy2 h. 2ab2

B. PROPERTIES OF RADICALS
Property

For all real numbers a and b, where a > 0 and b > 0, and for any integer m and n, where
m > 1 and n > 1:
1.

2.

an a

if n is odd.

5.

an | a |

if n is even.

6.

a b

a b n a b

3.

n m

b
n

7.

a
b

4. a b

8.
an b

and

an b a

am

m n

mn

mn

mn

an
bm

mn

an

mn

mn

bm

m n

an bm

an
bm

Look at the examples of each property.


1.

Radicals

b.

32 5 (2)5 2

d.

(3)4 | 3| 3

a.

27 33 3

c.

16 2 4 | 2| 2

31

2.

a. 5 4 = 54 = 20
5

c.
3.

8
4 2
2

2
4

4.

64 x2
4

4x

b. x x

5.

81 3
27 3
3

x4 x

24

x5
3

4 22
3

6.

81

23 2

x3 y6 z2 3 ( x y2 )3 z2 x y2 z2

a.

64 x2 4
4
4
4
16 x 4 2 4 x 2 4 x 2 x
4x

2 4 324 8

b.

a. 2 3 3 2 3 3 3 8 3

c.

x5 y5 xy

a.
c.

b.

5 3
3

23

34

7.

a.

8.

a.

c.

x8

34

53

24
33

3 2

32

b.

22 2

2 3

12

32

5 3 32

125 9

b.

x8

3 2

52 25

32

a2

1125
3

2 4 12 16

27
33

5 5

4 2

a
b

b.

4 3

32

b3

12

4 3 2

a2
b3

12

24

12

x8 x

Check Yourself 4
1. Simplify the expressions.
a.

b.

16

(2)4

c.

(3)5

d.

128

c.

x 2 x3

d.

x 3 x2 y2

x5

e.

27x3

2. Perform the operations.


a. 3 12

b.

3 9

3. Simplify the expressions.


3

a.

b.

625
3

c.

d.

3
3

d.

x5 y6

d.

32 16

4. Simplify the expressions.


a.

b.

40

81

c.

32

c.

a2 a3

5. Perform the operations.


a.
32

2 4

b.

x x3

Algebra 8

6. Simplify the expressions.


3

a.

b.

c.

a3

d.

a2

x4

x3

7. Write each expression in its simplest form.


3

a.

b.

5 4

c.

3 3 3

x27

d.

x16

e.

3 3

Answers
1. a. 2 b. 2 c. 3 d. 2 e. 3x 2. a. 6 b. 3 c. x d. x 3 y2
4. a. 2 3 5 b. 3 3 3 c. 2 4 2
6. a. 12

256
27

b.

9
2

c. 20 a7

d. xy 4 xy2
d. 24 x7

5. a. 2 6 2
7. a. 6 3

3. a.

1
2

b. 5 c. x d. 3 9

b. x 12 x c. a 6 a

d. 4

12

27

b. 20 x c. x d. x2 e. 9 81

C. RADICAL EQUATIONS
Definition

An equation that has a variable in a radicand is called a radical equation.


For example, the equations x = 25,

x 1 3, and

2 x 1 3 x 5 are radical equations.

Let us look at some examples of how to solve radical equations.


EXAMPLE

23

Solution

Solve

x 1 3.

( x 1)2 32 (take the square of both sides)

Check:

x 1 9

8 13
?

9 3

x8

33

Therefore, 8 is a solution.
EXAMPLE

24

Solution

Solve

3x 1 5.

( 3x 1)2 52
3x 1 25
3x 24
x8

Radicals

Check:

3 8 15
?

25 5
55

Therefore, 8 is a solution.
33

EXAMPLE

25

Solution

2 x 3 3 8.

Solve

2x 3 3 8

Check:

2 11 3 3 8

2x 3 8 3

25 3 8

2x 3 5

( 2 x 3)2 5 2

5 3 8

2 x 3 25

88

2 x 22

Therefore, 11 is a solution.

x 11
EXAMPLE

26

Solution

Solve

x2 12 x 6.

( x2 12 )2 ( x 6)2

Check:

(2)2 12 2 6
?

x2 12 x2 12 x 36

4 12 4

12 x 24

16 4

x 2

44

Therefore, 2 is a solution.
EXAMPLE

27

Solution

Solve

3x 2 5.

( 3x 2 )3 5 3

Check:

3x 2 125

3 41 2 5
3

3x 123

123 2 5
3

x 41

125 5
5 5

Therefore, 41 is a solution.
EXAMPLE

28

Solution

Solve

4x 1 5x 1 0.

4x 1 5x 1 0
( 4 x 1)2 ( 5 x 1) 2
4x 1 5 x 1
x2

Check:

4 2 1 5 2 1 0
?

9 9 0
?

3 3 0
0 0

Therefore, 2 is a solution.
34

Algebra 8

EXAMPLE

29

3x 1 3x 6 5.

Solve

3x 1 5 3 x 6

Solution

(take the square of both sides)

( 3x 1)2 (5 3 x 6 ) 2

3x 1 25 2 5 3 x 6 3 x 6

(simplify)

10 3x 6 30
( 3 x 6 )2 3 2
3x 6 9
3x 3
x1

Check:

3 1 1 3 1 6 5
?

4 9 5
?

2 35
5 5

Therefore, 1 is a solution.

Check Yourself 5
1. Solve each equation and check your answer.
a. x = 5

b. x + 1 = 3

d. x = 3

e.

2x 1 7

f.

3 x 1 1 8

g.

3x 5 4 6

h.

5x 1
30
2

i.

2x 6 x

j.

4x 3 2 x 2 0

k.

x3 x2 5

l.

x2 1 2 x

n.

x 2

o.

m.

x 1 3

p.

x 2x 1

c. x+1 = 6

3x 2 5x

Answer
1. a. 25 b. 4 c. 35 d. e. 25 f. 8 g. 3 h.
Radicals

1
17
i. 2 j.
k. 6 l. 3 m. 26 n. 22 o. 1 p. 1
2
5
4
35

EXERCISES

1 .2

1. Write the expressions in exponential form and


simplify if possible.
a. 21
3 5

e.

b.
f.

c.

( 3)6

g.

c. 7x = x

1
3

3 3
c. ( ) 2
2

b. b 3

d.

4x 6

5x 4 6

f.

7x 6 5

g.

5x 1 3 7

h.

2x 3 2 4

i.

x2 5 x 1

j.

x2 9 5

7x 6 4

l.

2x 5 x 4

x x 16 2

n.

e.

d. x

3
2

ab
c

3. Simplify the expressions.


3

b.

4 5

d.

5 5

e.

g.

27

h.

j.

2 4
3

m.

p.

k.

250

q.

c.

1016

243

f.

(5)4

i.

a3 a2

3 27
21

n.

3 9 3

a2

4 3

3 3

o.

r.

3
3

s.

c.
e.

4 3

1
9

8x 2 6

1
4x 3

2 x

2 x

11 3 4 27

d.

17 6 8

f.

2 x1

6. Simplify the expressions.


3

2 3 6

108

33x 6

b.

x 1 2 x x 1 2 x 8
3

q.

3 2

p.

162

l.

o. 6x x= 5

4. Perform the operations.


a.

m.

64x6

e. ( a )6

k.

a.

3x 14 8

b.

2. Write the expressions in radical form.


a. a 2

2x 1 3

a.
2

d. a x

5. Solve the equations.

7 3
10
2
9
27

( 2 6 )( 2 3 )

a. 23 24

b. 53 52

c. 43 42 4

d. 26 24 22

e. 3x 32x 33x+1

f. 2x 3x 5x

g. (x y)2 (y x)3 (x y)4


h. 298 + 298 + 298 + 298

36

Algebra 8

7. Simplify the expressions.


a.

25
23

b.

x 1

d.

10. Solve the equations for x.

53
54

c.
3

a
b x 1

e.

(2)
(2)4

f.

a. 4x+1 = 23x 4

ax 1
ax 2
5 2
104

c. 9x 2 (

b. 3x 4 = 81

1 3 x
)
27

d. 8x+1 = 42x 3

e. (2x 3)3 = (x + 1)3 f. (2x 4)6 = x6


g. 5 23x 1 23x+1 = 256

8. Perform the operations.


a. (2)5 (2)3

b. 32 (3)3 (3)

c. (4)2 (22)3 (22)4

d.

(2 3 )3 (2 2 )6
1
( )3
2

e. (a)7 (a4) (a)2

h.

12 x 12 x 12 x
81
4x 4x 4x

i.

3 4x 3 4 x 1

j.

3 93 x 1
((3)2 ) 4
81 (243) x

22 x
128
4

11. 3a = 25 and 3b = 5. Find

a
.
b

12. 3x = 4. Find 92x + 3x+1.

f. (a3)2 (a2)3 (a1)1

13. 2x = 3y = a. Write (12)xy in terms of a, x, and y.


14. 2a 3 and 4b + 7 are integers with 32a 3 = 54b + 7.
Find a + b.

9. Simplify the expressions.


2

15. (x + 1)x

a. 2x 1 + 2x 1 + 2x 1 + 2x 1

16

= 1. Find the sum of the possible

values of x.

b. ax + 2 ax 3ax
c. 3x+1 + 3x 1 + 3x+2 3x 2

16. 44 = 16x. Find x.

10 x 10 x 10 x 10 x
d.
5x 5x 5x 5x

e.

(2 7 35 )2
36 7

f.

313 311 39
312 310

g. (1)101 (1)125 (1)100 (1)99 (1)49


h.

4 0

((39) (2) ) (1)


(3)125 6 127 2126

Radicals

17. |x + y 3| + |x y 1| = 0. Find x.
18. Simplify |x 4| + 2|3 x| if 3 < x < 4.

2003

37

19. Find the sum of the possible values of x that


satifsfy the equation |(|2x + 4|) 2| = 4.

22 . x 72 72 72 ... and
y 42 42 42 ... . Find x + y.

20. Find the product of the possible values of x that


satisfy the equation |2x + 3| = |5 3x|.

23 . 2a 2a 2a ... 12 12 12 ... 3.
Find a.

21. Simplify the expressions.


3

a.

(3)2 8 16

24 .Simplify

72 48

b.

2.89 6.25 1.96


2

d.

5 1

2 4

4 6

...

1
252 254

1
254 256

27. How many different natural number solutions

12 2 16 2

g.

7 2

h.

does the equation x + y = 300 have?


5

2 7

28. ( 2 1)7 57122 57121. Find (2 1)7.

4 12 4 12

(Hint: (

1
2 1

) 2 1 )

( 6 20 ) ( 5 1)
2

x y x y
2

k.

38

f.

m.

18 3 29 16

e.

l.

25 . x2 x2 x2 ... 2. Find x.
26 .Simplify

5 1
3

j.

27 11 2 30 30 30 ... .

54

c.

i.

xy
if x < y < 0
xy

2.7 5.4 7.1 2 0.2


4

15 6

15 6 3

4 7 4 7

29.

15
1
9
16
9 24 16 . Find x .
8
x
x
x

30. Evaluate

31.

(50 51 52 53) 1.

x 2 x 2 4. Find x2.

(Hint: (a b)3 = a3 3ab(a b) b3)


Algebra 8

CHAPTER REVIEW TEST

1A

1. Simplify (|((2)3) 2|) [3 ||(3)2 3||].


A) 8

B) 12

C) 8

D) 28

6. |x 2| = |x + 3|. Find the value of x which satisfies the equation.


A)

2. Calculate |1 2| + |1 + 2|.
A) 0

B) 2

C) 2

1
4

D) 22

3. How many elements are there in the solution set


of |x 2| + 1 = 0?
B) 1

C) 2

D) 3

B) 2x 2

C) 2

C) 1

D)

D) 4

5. ||x + 1| 5| = 3. Find the sum of the possible

following numbers is negative?


B) a2

C) a3

D) (a)3

8. Simplify ( )3 .
2
A)

1
8

B)

1
8

C) 8

D) 8

A) 23x

10. Simplify

B) 24x

C) 29x 1

D) 28x+1

4m 4m
.
2m 2m 2m 2m

values of x.
A) 3
Chapter Review Test 1A

3
2

9. Simplify 4x+1 8x 1 16x.

4. 1 < x < 3. Calculate |x + 1| + |x 3|.


A) 2x

1
2

7. a is a positive real number. Which one of the


A) a1

A) 0

B)

2 m 1

B) 2

C) 2

D) 4

A) 22m

B) 2m +1

C) 2m 1

D) 2 m 2
39

11.

16. Evaluate (32 12) (18 + 23).

3x 1 3x 2 3x 3
27. Find x.
39

A) 1

12. 2 a b 2

B) 3

1
2 2 a b 1

A) 0

C) 13

D) 27

17. Evaluate

0. Find a.

B) 1

C) 3

D) 3

13. a = 2x, b = 3x, and c = 5x. Express 90x in terms


of a, b, and c.
A) (abc)

15. Evaluate
A) 1
40

A) 3

18. Evaluate
A) 1

B) a bc

C) ab c

B) 82

D) 262

2.25 2.89 1.44.

B)

1
2

C) 1

D) 6

4 12.

B) 3

C) 3 1

D) 3 + 1

2 2 4 12 .

B) 3

C) 3 1

D) 3 + 1

D) a b c

19. Evaluate

C) 38

C) 6

B) 180

4 2

14. Evaluate 38 22 + 32.


A) 6

A) 36

A)

3
2

20. Evaluate
D)

2
3

A) 7

27 m3 9 m2
.
12 m3 4 m2

B) 2

5
7 2

B) 27

9
4

C)

5
7 2

D) 3m+1

C) 57

D) 5
Algebra 8

CHAPTER REVIEW TEST


1. Evaluate
A)

(1)55 (172 ) (1) 64


.
(121 ) (1)82 (1)15

3
2

2. Evaluate

1B

B) 3

6. Evaluate

C) 1

D) 1

2 2 41
.
2 2 31

1
B)
3

A) 3

A)

569 568
.
568 567

1
6

5
6

B)

C)

5
3

D)

10
3

7. 3x = 5, 2y = 3, and 5z = 0.125. Find x y z.


A) 3
C) 2

B) 10

C) 3

D) 30

C) 3

D) 4

D) 6

8. 3x+1 + 2 3x 1 = 33. Find x.


3

3.

x(2 )
16. Find |x|.
( x2 )3

A) 2

B) 4

A) 1
C) 8

B) 2

D) 16

9. a = (52)3, b = 5(2 ), and c = 5(3 ). Which statement


is true?

4. What is half of 220?


A) 210

B) 221

5. Find the simplest form of


A) 3x

B)

Chapter Review Test 1B

1
3x

C) 219

D)

1
210

3x 3x 3x
.
9x 9x 9x

C) 32x

A) a > b > c

B) a > c > b

C) b > c > a

D) c > b > a

10. k = 1254 642. How many digits are there in the


number k?
D) 3x+1

A) 10

B) 11

C) 12

D) 13
41

11.

32 x 243x
9x 3. Find x.
81x1

A)

2
9

B) 2

41
A)
16

12 48 27
75 2 3

A) 23

15.

B) 1

C) 1

B) 2

19. Evaluate
C) 3

D) 4

A) 8
6

11
24

B)

4
7

20.
C)

11
7

D)

1
6

8 x 1

3
7

42 x 3

A) 9

B) 2

C) 2

11 2 30 8 2 15
32 2

D) 4

B) 6 3
D)

3 5 3 5
2

A) 5

D) 33

13 13 64 .

2 2

C) 3

18. Evaluate

2 2

3 15

A)

2
D)
3

x x2 x xn . Find n.

A)
42

3
C)
2

A) 22

D) 10

17. Evaluate

41
B)
4

13. Simplify

A) 1

C) 9

9
1 1 .
16

12. Evaluate

14. Evaluate

16. Evaluate

B) 25

3 2

C) 2

D) 1

2 4 2 4 ... .

B) 2

C) 8

D) 8

C) 114

D) 120

2. Find x.

B) 105

Algebra 8

Objectives

After studying this section you will be able to:


1. Define statistics as a branch of mathematics and state the activities it involves.
2. Describe some different methods of collecting data.
3. Present and interpret data by using graphs.
4. Describe and find four measures of central tendency: mean, median, mode, and range.

A. BASIC CONCEPTS
1. What is Statistics?
Statistics is the science of collecting, organizing, summarizing and analyzing data, and drawing conclusions from this data. In every field, from the humanities to the physical sciences,
research information and the ways in which it is collected and measured can be inaccurate.
Statistics is the discipline that evaluates the reliability of numerical information, called data.
We use statistics to describe what is happening, and to make projections concerning what will
happen in the future. Statistics show the results of our experience.
Many different people such as economists, engineers, geographers, biologists, physicists,
meteorologists and managers use statistics in their work.

Definition

statistics
Statistics is a branch of mathematics which deals with the collection, analysis, interpretation,
and representation of masses of numerical data.
The word statistics comes from the Latin word statisticus, meaning of the state.
The steps of statistical analysis involve collecting information, evaluating it, and drawing conclusions.
For example, the information might be about:

44

what teenagers prefer to eat for breakfast;


the population of a city over a certain period;
the quality of drinking water in different countries of the world;
the number of items produced in a factory.
Algebra 8

The study of statistics can be divided into two main areas: descriptive statistics and inferential
statistics.
Descriptive statistics involves collecting, organizing, summarizing, and presenting data.
Inferential statistics involves drawing conclusions or predicting results based on the data
collected.

2. Collecting Data
We can collect data in many different ways.

a. Questionnaires
A questionnaire is a list of questions about a given topic. It is usually printed on a piece of
paper so that the answers can be recorded.
For example, suppose you want to find out about the television viewing habits of teachers.
You could prepare a list of questions such as:

Do you watch television every day?


Do you watch television: in the morning?
in the evening?

What is your favourite television program?


etc.
Some questions will have a yes or no answer. Other questions might ask a person to choose
an answer from a list, or to give a free answer.
When you are writing a questionnaire, keep the following points in mind:

1. A questionnaire should not be too long.


2. It should contain all the questions needed to cover the subject you are studying.
3. The questions should be easy to understand.
4. Most questions should only require a Yes/No answer, a tick in a box or a circle round a
choice.

In the example of a study about teachers television viewing habits, we only need to ask the
questions to teachers. Teachers form the population for our study. A more precise population
could be all the teachers in your country, or all the teachers in your school.

Statistics and Graphics

45

b. Sampling
A sample is a group of subjects selected from a population.
Suppose the population for our study about television is all
the teachers in a particular city. Obviously it will be very
Xdifficult to interview every teacher in the city individually.
Instead we could choose a smaller group of teachers to
interview, for example, five teachers from each school.
These teachers will be the sample for our study. We could
say that the habits of the teachers in this sample are
probably the same as the habits of all the teachers in the city.

population

sample

A sample is a subset of a population.

The process of choosing a sample from a population is called sampling.


The process of choosing a sample from a population is called sampling.
When we sample a population, we need to make sure that the sample is an accurate one. For
example, if we are choosing five teachers from each school to represent all the teachers in a
city, we will need to make sure that the sample includes teachers of different ages in different
parts of the city. When we have chosen an accurate sample for our study, we can collect the
data we need and apply statistical methods to make statements about the whole population.

c. Surveys
One of the most common method of collecting data is the use of surveys. Surveys can be carried out using a variety of methods. Three of the most common methods are the telephone
survey, the mailed questionnaire, and the personal interview.

3. Summarizing Data
In order to describe a situation, draw conclusions, or make predictions about events, a
researcher must organize the data in a meaningful way. One convenient way of organizing the
data is by using a frequency distribution table.
A frequency distribution table consists of two rows or columns. One row or column shows the
data values (x) and the other shows the frequency of each value (f). The frequency of a value
is the number of times it occurs in the data set.
For example, imagine that 25 students took a math test and received the following marks.

46

10

10

6
Algebra 8

The following table shows the frequency distribution of these marks. It is a frequency distribution table.
mark

(x)

10

frequency

(f)

Note
The sample size is the number of elements in a sample. It is denoted by n.
We can see from the table that the frequency of 7 is 3 and the frequency of 8 is 5.
The sum of the frequencies is equal to the total number of marks (25).
The number of students took test is called the sample size (n). In this example the sample
size is 25.
The sum of the frequencies and the sample size are the same.
EXAMPLE

Twenty-five students were given a blood test to determine their blood type. The data set was
as follows:
A

AB

AB

AB

AB

AB

Construct a frequency distribution table of the data and find the percentage of each blood
type.
Solution

There are four blood types: A, B, O, and AB. These types will be used as the classes for the
distribution. The frequency distribution table is:
class

frequency

percent

16 %

28 %

36 %

AB

20 %

Total 25

Total 100

We can use the following formula to find the


percentage of values in each class:
f
% 100% where
n
f = frequency, and
n = total number of values (25).
For example, in the class for type A blood, the
percentage is
4
100% 16%.
25

Statistics and Graphics

47

B. PRESENTING AND INTERPRETING DATA


A graph is a diagram
that relates two or more
different types of
information.

When we have collected, recorded and summarized our data, we have to present it in a form
that people can easily understand.
Graphs are an easy way of displaying data. There are three kinds of graph: a line graph, a bar
graph, and a circle graph (also called a pie chart).

1. Bar Graph
The most common type of graph is the bar graph (also called a histogram). A bar graph uses
rectangular bars to represent data. The length of each bar in the graph shows the frequency
or size of a cooresponding data value.

48

Mark

Maths

Physics

Chemistry

Biology

Computer

10

History

Music

Music

History

Computer

10
9
8
7
6
5
4
3
2
1
0
Mathematics

Then we can draw bars to show the


marks for each subject.

Subject

Biology

We begin by drawing a vertical scale


to show the marks and a horizontal
scale to show the subjects.

Chemistry

Solution

The following table shows the marks that a student


received at the end of the year in different school subjects.
Draw a vertical bar graph for the data in table.

Physics

Marks

EXAMPLE

Lessons

Algebra 8

2. Line Graph
We can make a line graph (also called a broken-line graph) by drawing line segments to join
the tops of the bars in a bar graph.

Music

History

Computer

Biology

Chemistry

Physics

Music

History

10
9
8
7
6
5
4
3
2
1
0
Mathematics

Lessons

Computer

Biology

Chemistry

Physics

Marks

10
9
8
7
6
5
4
3
2
1
0
Mathematics

Marks

For example, look at the line graph of the data from Example 5.2.

Lessons

To draw the line graph, we mark the middle point of the top of each bar and join up the points
with straight lines.

EXAMPLE

Statistics and Graphics

The following table shows the number of cars produced


by a Turkish car company between 1992 and 2000.
Draw a bar graph and a line graph of the data in this
table.

Car Production
Year

Production

1992

110 659

1993

133 006

1994

99 326

1995

74 862

1996

65 007

1997

91 326

1998

88 506

1999

125 026

2000

140 159

49

160
140
120
100
80
60
40
20
1997

1998

1999

2000

1997

1998

1999

2000

1996

1995

1994

1993

0
1992

Number of cars (10 000)

Year
160
140
120
100
80
60
40
20
1996

1995

1994

1993

0
1992

First we need to choose the axes.


Let us put the years along the
horizontal axis and the production
along the vertical axis of the graph.
It will be difficult to show large
numbers such as 133 006 on the
vertical axis. Instead, we can
choose a different unit for the
vertical axis, for example: one unit
on the axis means 10 000 cars. We
write this information when we
label the axis.

Number of cars (10 000)

Solution

Year

6%

6.4%
5.3%

5.2%

5
4

6%

3.4%

2
1

Germany

0
Canada

a. Which country spent the largest percentage of


its GDP on education?

7%

Australia

domestic product. Look at the graph and answer


the questions.

Share of education expenditures as a percentage of GDP in


selected countries *

Norway

The graph below shows the amount of money


that seven different countries spend on education
in 2003, as a percentage of each countrys gross

United
Kingdom

The gross domestic product (GDP) of a country is


the total value of new goods and services that the
country produces in a given year.

USA

Turkey

EXAMPLE

* Source: Education at a Glance 2003 - OECD indicators

b. Which country spent the smallest percentage of


its GDP on education?
c. Find the percentage difference between the
countries which spent the largest and smallest
percentage of their GDP on education.
d. Which countries spent the same percentage of
their GDP on education?
50

Algebra 8

Solution

a. The USA spent the largest percentage (7% of its GDP).


b. Turkey spent the smallest percentage (3.4% of its GDP).
c. 7 3.4 = 3.6%
d. Norway and Australia spent the same percentage: both countries gave 6% of their GDP.

Practice Problems
1. The bar graph below compares different causes of death in the United States for the year
1999. Look at the graph and answer the questions.
Comparative causes of annual deaths in the United States (1999)*

Cause
* Source: World Health Organization

a. What was the most common cause of death?


b. What was the least common cause of death?
c. What is the ratio of the number of deaths caused by smoking to the number of deaths
caused by alcohol?
d. How many deaths are shown in the graph?
e. On avarage, how many people died per day from each canse in 1999? (Hint: There
were 365 days in 1999.)

Answers
1. a. Smoking b. Drug Induced c.
Statistics and Graphics

16
3

d. 632000 e. 1732

51

EXERCISES

2 .1

1. The set of quiz scores in a class is as follows.


8

6 10 4

7 6

6 8

4 10 8

Construct a frequency distribution table for this


data.

2. A students expenses can be categorized as shown


in the table.
Expenses
Food
Rent

4. The following table shows the amount of sea fish


caught in Turkey in 2003.
Fish
Anchovy

Quantity (1000 tons)


416

Horse Mackerel

295

Scad

16

Gray mullet

12

Blue fish

11

Pilchard

11

Whiting

12

Hake

Other

32

Source: Turkeys Statistical Yearbook 2004

Present this information in a circle graph.

Percent of total income.


30%
27%

Entertainment

13%

Clothing

10%

Books

15%

Other

5%

Present this information in a bar graph.

5. The following bar graph shows the hazelnut


production in Turkey from 1999 to 2003. Use the
graph to answer the questions.
700 000

Hazelnut production in Turkey (tons)

600 000
500 000

100 000

2003

200 000
2002

Number of class members


8

2001

Sport
Football

300 000

2000

chosen by each of forty students in a class.

400 000

1999

3. The following table shows the favorite sport

Basketball

Volleyball

a. Estimate the total production for all five years.

Swiming

12

Wrestling

b. Which year had the highest production?

Karate

Judo

Present this information in a circle graph.


52

c. Find the combined production for 2002 and


2003.
d. Draw a broken line graph of the data.
Algebra 8

Definition

An equation that can be written in the form


a0

ax2 + bx + c = 0,
is called a quadratic equation.
Im sick of being
an unknown

In the equation, a, b, and c are real number coefficients and x is a variable. A quadratic equation written in the form ax2 + bx + c = 0 is said to be in standard form. Sometimes, a quadratic equation is also called a second degree equation.
For example,

1
x+3=0
2
are all quadratic equations. By the definition of a quadratic equation, a cannot be zero.
However b or c or both may be zero. For instance,
2x2 = 0 and x2 9 = 0
3x2 + 5x = 0,

2x2 x 1 = 0 and 2x2

x2 + 3x 5 = 0,

are also quadratic equations.


We can see that quadratic equations are formed by second-degree polynomials. Polynomials
of a different degree do not form quadratic equations.
Let us look at the coefficients a, b, and c of some quadratic equations.
Equation

3x2 + 5x 9 = 0

1 x + 3x2 = 0

2x2 + 5x = 0

1
2

1 x2 = 0

(3 + 1)x2 = 0

3 + 1

x2 x 1
+ =0
2 3 4

1
2

x2 +

EXAMPLE

54

1
=0
2

1
3

1
4

Determine whether the following equations are quadratic or not.


1 2
x 2 x +5 = 0
2

a. x2 + 1 = 0

b.

d. x2 2x1 + 3 = 0

e. (x 1)(x + 2) = 0

c. 2x2 3x = 5
f. (x 2)x2 = 0
Algebra 8

Solution

To the best of our


knowledge, the origin of
the term quadratic is
Latin. It is derived from
quadratus which is the
past participle of quadrare
which means to make
square. From this it is
clear that part of the
word is connected to the
Latin word for four: it
refers to squaring, and
a square is a regular
four-sided figure.

a, b, c, and e are quadratic equations. Equation d is not quadratic, since the power of x is 1,
which does not meet the requirements for a quadratic. Equation f is a third degree equation,
so it is not quadratic.
To solve a quadratic equation we must find the values of the unknown x which make the
left-hand and right-hand sides equal. Such values are called the solutions or roots of the
quadratic equation. A number of techniques are available to help us obtain a solution to any
quadratic equation.

A. SOLVING EQUATIONS OF THE FORM ax2 = 0


We have seen that the solutions (or roots) of a quadratic equation are the values of x that make
the two sides of the equation equal. Let us find the roots of the simple quadratic ax2 = 0.
ax2 = 0 (a 0)
x2 = 0
xx=0
x = 0 or x = 0
x1 = x2 = 0
We can see that this equation has two equal roots. When the roots of a quadratic equation
are the same, we say that the equation has a double root.

If A B = 0, then
A = 0 or B = 0.

EXAMPLE

Solution

3
Solve the equation x2 0.
2
3
x2 0
2
x2 0
x1 x2 0

Quadratic Equations

55

B. SOLVING EQUATIONS OF THE FORM ax2 + bx = 0


Let us look at the solution of this more complex quadratic.
ax2 + bx = 0
x(ax + b) = 0
x = 0 or ax + b = 0, so
b
x1 = 0 and x2 = .
a
This kind of quadratic equation has two roots and one of them is always zero.
EXAMPLE

Solution

Solve the equations.


a. x2 + x = 0

b. 4x2 + 5x = 0

c. 3x2 2x = 0

a. x2 + x = 0

b. 4x2 + 5x = 0

c. 3x2 2x = 0

x(x + 1) = 0

x(4x + 5) = 0

x(3x 2) = 0

x = 0 or x + 1 = 0

x = 0 or 4x + 5 = 0

x = 0 or 3x 2 = 0

x1 = 0 or x2 = 1

x1 = 0 or x2 =

5
4

x1 = 0 or x2 =

2
3

2 3
3

C. SOLVING EQUATIONS OF THE FORM ax2 + c = 0


Look at the calculation.
ax2 + c = 0,

c0

ax c
x2

c
a

c
a

Here the sign of

c
is important.
a

c
> 0, the equation has no real solution, because we cannot find the square root of a
a
negative number.

If

c
< 0, the equation has two real solutions. These roots are symmetric, i.e. they are the
a
same numeral with opposite signs.

If

Note
All positive real numbers have two square roots. One root is the positive square root and the
other root is the negative square root, i.e. if a2 = b and a is a positive real number, then a = b.
56

Algebra 8

EXAMPLE

Solution

Solve the equations.


a. 3x2 27 = 0

b. 2x2 + 6 = 0

c. 7 4x2 = 2

a. 3x2 27 = 0

b. 2x2 + 6 = 0

c. 7 4x2 = 2

3x2 = 27

2x2 = 6

x2 = 9

x2 = 3

x = 3

4x2 = 5
x2 =

no real solution
(x2 cannot be negative)

x=

Check Yourself 1

5
4

5
5
=
4
2

Solve the equations.


1. 3x2 = 0

2. 5x2 20x = 0

3. 7x2 + 35 = 0

4. 2x2 8 = 0

Answers
1. 0 2. 0, 4 3. no real solution 4. 2
FIND THE MISTAKE!
Let a and b be two arbitrary numbers such that a b. Then
(a b)2 = a2 2ab + b2 = b2 2ab + a2
(a b)2 = (b a)2
ab=ba
2a = 2b
a = b.
Can you find the mistake in this working?

D. SOLVING EQUATIONS OF THE FORM ax2 + bx + c = 0


There are three basic methods for solving a quadratic equation of the form ax2 + bx + c = 0:
1. factoring,
2. completing the square, and
3. using the quadratic formula.

1. Factoring
If we can write ax2 + bx + c = 0 as the product of two linear factors, then we can easily solve
the equation.
To solve a quadratic equation by factoring, follow the steps.
1. Write the equation in standard form, ax2 + bx + c = 0, a 0.
2. Factor the left side of the equation.
Quadratic Equations

57

3. Apply the zero product property, that is, set each factor equal to zero.
4. Solve each equation to obtain the roots.
EXAMPLE

Solve by factoring.
a. x2 + 3x + 2 = 0

Solution

b. 6x2 19x 7 = 0

a. x2 + 3x + 2 = 0

c. 2x2 = x + 3

b. 6x2 19x 7 = 0
(2x 7)(3x + 1) = 0
2x 7 = 0 or 3x + 1 = 0
7
1
x1 = , x2 =
2
3

(x + 2)(x + 1) = 0
x + 2 = 0 or x + 1 = 0
x1 = 2, x2 = 1
c. 2x2 = x + 3
2x2 x 3 = 0
(x + 1)(2x 3) = 0
x + 1 = 0 or 2x 3 = 0
3
x1 = 1, x2 =
2

Note
When you are solving an equation, do not divide both sides by an expression containing
the variable for which you are solving. You may be dividing by zero. For example, to solve
x2 2x = 0, do not divide both sides by x, because x may be zero and you will also lose one
of the solutions.

Check Yourself 2
Solve the following equations.
1. 3x2 = 5x + 2
Answers
2
1
1. , 2 2. 2 ,
5
3

2. (5x 1)(x + 2) = x + 2

3. 4x(x + 1) = 3

3 1
3. ,
2 2

2. Completing the Square


The idea behind this method is to adjust the left side of the equation ax2 + bx + c = 0 so that it
becomes a perfect square, that is, the square of a first-degree polynomial. Expressions in the
form x2 + 2xy + y2 and x2 2xy + y2 are perfect square polynomials.
Numbers whose square
roots are integers or
quotients of integers
are perfect squares.

58

For example, x2 + 6x + 9 and x2 4x + 4 are perfect squares, because


x2 + 6x + 9 = (x + 3)2 and x2 4x + 4 = (x 2)2.
To write a quadratic equation as a perfect square, follow the steps,
Algebra 8

1. Make sure a = 1 in the quadratic. If it isnt 1,


divide each term by a.

x+2
x

2. Rewrite the equation so that the constant term is


alone on one side of the equation.
3. Take half of the coefficient of the x term and square
it.

x+2

4. Add this number to both sides of the equation.

5. Factor the left-hand side into a perfect square.

6. Solve for x by using the square root property.


x2 + 4x

Let us look at some examples of completing the square.

(x + 2)2 = x2 + 4x + 4

Start
_____________

Add
_____________

Result
___________________________

x2 + 4x

x2 + 4x + 4 = (x + 2)2

x2 + 12x

36

x2 + 12x + 36 = (x + 6)2

x2 6x

x2 6x + 9 = (x 3)2

x2 + x

1
4

x2 + x +

1
1
= (x + )2
2
4

Note
The expression x2 + 2bx is equivalent to (x + b)2 b2.
EXAMPLE

Solution

Solve by completing the square.


a. x2 + 6x 7 = 0

b. 2x2 4x + 1 = 0

a. x2 + 6x 7 = 0

b. 2 x2 4x+ 1= 0
1
x2 2 x+ = 0
2
1
x2 2 x =
2
1
x2 2 x+ 1= + 1
2
1
(x 1)2 =
2
1
x 1=
2

x2 + 6x = 7
x2 + 6x + 9 = 7 + 9
(x + 3)2 = 16
x + 3 = 16
x1 = 7,

x2 = 1

x1 =1
Quadratic Equations

2
,
2

x2 =1+

2
2
59

Check Yourself 3
Solve the equations.
1. x2 + 8x 3 = 0

2. 2x2 5x 3 = 0

3. 2x2 2 = 4x

Answers
1. 4 19

2. 3,

1
2

3. 1 2

3. The Quadratic Formula


The final method will work on any quadratic equation. Therefore, we can use it when the other
easier methods fail or are not easy to apply. Look at the derivation of the quadratic formula.
ax2 + bx+ c = 0

(a 0, a, b, c )

b
c
x2 + x+ = 0
a
a

(divide both sides by a)

b
c
x2 + x =
a
a

(try to get a perfect square)

b
b
c
b
x2 + x+( )2 = +( )2
2a
2a
a
a

(add (

(x+

b 2
) to both sides)
2a

b 2 4ac+ b2
) =
2a
4a2

x+

b
b 2 4ac
b 2 4ac

=
=
2a
2a
4a2

x=

b2 4ac b b 2 4ac
b

=
,
4a2
2a
2a

x1

b+ b 2 4ac
b b 2 4ac
, x2
2a
2a

QUADRATIC FORMULA
If ax2 + bx + c = 0, a 0, then x =

EXAMPLE

60

b b 2 4 ac
2a

Solve x2 + 2x 8 = 0 using the quadratic formula.


Algebra 8

Solution
Make sure that you write
a quadratic equation in
standard form before
you identify the values
a, b, and c.

EXAMPLE

Solution

First we identify the coefficients a, b, and c. For this equation a = 1, b = 2, and c = 8.


Let us substitute the values of a, b, and c into the quadratic formula.
x=
x1 =

Solution

2 6 4
= = 2,
2
2

x2 =

2 6 8
=
= 4
2
2

Solve 3x2 + 2x 4 = 0.
For this equation a = 3, b = 2, and c = 4.
x=

EXAMPLE

2
b b 2 4ac 2 2 4 1 (8)
2 36
=
=
2a
2 1
2

2
b b2 4ac 2 2 4 3 (4)
2 52
=
=
2a
23
6

x1 =

2 + 52 2 + 2 13 1+ 13
=
=
6
6
3

x2 =

2 52 2 2 13 1 13
=
=
6
6
3

Find the real solutions of the equation 9 +

3 2

= 0 , x 0.
x x2

3 2

= 0 , is not a quadratic equation. However,


x x2
we can make it quadratic by multiplying each side by x2, since x 0. The result is

In its present form, the equation 9 +

9x2 + 3x 2 = 0, x 0. Now, a = 9, b = 3, and c = 2.


b2 4ac = 32 4 9 (2) 81
x1 =

3 81 12
2
=
=
29
18
3

x2 =

3 + 81
6
1
=
=
18
18 3

Check Yourself 4
Solve the equations.
2. x2

1. 4x2 + 3x 1 = 0

5
= 3 x
2

3. 2x2 4x = 5

Answers
1. 1,
Quadratic Equations

1
4

2.

3 19
2

3.

2 14
2
61

4. Discriminant of a Quadratic Equation


discriminant of a quadratic equation

Definition

The quantity b2 4ac is called the discriminant of a quadratic equation.


The discriminant tells us whether the equation has real solutions, and also tells us how many
roots of an equation exist. The discriminant is denoted by (delta).
x=

b
,
2a

= b2 4ac

For a quadratic equation ax2 + bx + c = 0, the value of determines the number of real roots.
1. If > 0, there are two distinct real roots.
2. If = 0, there is one real root (a double root).
3. If < 0, there is no real root.
Use the discriminant to check the number of roots before you solve a quadratic equation.
EXAMPLE

10

Solution

Solve x2 + 6x + 7 = 0.
First, find .
= b2 4ac = 36 4 7 1 = 8
So is positive.
x1 =

6 + 8
= 3 + 2,
2

x2 =

6 8
= 3 2
2

We can see that > 0 and there are two real roots.
EXAMPLE

11

Solution

Solve x2 4x + 4 = 0.
First, check .
= b2 4ac = 16 4 4 1 = 0
So = 0 and x1 =

4 0
,
2

x2 =

4+ 0
.
2

Hence x1 = x2 = 2.
We can see that = 0 and there is only one real root (a double root).
EXAMPLE

12

Solution

Solve x2 2x + 5 = 0.
= b2 4ac = 4 4 1 5 = 16
is negative, so there is no real root.

62

Algebra 8

EXAMPLE

13

Solution

For which values of k does the equation 3x2 4x + k = 0 have no real solution?
If there is no real solution, the discriminant must be negative. Therefore,
= (4)2 4 3 k = 16 12 k < 0 .
16 12 k < 0, 16 < 12 k,
16
< k,
12

4
k> .
3

So the equation has no real solution for k >

EXAMPLE

14

Solution

4
.
3

For what values of m does the equation x2 + 3mx 5m 1 = 0 have a double root?
A quadratic equation has a double root if its discriminant is 0. Therefore,
= (3m)2 4 1 (5m 1) = 0.
9m2 + 20m + 4 = 0
(9m + 2)(m + 2) = 0
9m + 2 = 0 or m + 2 = 0.
So m1 =

EXAMPLE

15

Solution

2
and m2 = 2 .
9

The equation mx2 + (2m + 1)x + m 1 = 0 has two real roots. Find m.
If the quadratic equation has two real roots, then its discriminant is positive.
= b2 4ac = (2m+ 1)2 4m(m 1)
= 4m2 + 4m+ 1 4m 2 + 4m
= 8 m+ 1
= 8 m+ 1 > 0
8 m > 1
m>

1
8

Therefore, the equation has two real roots if m >


Quadratic Equations

1
.
8
63

EXAMPLE

16

Solution

Prove that (a2 + b2)x2 + 2(a + b)x + 2 = 0 has no real root if a and b are unequal.

= 4(a + b)2 4(a2 + b2) 2


= 4(a2 + 2ab + b2) 8(a2 + b2)
= 4a2 + 8ab + 4b2 8a2 8b2
= 4a2 + 8ab 4b2
= 4(a b)2
But (a b)2 is always non-negative, and it is also non-zero, since a b. So < 0.
Thus the equation (a2 + b2)x2 + 2(a + b)x + 2 = 0 has no real roots if a and b are unequal.

EXAMPLE

17

Solution

From each corner of a square piece of sheet metal, a man removes a square of side 3 cm. He
turns up the edges to form an open box. If the box holds 48 cm3, what are the dimensions of
the piece of sheet metal?
Let x be the length of a side of the square.
3

x6

3
3

x6

x6

3
3

x6

Since the volume of the box is 48 cm3, we have


3(x 6)2 = 48
(x 6)2 = 16
x 6 = 4
x1 = 10, x2 = 2.
We discard the solution x = 2 since length cannot be negative. So the sheet of metal is
10 cm by 10 cm.
64

Algebra 8

EXAMPLE

18

Solution

A ladder 5 m long is leaning against a house.


The distance from the bottom of the ladder to
the house is 1 m less than the distance from
the top of the ladder to the ground. How far is
the bottom of the ladder from the house?

Let us use x to represent the distance from the top of the ladder to the ground. The ladder
then forms the right triangle shown in the diagram. By using the Pythagorean Theorem, we
get the equation
x2 + (x 1)2 = 52
x2 + x2 2x + 1 = 25
2x2 2x 24 = 0

x2 x 12 = 0
(x + 3)(x 4) = 0
x = 3 or x = 4.

x1

Since the length cannot be negative, x = 4. So the distance from the bottom of the ladder to
the house is 3 m.
EXAMPLE

19

Solution

A motorboat heads upstream a distance of 48 km on a river whose current is running at 3 km per


hour (km/h). Then the motorboat returns. The trip upstream and back takes 12 hours. Assuming
that the motorboat maintained a constant speed relative to the water, what was its speed?
We use x to represent the constant speed of the motorboat relative to the water. Then
the true speed going upstream is x 3 km/h, and the true speed going downstream is
x + 3 km/h. Since Distance = Velocity Time, we can write Time = Distance / Velocity.
Therefore, the boat takes
travel upstream and
downstream.

48
hours to
x3

48 km

48
hours to travel
x+ 3
x 3 km/h

Since the total time is 12 hours,


48
48
+
= 12; x 3.
x 3 x+3

x + 3 km/h

Multiply both sides by (x 3)(x + 3):


Quadratic Equations

65

48(x 3) + 48(x + 3) = 12(x 3)(x + 3)


x2 8x 9 = 0
(x 9)(x + 1) = 0
x = 9 or x = 1.
Since the speed cannot be negative, the speed of the boat is 9 km/h.

Check Yourself 5
1. Evaluate the discriminant of the equation 2x2 x 3 = 0, and describe the roots.
2. The sum of two numbers is 10 and sum of their squares is 68. Find the numbers.
3. For which value(s) of m does the equation mx2 + mx 1 = 0 have a double root?
Answers
1. = 25, two real roots

2. 2, 8

3. 4

M AT H F U N
A mathematician, a physicist, and an engineer were traveling through Scotland when they
saw a black sheep through the window of the train.
Aha, said the engineer, I see that Scottish sheep are black.
Hmm, said the physicist, You mean that some Scottish sheep are black. We
havent seen all the sheep yet.
No, said the mathematician, All we know is that there is at least one sheep
in Scotland, and that at least one side of that one sheep
is black! We havent seen the other
side of the sheep yet.

66

Algebra 8

T HE G OLDEN R ATIO
The Golden Ratio appears again and again in art, architecture, music and nature. Its origins go back to the days
of the ancient Greeks, who thought that a rectangle with sides in the Golden Ratio, called a golden rectangle,
exhibited the most aesthetically pleasing proportion. The use of the Golden Ratio has been of interest to artists
and architects since before the building of the Parthenon in Greece in the fifth century B.C. The rectangle drawn
around the Parthenon with its upper triangular structure intact, as shown in Figure 1, is a golden rectangle.
1

x1

x
Figure 1

Figure 2

Figure 2 shows the dimensions of a golden rectangle. The ratio of length to width
in the rectangle is x to 1. A square P with sides of 1 unit has been marked, leaving
a smaller rectangle Q. For the smaller rectangle Q, the ratio of length to width is 1
to x 1. In order for the larger rectangle to be a golden rectangle, the two ratios
need to be equal, creating a proportion:
x
1
=
1 x1
When we apply cross multiplication to this proportion, we obtain a quadratic
equation:
x
1

x(x 1) = 1 ; x2 x = 1 ; x2 x 1 = 0.
1 x 1
Since we cannot factor this equation, we apply the quadratic formula.
The solutions are x1

1 5
1 5
and x2
.
2
2

Because x represents the length of a rectangle, the negative solution is discarded


1 5
1.618034 . This is the Golden Ratio.
2
The Golden Ratio occurs in nature as well as in art. For example, in sunflowers,

and the positive solution is

the ratio of the number of clockwise spirals to the number of counterclockwise


spirals approximates the Golden Ratio.

EXERCISES

3 .1

A. Solving Equations of the Form ax2 = 0


1. Solve the equations.
a. 2x2 = 0

b. (3 2)x2 = 0

1
c. x2 = 0
5

d. 0.07x = 0

B. Solving Equations of the Form


ax2 + bx = 0
2. Solve the equations.
2

D.Solving Equations of the Form


ax2 + bx + c = 0
9. Solve by factoring.
a. x2 x = 0

b. 3x2 + x = 0

c. x2 49 = 0

d. x2 25 = 0

e. x2 x 2 = 0

f. x2 + 3x + 2 = 0

g. x2 2x + 1 = 0

h. 6x2 + x 15 = 0

i. 10x2 19x + 6 = 0

j. 12x2 5x 2 = 0

10. Solve by factoring.


2

a. 2x + 5x = 0

b. 7x + 3x = 0

c. 3x2 8x = 0

d.

2
x 6 x2 = 0
3

a. x2 + (a + 1)x + a = 0
b. (x + 1)2 2(x + 1)(x 3) + (x 3)2 = 0
c. 2ax2 + (5b 2a)x 5b = 0

11. Solve by completing the square.

3. Solve (x + 2)2 = 2(x + 2).


4. Solve (2x 1)(x + 3) = 3.
5. Solve (x + 1) 3(x + 1) = 0.

a. x2 4x 1 = 0

b. x2 + 4x = 3

c. x2 6x 13 = 0

d. 3x2 2x + 4 = 0

e. 4x2 + 8x + 15 = 0

f. 2x2 + 7x + 11 = 0

g.

C. Solving Equations of the Form


ax2 + c = 0
6. Solve the equations.
a. x2 16 = 0

b. 7x2 + 3 = 0

c. 9x2 25 = 0

d. 0.6x2 15 = 0

e. 5x2 + 4 = 0

f.

2 x2

=0
25 8

7. Solve the equations.


a. (x 2)2 9 = 0
2

c. (x + 1) = 5

d. (1 x) 1 = 8

8. Solve (4x + 1)(x 1) = (x 1)(x + 1) 3(x 3).


68

h. 3x2 + x =

1
2

12. Solve by using the quadratic formula.


a. x2 4x + 2 = 0

b. 2x2 + x 1 = 0

c. 4x2 + 12x + 9 = 0

d. 3x2 5x + 1 = 0

e. 5x2 + 4x + 7 = 0

f. 3x2 7 = 2 2x

g. x2 + x + 1 = 0
h. 25x2 + 40x + 16 = 0
i. (2x 3)2 = 11x 19
j.

b. (x + 3)2 + 7 = 0

1 2
1
x 3x + = 0
2
2

3 2 1
1
x x =0
4
4
2

2
k. 4x 1 = x(10 x 9)
3

l.

x2 1
=11( x +1)
2
Algebra 8

13. Solve by using the quadratic formula.


a. 4
b.

x2 (4k + 2)x + 7k + 2 = 0 form a perfect square?

1 2
=0

x x2

1
1
2
+
=
x 1 x 1 3
2

2
2
c. (x 3) (x 2) = 1 x
16
4
2

d.

17. For which values of k does the equation

18. The longest side of a right triangle is 6 cm less


than twice the length of the medium side. The
shortest side is 6 cm. Find the length of the two
other sides.

1
1
5
+
=
x 1 x 4 4

14. Use the discriminant to determine whether each


quadratic equation has two real solutions, a
double root, or no real solution, without solving
the equation.
a. 2x2 + 3x + 1 = 0

19. A wire that is 32 cm long is cut into two pieces,

and each piece is bent to form a square. The total


area enclosed by the two squares is 34 cm2. Find
the length of each piece of wire.

b. x2 + 5x 6 = 0

c. 4x2 + 12x + 9 = 0 d. 25x2 20x + 4 = 0


e. x2 + 4x + 7 = 0

f. 2x2 x + 2 = 0

Mixed Problems
20. a, b, c, and d are four consecutive even natural
15. Consider the equation ax2 + 3x + 10 = 0. For

which values of a does the equation have

numbers. The sum of a and c is one fifth of the


product of b and d. Find a, b, c, and d.

a. two distinct real roots?


b. one double root?
c. no solution?

21. Two squares have sides (x + 6) cm and (2x + 1)

cm respectively. The sum of their areas is


697 cm2. Find the areas of the squares.

16. For which values of m does the equation


2x2 + 2x + m + 4 = 0 have
a. two real solutions?
b. one solution?
c. no real solutions?
Quadratic Equations

22. A year ago, a father was eight times as old as his

son. Now his age is the square of his sons age.


How old are they now?
69

We have seen that the roots of an equation depend on its coefficients. Therefore, there exist
certain relations between the coefficients and the roots of an equation.
In this section we will consider the relations between the roots and the coefficients a, b and
c of a quadratic equation.
We know that the roots of the quadratic equation ax2 + bx + c = 0, a 0 are
x1 =
Franois Vite
(or Vieta)
1540-1603, French
mathematician. Vieta
was a founder of
modern algebra, who
introduced the use of
letters as algebraic
symbols and correlated
algebra with geometry
and trigonometry.
Vieta presented
methods for solving
equations of second,
third and fourth
degree. He knew the
connection between
the positive roots of
equations and the
coefficients of the
different powers of the
unknown quantity.
The word coefficient
is actually due to Vieta.
When Vieta applied
numerical methods to
solve equations, he
used methods which
were similar to those
used by earlier Arabic
mathematicians.

b
b +
and x2 =
.
2a
2a

Let us use these formulas above to find the sum of the roots of a quadratic equation.
x1 + x2 =

b b+
+
2a
2a

b + ( b+ )
2a

2b
b
=
2a
a

Therefore,

x1 + x2 = -

b
.
a

We can use the same expressions for x1 and x2 to find the product of the roots of a quadratic
equation.
b b+ ( b ) ( b+ )
x1 x2 =

=
4a2
2a 2a
=

b2 (b2 4ac ) 4ac c


= 2=
4a2
4a
a

Therefore,

x1 x2 =

c
a

These relations were discovered by Franois Vieta, a French mathematician, and so they are
together called Vietas theorem.
70

Algebra 8

Vietas theorem

Theorem

Let x1 and x2 be the roots of the quadratic equation of the form ax2 + bx + c = 0, a 0. Then
b
x1 + x2 = ,
a

c
x1 x2 = .
a

By using Vietas theorem we can now find the sum and product of the roots of a quadratic
equation without calculating the roots.

Note
By using Vietas theorem we can also see the following.
1.

x + x2 b / a b
1
1
(sum of the reciprocals of the roots)
+ = 1
=
=
x1 x2
x1x2
c/a
c
2

c b2 2ac
b
2. x12 + x22 = (x1 + x2 )2 2x1x2 = 2 =
(sum of the squares of the roots)
a
a2
a

3. x13 + x23 = (x1 + x2 )3 3x1x2 (x1 + x2 ) =


EXAMPLE

20

Solution

Derive a formula for the difference of the roots of a quadratic equation in standard form.
Let us consider the two differences. First,
x1 x2 =

b b+ b + b 2

=
=
=
.
2a
2a
2a
2a
a

Similarly, x2 x1 =

.
a

So we can say that x1 x2 =


EXAMPLE

21

Solution

3abc b 3
(sum of the cubes of the roots)
a3

.
a

Find the sum and product of the roots of the given equations, without solving the equations.
a. 2x2 + 6x + 5 = 0

b. x2 3x 5 = 0

a. a = 2, b = 6 and c = 5

b. a = 1, b = 3 and c = 5

b 6
x1 + x2 = =
= 3
a
2
x1 x2 =

c 5
=
a 2

b 3
x1 + x2 = = = 3
a 1
x1 x2 =

c 5
=
= 5
a
1

Note
The quadratic equation 2x2 + 6x + 5 = 0 has no real root. However, by Vietas theorem, the
5
sum of the roots is 3 and the product of the roots is , which are real numbers. Can you
2
say why?
Quadratic Equations

71

Check Yourself 6
1. Find the sum and the product of the roots of the following equations, using Vietas theorem.
7
a. 3x2 + 5x 1 = 0
b. x2 4x + = 0
c. x2 + 7x 1 = 0
2
2. For the previous equations, find
a. 1 + 1 .
b. x12 + x22.
c. x13 + x23.
x1 x2
Answers
5 1
7
1. a. ,
b. 4,
c. 7, 1
2. a. 5 b. 9 c. 322
3 3
2
EXAMPLE

22

Solution

x1 and x2 are the non-negative roots of the equation 3x2 + 2mx + 1 = 0. Given x1 = 3x2, find
x1, x2, and m.
x1 = 3x2
x1 x2 =

c 1
= ;
a 3

1
3x2 x2 = ;
3

x1 = 3x2 then x1 = 3
x1 + x2 =

2m
;
3

1
x2 2 = ;
9

x2 =

1
since x2 > 0;
3

x2 =

1
3

1
=1
3

1
2m
1+ =
;
3
3

4
2m
=
3
3

m = 2
EXAMPLE

23

Solution

x1 and x2 are the roots of the equation x2 3x + 1 = 0. Find the value of

Since x1 + x2 =

x1
x2

x2
x1

(3)
1
= 3 > 0 and x1 x2 = =1> 0, both x1 and x2 are positive.
1
1

x1
x
+ 2 =k
x2
x1
k2 =

x12 2 x1x2 x2 2
+
+
x2
x1
x1x2

k2 =

x13 + x2 3
+ 2 x1x2
x1x2

9 + 27
1
k =
+ 2 1 = 20
1
2

k1 = 2 5 , k2 = 2 5
k= 2 5
72

(why?)
Algebra 8

EXAMPLE

24

Solution

x2 + x 6m = 0 and x2 2mx + 3 = 0 have a common root. Find m.


Let x1 and x2 be the roots of x2 + x 6m = 0 and
x1 and x3 be the roots of x2 2mx + 3 = 0, then
x1 + x2 = 1
(1)
x1 + x3 = 2m

and (2)

x1x2 = 6m
.

x1x3 = 3

From (1) we get x2 x3 = 1 2m.


From (2) we get

x2
= 2m ; x2 = 2mx3.
x3

From (1) and (2),


2 mx3 x3 = 1 2m
x3 (2 m+ 1) = 1+ 2m
x3 =

1+ 2 m
= 1.
1+ 2 m

If we substitute 1 for x in the second equation, we have


4 2m = 0, so m = 2.

Note
We can apply Vieta's theorem to a cubic polynomial equation.
Let x1, x2 and x3 be the roots of the equation ax3 + bx2 + cx + d = 0. Then,
b
1. x1 + x2 + x3 = .
a

c
2. x1x2 + x1x3 + x2 x3 = .
a

d
3. x1x2 x3 = .
a

Check Yourself 7
1. One of the roots of the equation 2x2 mx + 8 = 0 is 3 more than the other root. Find m.
2. x1 and x2 are the roots of the equation x2 + x m + 1 = 0. x1 x2 = 5 is given. Find m,
x1, and x2.
Answers
1. 10 2. 7, 2, 3

Mathematics is one component of any plan for liberal education.


Mother of all the sciences, it is a builder of the imagination, a weaver
of patterns of sheer thought, an intuitive dreamer, a poet. The study
of mathematics cannot be replaced by any other activity.
Quadratic Equations

73

EXERCISES

3 .2

1. Find the sum and the product of the roots of each

6. One of the roots of the equation

equation, without solving it.

3
6x2 + 13x + n2 + 2n 2 = 0 is . Find n and
2
the other root of the equation.

a. 10x2 11x 12 = 0
b.

7 2 9
5
x + x =0
8
7
3

c. (x 2)(3x 4) = 13
d. x + 7 = (2x 1)(3x 2)
2

7. Let x1 and x2 be the roots of the equation

e. (4x + 3) = (3x + 1)

4x2 + 5x = 0. Find x12x2 + x1x22.

f. x2 = 2(3x 2x)

2. The sum of the roots of the equation

8. Consider the equation

(5k + 2)x2 + 7kx 8k = 0 is 3. Find the product

(m + 2n)x2 (m + 2n)x + m n = 0.

of the roots.

If the arithmetic and the geometric means of the


roots of the equation are equal, find the relation
between m and n.

3. The product of the roots of the equation


(4m2 1)x2 + (2m + 1)x + 2m 1 = 0 is 5.
Find m.

9. Let x1 and x2 be the roots of the equation


x2 8x + 5k = 0. x1 = 2x2 1 is given. Find k.

4. Find the sum of the squares of the roots of the


following quadratic equations.
a. 2x2 + 5x + 1 = 0
b. x2 + 7x + 2 = 0
c. 3(x + 2)(x 1) = 4(x 2) 1

10. Let x1 and x2 be the roots of the equation

x2 + (m + 1)x + m + 2 = 0.
2x1 + 3x2 = 13 is given. Find m.

d. 4x(3 4x) = x 1

5. One of the roots of the equation


4
20x2 + (2m2 + 2m + 1)x + 20 = 0 is . Find
5
m and the other root of the equation.
74

11. Let x1 and x2 be the roots of the equation


(k + 1)x2 kx + k 4 = 0.

6
6
+ = 30 is given.
x1 x2

Find k.
Algebra 8

12. Consider the equation x2 (3k + 1)x + 8 = 0.


2
1

2
2

x + x = 20 is given. Find k.

13. Consider the equation x2 + (m + 1)x m = 0.


2
1

2
2

x + x = 13 is given. Find m.

18. Let p and q be the roots of the equation

2x2 5x + p2 + q2 = 0. Find the discriminant of


the equation.

19. Find x12 +

1
2

x1

, if x1 is the root of the equation

1
1

x + 6 x + +9 = 0.
x
x

14. Let x1 and x2 be the roots of the equation


kx2 + (k 1)x 2 + k = 0.
2
2
3
+
= is given. Find k.
x1 + 3 x2 + 3 2

20. Consider the equation x3 + cx + 1 = 0.

x1 =

1
1
is given. Find the roots of the
+
x2 x3

equation and c.

15. Consider the equation x2 + (k 2)x + k 6 = 0.


If the equation has two negative roots, find all the
possible values of k.

16. Find two positive consecutive numbers such that


the sum of their squares is 85.

21. The roots of the equation x2 5x + p = 0 are also

the roots of the equation x3 + qx + 30 = 0. Find


p + q.

22. The sum of the two roots of the equation


2x3 x2 7x 3 = 0 is 1. Find the roots of the
equation.

23. From each corner of a square piece of sheet


17. Find the product of the roots of the equation

1 1
1
1
, a + b 0.
+ =
a b a+b+ x x

Quadratic Equations

metal, a man removes a square of side 2 cm. He


turns up the edges to form an open box which
holds 24 cm3. What are the dimensions of the
piece of sheet metal?
75

Let x1 and x2 be the roots of the equation ax2 + bx + c = 0.


b
c
We can write the equation ax2 + bx + c = 0 as x2 + x + = 0.
a
a
b

x1 + x2 = a , and

We know that
c

x1 x2 = a .

So we can write the equation again as x2 (x1 + x2)x + (x1 x1) = 0.


In other words, if we know the sum and product of the roots of a quadratic equation then we
can write the equation as x2 Sx + P = 0 where S = x1 + x2 and P = x1 x2. This means
that we can derive (find) a quadratic equation if we know its roots.

Note
If the roots of a quadratic equation are x1 and x2, then (x x1) (x x2) = 0.
EXAMPLE

25

Solution

Find a quadratic equation whose roots are 1 and 5.


x1 = 1 and x2 = 5
S = x1 + x2 = 1 + 5 = 4
P = x1 x2 = (1) 5 = 5
Hence, the equation is
x2 Sx + P = 0
x2 4x 5 = 0.

EXAMPLE

26

Solution

Find a quadratic equation whose only root is


x1 = x2 =

1
.
3

1
3

1 1 2
S = x1 + x2 = + =
3 3 3
1
P = x1 x2 = .
9
Hence, the equation is x2
76

2
1
x+ =0, or 9 x2 6 x+1 =0.
3
9
Algebra 8

EXAMPLE

27

Solution

Find the equation whose roots are 2 + 3 and 2 3.


x1 = 2 + 3, x2 = 2 3
S = x1 + x2 = 4
P = x1 x2 = (2 + 3) (2 3) = 1
Hence, the equation is x2 4x + 1 = 0.

EXAMPLE

28

Solution

Find the equation whose roots are 1 more than the roots of x2 3x 4 = 0.
Let the roots of x2 3x 4 = 0 be x1 and x2. Let the roots of the equation we are looking for
be x3 and x4.
x1 + x2 = 3,

x1 x2 = 4

x3 = x1 + 1,

x4 = x2 + 1

S = x3 + x4 = (x1 + 1) + (x2 + 1) = x1 + x2 + 2 = 3 + 2 = 5
P = x3 x4 = (x1 + 1) (x2 + 1) = x1 x2 + x1 + x2 + 1 = (4) + (3) + 1 = 0
Hence the equation is x2 5x = 0.
EXAMPLE

29

Solution

The equation x2 + (k + 4)x + k 4 = 0 has symmetric roots. Find them.


x1 = t, x2 = t (symmetric roots)
x1 + x2 =
t +( t) =

b
a

( k+ 4)
1

0 = k 4
k = 4

x1 x2 =

c
a

t ( t) =

k 4
1

t2 = 8
t2 = 8

t = 22
Therefore, x1 = 22, x2 = 22.

Check Yourself 8
1. Find a quadratic equation whose roots are 2 and 3.
2. The roots of the equation x2 x 2 = 0 are x1 and x2. Find the equation whose roots are
x3 and x4, where x3 = 2x1 + 1 and x4 = 2x2 + 1.
Answers
1. x2 x 6 = 0 2. x2 4x 5 = 0
Mathematics has beauty but not everyone sees it.
Quadratic Equations

77

EXERCISES

3 .3

1. Find the equation with the given roots.

6. The roots of the equation x2 kx 3k + 1 = 0


are two more than the roots of the equation
x2 (k 4)x 4k = 0. Find k.

1 3
,
2 2

a. 1, 1

b.

c. 0, 4

d. 2 + 2, 2 2

e. 3 2, 3 + 2

f.

1
1
,
p2 q 2

7. Find three consecutive integers a, b, c, such that


a2 + b2 + c2 = 110.

8. Find the number that is 3 more than twice its


2. Find the quadratic equation whose sum of the

square root.

roots is 2 and product of the roots is 5.

9. A swimming pool can be filled by two pipes


x1 and x2. Find the equation whose roots are x3
and x4, such that x3 = 2x1 and x4 = 2x2.

together in six hours. If the larger pipe alone


takes five hours less than the smaller pipe alone
to fill the pool, find the time in which each pipe
alone would fill the pool.

4. Find the equation whose roots are 2 less than the

10. Alex can do a job in one hour less than Jane. If

3. The roots of the equation x2 4x 3 = 0 are

roots of the equation 2x2 6x + 58 = 0.

5. The roots of each given equation are x1 and x2.

6
5
hours. How long would it take each person working

Alex and Jane work together the job takes

alone?

Write a new equation with roots x3 and x4.


a. x2 2x 3 = 0

b. x2 + 5x + 4 = 0

x3 = x1 1

x3 = 2x1 + 1

x4 = x2 1

x4 = 2x2 + 1

c. 6x2 9x 6 = 0
x3 = x1 x2
x4 = x2 x1

11. A man completed a job for $156. It took him


seven hours longer than he expected and so he
earned $14 an hour less than he anticipated.
How long did the man expect the job to take?

d. x2 2 x+ 1 = 0
x3 =
x4 =

x12
x2
2
2

x
x1

12. The sum of the numerator and denominator of a


certain positive fraction is 11. If 1 is added to both
the numerator and the denominator, the fraction
3
is increased by
. Find the fraction.
56

e. x2 2mx + 3m 2 = 0 f. mx2 2mx 1 = 0

78

x3 = x1 2x2

x3 = 2x1 x2

x4 = x2 2x1

x4 = 2x2 x1

13. The area of an isosceles right triangle is 81 m2.


Find the perimeter of the triangle.
Algebra 8

14. The distance between two cities A and B is

19. Two painters working together can paint the front

140 km. A car driving from A to B left at the same


time as a car driving from B to A. The cars met
after one hour, then the first car reached city
B 35 minutes later than the second car reached
city A. Find the speed of each car.

of a house in 16 hours. One of the painters alone


can finish this job in 24 hours less time than the
other painter alone. How much time does each
worker need to do this job alone?

20. A motorcyclist traveled at a constant speed for 60


km. If he had gone 10 km/h faster, he would have
shortened his traveling time by one hour. Find the
speed of the motorcyclist.

15. An aeroplane traveled a distance of 400 km at an


average speed of x km/h. Write down an expression
for the time taken. On the return journey, the
speed increased by 40 km/h. Write down an
expression for the time for the return journey. If
the return journey took 30 minutes less than the
outward journey, write down an equation in x and
solve it.

21. The area of a triangle is 36 m2. The length of the


base is twice the height. Find the length of the
base and the height of the triangle.

22. A jeweler wishes to mix an alloy of 25% silver with


another alloy of 40% silver. How much of each
should he use to produce 60 kg of an alloy which
is 30% silver?

16. A car drove from one city to another and returned

by a different route. The outward journey was 48


km and the return journey was 8 km shorter. The
speed of the car on the return journey increased
by 4 km/h. The return journey took one hour less
time. Find the speed of the car on the outward
journey.

23. One of two pipes can fill a pool 24 hours faster

than the other one. The slower pipe filled the pool
for eight hours, then the other pipe was opened.
The pipes filled together for twenty hours and
2
of the pool. Find the time that each pipe
3
requires to fill the pool alone.

filled

17. A group of women plan to share equally in the


$14,000 cost of a boat. At the last minute three of
the women decide not to pay. This raises the
share for each of the remaining women by $1500.
How many women were in the original group?

18. A car drove from a city A to a city B. The distance


between the two cities is 350 km. After 200 km,
the speed of the car decreased by 20 km/h. The
total trip took 5 hours. Find the speed of the car
in the first part of the journey.
Quadratic Equations

24. A worker can clean a pool in four hours less time


than it takes another worker. If the men work
8
together the job takes
hours. How long would
3
it take each man working alone?

25. The length of a rectangle is 2 m more than its


width and the area is 48 m2. Find the length and
width of the rectangle.
79

A. WRITING EQUATIONS IN QUADRATIC FORM


standard form of an equation

Definition

An equation is in standard form if the only term on the right-hand side of the equation is zero.
For example, the equations 6x2 + 2x 3 = 0 and x4 5 = 0 are both in standard form. The
equation 6x2 + 2x = 3 and x4 = 5 are not in standard form.
Certain equations that are not quadratic can be expressed in quadratic form using
substitutions. These equations can be recognized because when they are written in standard
form, the exponent of the variable in one term is half the exponent of variable in the other term.
For example, we can write standard form equations such as
x4 + 17x2 + 72 = 0
2x8 + 4x4 = 0
x x 12 = 0
as quadratic equations, because the exponent of the first variable is twice the exponent of the
second variable.
Look at the steps to write an equation as a quadratic.
1. Let t be a variable term with the half exponent.
2. Substitute t in all the terms with the variable.
3. Solve for t.
4. Back substitute for the original variable.
EXAMPLE

30

Solution 1

Solve x4 13x2 + 36 = 0.
The equation x4 13x2 + 36 = 0 is not a quadratic equation but we can write it as
(x2)2 13x2 + 36 = 0. For this reason, it is a quadratic in x2. Let x2 = t.
First we solve for t, then solve the resulting equations for x.
(x2)2 13x2 + 36 = 0
x2 = t, so t2 13t + 36 = 0. By factoring,
(t 4)(t 9) = 0
t = 4 or t = 9.
Since t = x2
x2 = 4
x = 2

80

or

x2 = 9
x = 3

.
Algebra 8

Solution 2

Alternatively, we can directly factorize the equation.


x4 13x2 + 36 = 0
(x2 4)(x2 9) = 0
x2 = 4
x = 2

or

x2 = 9
x = 3

In both solutions, the roots of the given equation are 3, 2, 2, 3.


EXAMPLE

31

Solution

Solve (5x 1)2 + 4(5x 1) 5 = 0.


For the equation (5x 1)2 + 4(5x 1) 5 = 0, we let t = 5x 1 so that t2 = (5x 1)2. Then
the original equation becomes t2 + 4t 5 = 0. First solve for t:
(t + 5)(t 1) = 0, so
t = 5 or t = 1.
Now, solve for x.
5x 1 = 5
5x = 4
x1 =

or

4
5

5x 1 = 1
5x = 2
x2 =

2
5

Hence, the roots of the equation are


EXAMPLE

32

Solution

4
2
and .
5
5

Solve (a2 a)2 2(a2 a) = 0.


Let t = a2 a.
The equation becomes
t2 2t = 0
t(t 2) = 0
t = 0 or t = 2.
Now, solve for a.
1. a2 a = 0
a(a 1) = 0
a = 0 or a = 1
2. a2 a = 2
a2 a 2 = 0
(a 2)(a + 1) = 0
a = 2 or a = 1
Hence, the roots of the equation are 1, 0, 1, 2.

Quadratic Equations

81

EXAMPLE

33

Solution

1
2

Solve x + 5 = 2 x2 + 2 .
x
x

First x2 0, so x 0.
Let x

1
= t.
x

How can we write the right-hand side in terms of t?


Lets take the square of t:
2

1
1

t2 = x = x2 + 2 2, so
x
x

1
x2 + 2 = t 2 + 2. Now, let us multiply both sides of the equation by 2:
x
2
2 x2 + 2 = 2t2 + 4.
x
Then we have the equation
t4 +5 = 2t2 + 4
t4 2t 2 +1= 0
(t2 1)2 0
t2 1= 0
t =1 or t = 1.
1
Since t = x ,
x
1
1
x =1 or x = 1.
x
x
Let us multiply the equations by x:
x2 x 1= 0 or x2 + x 1= 0.
By using the quadratic formula,
x1 =

1 5
1+ 5
1+ 5
1 5
, x2 =
, x3 =
, x4 =
.
2
2
2
2

Check Yourself 9
Solve the equations.
1. (x + 15)2 3(x + 15) 18 = 0.
2. (x + 5)4 6(x + 5)2 7 = 0.
3. x 9x + 14 = 0.
Answers
1. 18, 9 2. 5 7 3. 4, 49
82

Algebra 8

B. EQUATIONS INVOLVING PRODUCTS AND QUOTIENTS


If the product of two or more numbers is zero, then at least one of the factors must be zero.
If the quotient of a division is zero, then the dividend must be zero.

Note
1. P Q = 0 if and only if P = 0 or Q = 0, where P = P(x) and Q = Q(x).
2.

EXAMPLE

34

Solution

P
= 0 if and only if P = 0 and Q 0.
Q

Solve (x2 1)(x2 2x 8) = 0.


Try to factorize each part if possible:
(x 1)(x + 1)(x 4)(x + 2) = 0.
If the product is zero then at least one of the factors is zero.
x1=0;

x=1

x+1=0;

x = 1

x4=0;

x=4

x+2=0;

x = 2

Thus, the roots of the equation are 2, 1, 1, 4.

EXAMPLE

35

Solution

Solve (x2 4x + 10)(x2 5x + 2) = 0.


We cannot factorize the parts.
Let us try to solve x2 4x + 10 = 0 and x2 5x + 2 = 0 as two different quadratic
equations.
x2 4x + 10 = 0; = 16 4 1 10 = 24 < 0.
Therefore, this equation has no real root.
x2 5x + 2 = 0 ;

= 25 4 1 2 = 17

This time, by the quadratic formula, the roots are


x1 =
Quadratic Equations

5 17
5+ 17
, x2 =
.
2
2
83

EXAMPLE

36

Solution

Solve x3 x2 4x + 4 = 0.
First try to factorize the expression.
x3 x2 4x + 4 = x2(x 1) 4(x 1) = (x 1)(x2 4) = (x 1)(x 2)(x + 2).
Now, the question becomes:
solve (x 1)(x 2)(x + 2) = 0.
So the solution is
x1=0;

x1 = 1

x2=0;

x2 = 2

x+2=0;

EXAMPLE

37

Solution

Solve

x3 = 2.

x2 5x 6
= 0.
x+ 2

The denominator of a fraction cannot be equal to zero, so x + 2 0, x 2.


We need to solve x2 5x 6 = 0.
x2 5x 6 = (x 6)(x + 1)
(x 6)(x + 1) = 0
x1 = 1 or x2 = 6
Since these roots are not equal to 2, x1 = 1 and x2 = 6 are both solutions to the equation.

EXAMPLE

38

Solution

Solve

x2 + 7 x 8
= 0.
x2 3x+ 2

First solve x2 3x + 2 0: x 2, x 1.
x2 + 7x 8 = (x 1) (x + 8) = 0
So x1 = 1 and x2 = 8 are the roots of the numerator.
We know that x 1, so the only solution is 8.

Note
It is very important to check the roots of the numerator to see whether they make the
denominator zero or not. We can do this either by substituting the roots of the numerator in
the denominator, or by finding the roots of the denominator directly and checking whether
they are common or not.
84

Algebra 8

EXAMPLE

39

Solution

Solve

x 7 x+ 3

= 1.
x 4 x+ 4

x 4 0, and x + 4 0, so x 4, and x 4 (since the denominators cannot be zero).


x 7 x+ 3

1 = 0.
x 4 x+ 4

Now we have

Let us make the denominators common:


(x 7)( x+ 4) ( x+ 3)( x 4) ( x2 16)
= 0.
(x+ 4)(x 1)
x2 2 x
=0
(x 4)(x+ 4)

x2 2x = 0 ; x(x + 2) = 0 ; x1 = 0 and x2 = 2.

EXAMPLE

40

Solution

Solve

4
x+ 1 x2 5

=
.
x+ 1 x 1 x2 1

4
x+ 1 x2 5
is not a quadratic equation. However,

=
x+ 1 x 1 x2 1
we can make it quadratic by multiplying each side by x2 1 since (x2 1 0 ; x 1). The result is

In its present form, the equation

4(x 1) (x + 1)2 = x2 5
4x 4 x2 2x 1 = x2 5
2x2 + 2x = 0
2x(x 1) = 0
x1 = 0 or x2 = 1, but x 1, so x = 0 is the only possible solution.

EXAMPLE

41

Solution

5x
x
Solve
+ 6 = 0.

x+ 2
x+ 2

x + 2 0 ; x 2
Let t =

x
, then the equation becomes
x+ 2

t2 5t + 6 = 0
(t 3)(t 2) = 0
t1 = 2 or t2 = 3.
Quadratic Equations

85

Now, solve for x:


x
=2
x+ 2

or

x
=3
x+ 2

x = 2x+ 4

x = 3 x+ 6

x2 = 4

x1 = 3.

So the roots are 4 and 3.

Check Yourself 10
1. Solve

9
+ 2 x = 4.
4x+ 3

2. Solve

x+ 1 x 3
12

=
.
x 2 x+ 2 x2 4

Answers
1 3
1. ;
4 2

2. no real solution

C. EQUATIONS INVOLVING RADICALS


When the variable in an equation occurs in a square root, cube root, and so on, that is, when
it occurs in a radical, the equation is called a radical equation. For example, the equations
x+ 1 = 2 and

x 1 5 = 0 are radical equations. Sometimes a suitable operation will

change a radical equation to an equation that is linear or quadratic.


To solve a radical equation, follow the procedure.
1. Isolate the radicals
Isolating a radical means putting the radical on one side of the equation and everything else
on the other side, using inverse operations. If there are two radicals in the equation, isolate
one of the radicals.
2. Get rid of the radical sign
Raise both sides of the equation to a power equal to the index of the isolated radical.
3. If there is still a radical sign left, repeat steps 1 and 2.
4. Solve the remaining equation
86

Algebra 8

5. Check for extraneous solutions


When you solve a radical equation, extra solutions may come up when you raise both sides
to an even power. These extra solutions are called extraneous solutions. In radical equations
you check for extraneous solutions by putting the values you found into the original
equation. If the left side of the equation does not equal the right side then you have an
extraneous solution.

Note
1. If a value is an extraneous solution, it is not a solution to the original problem.
2. It is very important to check your results in the original equation. In many equations, one
of the results may not satisfy the original equation. However, sometimes it is possible that
all results that you have found will be acceptable.
EXAMPLE

42

Solution

Solve the radical equation

2 x+ 5 = 7.

Here the radicand is already alone; we do not need to isolate it. So take the square of both sides:
2x + 5 = 49 ; 2x = 44 ; x = 22.
Now let us check to see if x = 22 is an extraneous solution:
2 x+ 5 = 7
2 22 + 5 = 7
49 = 7
7 = 7.

Since the last statement is true, x = 22 is not an extraneous solution. Therefore, there is one
solution to this radical equation, x = 22.
EXAMPLE

43

Solution

Solve

2 x 5 + x = 4.

First we isolate the radical:


2 x 5 = 4 x.

Now take the squares of both sides to eliminate the square root:
2x 5 = 16 8x + x2.
The new equation is x2 10x + 21 = 0
(x 3) (x 7) = 0
x = 3 or x = 7.
Let us check the results in the original equation:
Quadratic Equations

87

x=7;

2 .7 5 + 7 = 4

x=3;

2 3 5 +3 = 4

9 +7 = 4

1+ 3 = 4

3+7 = 4

4=4

10 = 4

This is true, so x = 3 is a solution.

This is false, so 7 is an extraneous solution.


Hence, the only solution to the equation is x = 3.
EXAMPLE

44

Solution

Solve

10 x+ 56 2 x +8 = 4 .

In this question there are two radical expressions. We can isolate only one expression, so it
is better to isolate the more complex one. So we have
10 x+ 56 = 2 x+ 8 + 4.

Take the squares of both sides:


10 x+ 56 = (2 x+ 8) + 2 4 2 x+ 8 +16 .

This is a new equation involving radical expressions. Follow the same steps again to isolate
the second radical.
8 x+ 32 = 8 2 x+ 8
x+ 4 = 2 x+ 8
x2 + 8 x+ 16 = 2 x+ 8
x2 + 6 x+ 8 = 0
(x+ 4) (x+ 2) = 0

x = 4 ,

x = 2

Now, check these results in the original equation.


x = 4 ;

2 (4) + 8 10 (4) +56 = 4


0 16 = 4
4 = 4

This is true!
x = 2 ;

2 (2) + 8 10 (2) + 56 = 4
4 36 = 4
2 6 = 4
4 = 4

This is true!
Hence, both 4 and 2 are solutions to the equation.
88

Algebra 8

EXAMPLE

45

Solution

Solve the equation 5 + 3 x+ 3 = 3.

5 + 3 x+ 3 = 3
3

x+ 3 = 2 (by taking the cube of both sides)

x+ 3 = 8

x = 11. We do not need to check for extraneous solutions because this is an odd power.
Therefore, 11 is the only solution to the equation.

EXAMPLE

46

Solve the equation

4x+ 1 + x+ 2 = 10 x+ 5 .

( 4 x+1 + x+ 2 )2 = ( 10 x+ 5) 2

Solution

4x+ 1+ 2 (4x+ 1)(x+ 2) + x+ 2 = 10 x+ 5


2 (4 x+1)( x+ 2) = 5 x+ 2
(2 (4 x+1)(x+ 2))2 (5 x+ 2)2
4(4 x+1)(x+ 2) 25 x2 + 20 x+ 4
16 x2 + 36 x+ 8 = 25 x2 + 20 x+ 4
9x2 16 x 4 = 0
(9 x+ 2)( x 2) = 0
x=

2
or x = 2
9

Now, check these results in the orijinal equation.


x=

2
;
9

x=2 ;

8
2
20
1
16 5
+1 + + 2 = + 5 =
+
= ;
9
9
9
9
9
3
8 +1 + 2 + 2 = 20 + 5 ;

5 =5

Both statements are true, so both x =


Quadratic Equations

5 5
=
3 3

2
and x =2 are solutions.
9
89

Check Yourself 11
Solve the equations.
2x 4 = x 2

1.

5x+ 3 = 4

4.

3x2 2 x+15 + 3x2 2 x+ 8 = 7

2.

3.

x+ 2 4 x+ 8 = 3

Answers
1
13
1.
2. 2, 4 3. 7 4. , 1
3
5

D. EQUATIONS INVOLVING AN ABSOLUTE VALUE


On the real number line, the absolute value of x is the distance from the origin to the point
x. For example, there are two points whose distance from the origin is three units, 3 and 3.
So the equation |x| = 3 has two solutions, 3 and 3. Let us first remember the mathematical
definition of absolute value.
absolute value of a function

Definition

For all real numbers x,


f x , for f x 0

f ( x) =
f x , for f x < 0.

We can use this information to begin solving equations involving one or more absolute values.
EXAMPLE

47

Solution

The absolute value of a


number is never negative.
|a| 0

EXAMPLE

48

Solution 1

Solve the equation |x 2| = 5.


Case 1
__________________

Case 2
_________________

x20; x2
x2=5
x=7

x2<0; x<2
(x 2) = 5
x = 3

So the solutions are 3 and 7.


Solve |2x 3| = x + 1.
Case 1
__________________
2x 3 0 ; x

Case 2
____________________

3
2

2x 3 < 0 ; x <

2 x 3 = x+ 1
x= 4
3
since 4 > , x = 4 is a solution.
2

So the solutions are 4 and


90

2
.
3

3
2

(2 x 3) = x+ 1
2
2 3
x=
since
< ,
3
3 2
2
is a solution.
x=
3

Algebra 8

Solution 2

Let us take the square of both sides:


4x2 12x + 9 = x2 + 2x + 1

|f(x)|2=f 2(x)= f(x). f(x)

3x2 14x + 8 = 0
(x 4) (3x 2) = 0.
So the solutions are 4 and

EXAMPLE

49

Solution

2
.
3

Solve x2 2|x + 2| + 1 = 0.
Case 1
_______________________

Case 2
________________________

x + 2 0 ; x 2
x2 2(x + 2) + 1 = 0
x2 2x 3 = 0
(x 3)(x + 1) = 0
x = 3 or x = 1

x + 2 < 0 ; x < 2
x2 2((x + 2)) + 1 = 0
x2 + 2x + 5 = 0
= 16
Since < 0, there are no real roots.

Both 3 and 1 are greater than 2, so the solutions to the equation are 1 and 3.

EXAMPLE

50

Solution

Solve the equation |x2 5x + 3| = 3.


Case 1
_______________________

Case 2
________________________

x2 5x + 3 0
x2 5x + 3 = 3
x2 5x = 0
x(x 5) = 0
x = 0 or x = 5
These values satisfy the inequality, so they
are solutions.

x2 5x + 3 < 0
(x2 5x + 3) = 3
x2 5x + 6 = 0
(x 3)(x 2) = 0
x = 3 or x = 2
These values also satisfy the inequality, so
they are also solutions.

Hence, the solutions to the equation are 0, 2, 3, 5.

EXAMPLE

51

Solution

2
2
Solve the equation x 3x+ 1 = x 4x+ 4 .

Remember that x2 4x + 4 = (x 2)2. Hence, the expression becomes


x2 3x+ 1 = (x 2)2 = x 2

For any real number x,


x2 = x .

Quadratic Equations

x2 3x+ 1 = x 2 .
91

Case 1
_______________________

Case 2
________________________

x20; x2

x2<0; x<2

(x2 3x + 1) = x 2

x 3x + 1 = x 2
x2 4x + 3 = 0

x2 2x 1 = 0
x1 = 1 2, x2 = 1 + 2

(x 3)(x 1) = 0

Since (1 + 2) is greater than 2,

x = 3 or x = 1

x = (1 2) is the only solution.

Since 1 2,
x = 3 is the only solution.
Hence, the solutions to the equation are (1 2) and 3.

EXAMPLE

52

Solution 1

Solve the equation |2x 3| = |x 7|.


Lets take the square of both sides. Then,
4x2 12x + 9 = x2 14x + 49
3x2 + 2x 40 = 0
(3x 10)(x + 4) = 0
x1 = 4 or x2 =

10
.
3

Therefore, both 4 and


Solution 2

2x 3 = x 7

or

x = 4
If |f(x)| = |g(x)|
then f(x) = g(x).

So both 4 and

10
are solutions to the equation.
3

2x 3 = (x 7)
10
x=
3

10
are solutions to the equation.
3

Check Yourself 12
Solve the equations.
1. |2 x| = 2x + 1

2. x|x + 1| 2 = 2x

Answers
1
1.
2. 2, 1, 2
3
Algebraic symbols are used when you do not know what you are talking
about.
92

Algebra 8

The following problem is posed and solved in the Chiu Chang Suan-Shu,
a Chinese mathematical treatise which is over two thousand years old.
A tree of height 20 m has a circumference of 3 m. An arrowroot vine
winds seven times around the tree before it reaches the top. What is the
length of the vine?
Can you solve this puzzle?

EXERCISES

3 .4

A. Writing Equations in Quadratic


Form
1. Solve the equations.
a. x4 13x2 + 36 = 0
4

B. Equations Involving Products and


Quotients

b. 3x 8x 3 = 0
c. (x2 9)2 4(x2 9) + 3 = 0
4

1
2
6
+ 2
= 2
x 3x+ 3 x 3x+ 4 x 3x+ 5
e. (x + 1)(x + 2)(x + 3)(x + 4) = 120

d.

d. (x + 5) 6(x + 5) 7 = 0
2

3. Solve the equations.


a. (16x3 x)(x2 6x + 5) = 0

3 4x
e.
2(3 4 x) + 25 = 0
5

c. x3 + 4x2 24 = 0

f.

d. x3 5x2 + 9x 45 = 0

x+ 2 4 x+ 2 = 6

b. (x2 + 8x)(x2 + 8x 6) = 280

g. x3 7 x 2 8 = 0

e.

1
7
1
h. x+ x+ = 2
x 2
x

i.

x2 + 2
8x
+ 2
=6
x
x +2

2. Solve the equations.


1

a. 3 x2 + 2
x

7 x+ = 0
x

f. (x2 5x + 6)2 (2x2 5x + 1)2 = 0


g. (6x2 5x 4)2 + (10x2 29x + 21)2 = 0
h.

3x2 9x
12
2
=3
2
x 3x

i.

3x+ 4 6(x 2)

=1
x2
3x+ 4

j.

4 x2
x 2 8
+ + 4 + = 0
2
x
9
3 x 3

b. (x 2x 5) 2(x 2x 3) 4 = 0
c.

3
= 3 x x2
1+ x+ x2

Quadratic Equations

2x+ 3 3x 2
=
4x 1 3x+ 2

93

C. Equations Involving Radicals


4. Solve the equations.
a.

D.Equations Involving an Absolute


Value
6. Solve the equations.

x 1 1= 0

a. |x| = x + 2

b. x + x = 5
c.

d.

f.

3 x +1 = x 2

g.

j.

d. |x2 4x| = 5
e. |x2 2x + 3| = 6

x 1 = x 1
x 3+ x 8 =5

i.

c. x |x 1| = 2

x 1 = 2

e.

h.

b. |2x 5| = x 2

f. x2 + |x 1| + 1 = 0
g. x2 + |2x 1| + 3 = 4x + 2

x2 x +6 2 = 0

x +16 x = 2

h.

x2 2 x +1 = 2 x 3

i.

x2 + x2 6 x +9 = 0

7. Solve the equations.

a. |x| + x2 = 0

x2 6 x x2 6x 3 = 5

b. ( x +1)( x 1) =

4x 1+ 2 x + 3 =1

1
2

c. (x + 1)2 2|x + 1| + 1 = 0

5. Solve the equations.

d. |x x2 1| = |2x 3 x2|
a.

x +1 9 x = 2 x 12

e.
b.

2 x +5 + 5 x +6 = 12 x + 25

x2 4 x + 3
x2 + x 5

=1

f. |x + |3 2x|| = 3 x
c.
d.

x x +1+ x +9 x + 4 = 0
2

2 x + 6 x +1 = x +1

Mixed Problems
8. Solve the equations.

e.

1 x2 3x = x +1

f.

x2 +1+ x2 8 =1

g.

3x2 +5 x +8 3 x2 +5 x +1 =1

h. 1+ x + 2 = 2 x + 2 +5
94

a.

x2 2x +1 =1 x

b. x2 + x2 6 x +9 = 2 x + 3
2
c. x + x +1 = 2 x 4x + 4

d. x2 x + 2 = x2 + 4x + 4 + x
Algebra 8

Sometimes we need to solve two or more equations simultaneously. A set of equations like
this is called a system of equations. There are no concrete rules that we can follow to solve
systems of equations, but let us look at some general strategies.

EXAMPLE

53

Solve the system of equations.


x+y=5
xy = 6

Solution

We can write y = 5 x. Now substitute this value of y in the second equation.


x(5 x) = 6
x2 5x + 6 = 0
(x 2)(x 3) = 0
x = 2 or x = 3
If x = 2 then y = 3.
If x = 3 then y = 2.
Therefore, the solutions of the system are (2, 3) and (3, 2).

EXAMPLE

54

Solve the system.


x2 + y2 = 65
x y = 28

Solution 1

Multiply the second equation by 2 and then add and subtract the resulting equations.
x2 + y2 = 65
2x y = 56
+
________________________

x2 + y2 = 65
2x y = 56
______________________

x2 + 2xy + y2 = 121

x2 2xy + y2 = 9

(x + y)2 = 121

(x y)2 = 9

x + y = 11

x y = 3

Therefore, we have four cases.


Quadratic Equations

95

Case 1
__________

Case 2
__________

Case 3
__________

Case 4
__________

x + y = 11

x + y = 11

x + y = 11

x + y = 11

xy=3
+
____________

x y = 3
+
_____________

xy=3
+
_____________

x y = 3
+
_____________

2x = 14

2x = 8

2x = 8

2x = 14

x=7

x=4

x = 4

x = 7

y=4

y=7

y = 7

y = 4

The solutions of the system are therefore (7, 4), (4, 7), (7, 4), (4, 7).

Solution 2

x2 + y2 = 65

x y = 28
y=

28
, x 0, substitute y in the first equation.
x
2

28
x2 +
= 65
x

x2 +

28 2
= 65
x2

x4 65 x2 + 784 = 0
(x2 49)( x2 16) = 0
x2 = 49 or x2 = 16
x = 7 or x = 4

By substituting these x values in y =

28
, we get the y values.
x

The solutions of the system are (7, 4), (7, 4), (4, 7), (4, 7).

EXAMPLE

55

Solve the system.


x2 = 13x + 4y
y2 = 4x + 13y

96

Algebra 8

Solution

Let us subtract the two equations side by side.


x2 y2 = 9x 9y
(x y)(x + y) = 9(x y)
x y = 0 or x + y = 9
Case 1
______________________________________________________
xy=0; x=y
Let us substitute x in the equation y2 = 4x + 13y.
y2 = 4y + 13y
y2 17y = 0
y1 = 0 or y2 = 17
x1 = 0 or x2 = 17
Therefore we have two solutions, (0, 0), (17, 17).
Case 2
______________________________________________________
x+y=9; x=9y
Let us substitute x in the equation y2 = 4x + 13y.
y2 = 4(9 y) + 13y
y2 9y 36 = 0
(y 12)(y + 3) = 0
y3 = 12 or y4 = 3
x3 = 3 or x4 = 12
Therefore we have two more solutions, (3, 12), (12, 3).
Hence, the solutions of the system are (0, 0), (17, 17), (3, 12), (12, 3).

EXAMPLE

56

Solution

Solve the system.


x+ y x y 5
x y + x+ y = 2

2
2
x + y = 20

x y 0 ; x y and x + y 0 ; x y.
Let us multiply both sides of the equation by 2(x y)(x + y).
2(x + y)2 + 2(x y)2 = 5(x y)(x + y)
2x2 + 4xy + 2y2 + 2x2 4xy + 2y2 = 5x2 5y2
x2 + 9y2 = 0.

Quadratic Equations

97

2
2

x + 9y = 0
Now, we have the system
.
x2 + y2 = 20

Let us add these two equations side by side.


10y2 = 20
y2 = 2 ; y = 2
x2 = 18 ; x = 32
Hence, the solutions of the system are (32, 2), (32, 2), (32, 2), (32, 2).

Check Yourself 13
Solve the systems of equations.
x2 y = 2
1.
x+ y = 4

x2 + 3xy 10y2 = 0
2.
x2 + 2xy y2 = 28

Answers
1. (2, 2), (3, 7)

2. (4, 2), (4, 2), (52, 2), (52, 2)

One persons constant is another persons variable.

EXERCISES

3 .5

1. Solve each system of equations.

2. Solve each system of equations.

9x2 y2 = 44
b.
3x y = 11

x2 + y2 + 6x+ 2y = 0
a.
x+ y+ 8 = 0

3x+ xy y2 = 0
b.
2 x2 3xy+ y2 = 0

x = 3 y
c.
y2 x = 39

1 1 1
+ =
d. x y 6

x+ y = 25

6 x + xy y = 0
c.
3x2 xy y2 = 0

1
1
x+ y + x y = 2
d.
4
3
x+ y + x y = 7

x2 3y2 = 13
e.
xy = 4

x2 + y2 = 18
f.
xy = 9

x+ y xy = 2
e.
xy(x+ y ) = 48

x3 y3 = 8
f.
x y = 2

x+ y+ xy = 5
g.
xy+ x y = 13

x2 + 3x 4y = 20
h.
2
x 2 x+ y = 5

x+ y 2(x y)

=1

x+ y
g. x y
2
2
x 5xy+ 2y = 4

3x2 2 xy y2 = 7
h.
x2 + xy+ 8y2 = 14

x y = 12
a.
x y = 108

98

Algebra 8

CHAPTER REVIEW TEST

3A

1. Find the solutions of the quadratic equation


2x + 5x + 3 = 0.
A)

1
and 3
2

D)

B) 3 and

1
and 3
2

1
2

C) 1 and

E) 1 and

3
2

C) 2

A) 8 and 6

B) 4 and 3

D) 3 and 8

C) 3 and 4

E) 8 and 3

6. What is the sum of the roots of the equation


x4 13x2 + 36 = 0?

m + n = 5. Find m.
B) 1

Find the values of h and k.

1
2

2. 1 is a root of x2 + mx + n = 0 and

A) 1

5. The sum and product of the roots of the equation


2x2 hx + 2k = 0 are 4 and 3, respectively.

D) 2

E) 3

3. Which one of the following is false for the


equation x2 7x + 1 = 0?
A) The roots are not integer.
B) The roots have the same sign.

A) 13

B) 18

C) 1

D) 0

E) 13

7. Which one of the following equations has roots


2 and 3?
A) 2x2 x 6 = 0

B) x2 + x 6 = 0

C) x2 x 6 = 0

D) x2 + x + 6 = 0

E) 2x2 + x 6 = 0

C) The roots are rational.


D) The roots are both positive.
E) The sum of the roots is positive.

8. The sum of a number and its reciprocal is


4. Find the discriminant of x2 25x + 4 = 0.
A) 2

B) 4

Chapter Review Test 3A

C) 36

D) 16

E) 24

Find the number.


A) 1

B) 1

C)

4
3

D)

1
2

25
.
12

E) 3
99

9. What are the roots of the equation

13. What is the solution of the equation x = x + 6?

1 3
+ 2 = 0?
x2 x

A) 1 and

1
2

D)

A) 9
1
and 1
2

B)

C)

1
and 1
2

B) 4

C) 4

D) 16

E) 9

1
4
and
2
3

E) 1 and 1

14. Which one of the following is not a solution of the


system
5 x2 6 xy+ 5 y2 = 29
?

7 x2 8 xy+ 7 y2 = 43

A) (2, 3)

B) (2, 3)

D) (3, 2)

10. One of the roots of the equation ax2 + bx + c = 0

C) (2, 3)

E) (3, 2)

is twice the other root. What is the relation


between the coefficients a, b, and c?
A) 4b2 = 4c

B) 2b2 = 9ac

D) b2 = 8ac

C) 2b2 = 9a

E) 9b2 = 2ac

15. Two technicians can complete a mailing job in 12


hours while working together. Alone, the first
technician can complete the mailing job seven
hours faster than the other technician. How long
will it take each technician to complete the mailing
alone?

11. The diagonal of a rectangle is 5 cm, and the area

A) 14 and 21

is 12 cm2. Find the perimeter of the rectangle.


A) 12

B) 24

C) 10

D) 14

B) 21 and 28

D) 11 and 18

C) 7 and 14

E) 15 and 22

E) 15

16. The equations 6x2 18x + 12 = 0 and


mx2 nx + 1 = 0 have the same solution. Find
m and n.
A)

12.

x 1 x+ 1 + 1 = 0. Find the value of 4x.

A) 5
100

B) 4

C) 0

D) 16

E) 4

3
1
and
2
2

D)

1
3
and
2
2

B)

3
1
and
2
2

E)

C)

3
1
and
2
2

3
1
and
2
2
Algebra 8

CHAPTER REVIEW TEST

3B

1. x1 and x2 are roots of the equation

5. The longest side of a right triangle is 3 cm less

x2 + (2 x1)x + 7x2 = 0. What is x1 + x2?


A) 2

B) 5

C) 7

D) 10

E) 14

than twice the length of the medium side. The


shortest side is 3 cm. Find the lengths of the other
two sides.
A) 8 and 15

B) 7 and 14

D) 4 and 5

C) 8 and 10

E) 6 and 8

2. Which one of the following is true for the


equation |x|2 + |x| 6 = 0?

6. A triangle has area of 2 cm2. The base of the

A) There is only one root.

triangle is 3 cm longer than its height. Find the


length of the base and the height of the triangle.

B) The product of the roots is 3.


C) The product of the roots is 4.

A) 5 and 2

D) The sum of the roots is 0.


E) The sum of the roots is 1.

B) 4 and 1

D) 7 and 4

C) 6 and 3
E) 8 and 5

7. Which one of the following equations has roots


3. The sum of the squares of the roots of the equation
x2 + 2hx = 3 is 10. Find the value of h.
A)

1
2

B) 3

C)

3
2

D)

1
2

E) 1 or 1

1 3 and 1 + 3?
A) x2 2x 2 = 0

B) x2 + 2x 2 = 0

C) x2 x 3 = 0

D) x2 2x + 2 = 0

E) x2 6 = 0

8. The sum and product of two numbers are 21 and


4. Find the discriminant of 2x 2x + 5 = 0.
2

A) 36

B) 4

Chapter Review Test 3B

C) 25

D) 16

104, respectively. What is the bigger number?


E) 9

A) 16

B) 22

C) 13

D) 14

E) 19
101

9. What are the roots of the equation

x
x
3
+7
6 = 0?
x+ 1
x+ 1

A)

3
and 2
4

D)

B)

A) 9

1
and 2
2

1
and 3
2

13. What is the solution to the equation x 2 + x 4 = 2?

C)

B) 1

C) 4

D) 4

E) 16

1
4
and
2
3

E) 1 and 2

14. Which one of the following is not a solution of the


system
x2 + y2 = 26
?

x2 y2 = 24

A) (1, 5)

B) (5, 1)

D) (5, 1)

10. Which number is a solution to the equation

C) (5, 1)

E) (5, 1)

x2 + |x 1| 3 = 0?
A) 3

B)

2
2

C)

3
2

D) 0

E) 1

15. A pipe can fill a pool four hours faster than another
pipe. The slower pipe filled the pool for seven
hours, then the other pipe was opened. The pipes
then filled the pool in two hours, working together.
How long would each pipe take to fill the pool
alone?

11. x y = 12 and x y = 108 are given. Find x + y.


A) 20

B) 24

C) 28

D) 30

E) 32

A) 14 and 18 hours

B) 8 and 12 hours

C) 11 and 15 hours

D) 9 and 13 hours

E) 10 and 14 hours

16. The roots of the equation x3 3x + 7 = 0 are p,


12. x 5 = x 3 is given. Find the value of x.
A) 5
102

B) 7

C) 1

D) 7

E) 4

q, and r. What is the product (p + 1)(q + 1)(r + 1)?


A) 8

B) 9

C) 7

D) 8

E) 9
Algebra 8

A. CARTESIAN PRODUCT AND ANALYTIC PLANE


1. Ordered Pairs
Everyone knows that in a football match different scores, like 3 1 and 1 3, have a different
meaning. When the score is written as 3 1, it is clear that the home side is the winner. But
when it is written as 1 3, we understand that the away side is the winner. A similar approach
is also valid for ordered pairs (3, 1) and (1, 3), that is, they don't have the same meaning.
Definition

ordered pair
Given that a and b are any two elements, (a, b) is called an ordered pair where a is the first
component and b is the second component.
For example, below we have ordered pairs where components are numbers, names, colors, etc.:
ordered pair

first component second component

(28, December)

28

December

(Ren, Descartes)
(9, 1)
(black, white)
(170 cm, 70 kg)
(Descartes, Ren)

Ren
9
black
170 cm
Descartes

Descartes
1
white
70 kg
Ren

Note that the pairs (Ren, Descartes) and (Descartes, Ren) are not the same since their
components are in different order.

Note
The ordered pair (a, b) is not the same as (b, a) since they are written in different orders.

What is the difference between the two clocks above?

104

Algebra 8

EXAMPLE

Solution

Give three examples for the ordered pair (x, y) supporting the equation 2x y = 4.
Let x = 0, then y = 4.
Let x = 5, then y = 6.
Let x = 1.5, then y = 7.
So three such ordered pairs may be (0, 4), (5, 6), (1.5, 7).
Here, note that we can find an infinite amount of such ordered pairs and although (0, 4) is
an answer, (4, 0) is not!

EQUALITY OF ORDERED PAIRS


Two ordered pairs are equal if, and only if, corresponding components are equal to each
other. That is, (a, b) = (c, d) if, and only if, a = c and b = d.

EXAMPLE

Solution

Given that (2x 1, 7) = (3, 3y 8), find x and y.


(2x 1, 7) = (3, 3y 8)
2x 1 = 3 and 7 = 3y 8
x = 2 and y = 5

Note
(x1, x2, x3) is named as an ordered triple.
(x1, x2, x3, x4) is named as an ordered quadruple.
(x1, x2, ..., xn) is named as an ordered n-tuple.

Check Yourself 1
1. How many different ordered pairs are given below?
(5 hours, 20 minutes) (28, December, 2003) (20 minutes, 5 hours) (Italy, Rome)
2. Given that (6, 2x y) = (x + y, 3), find x and y.
3. Complete the following ordered pairs (x, y) so they support y + 1 = 3x.
(2, ?)

(?, 0)

(0, ?)

Answers
1. 3; (28, December, 2003) is an ordered triple! 2. 1; 5 3. (2, 5), (1/3, 0), (0, 1)
Functions

105

2. Set Notation
A set is a collection of objects. Each one of these objects that
form a set is called an element of that set.
For example, the months of a year form a set with 12
elements. All even numbers form another set with an
infinite amount of elements. All dogs with seven legs is
another set with no elements. Think about the students in
your class. Is it a set? How many elements does it have?

set of strawberries!

Notation

1. We usually name sets with capital letters like A, B, C, etc.


2. If a1, a2, a3, ..., an are elements of a set A, we list all elements of this set as
A = {a1, a2, a3, ..., an} and denote the number of elements by n(A).
3. The symbol means is an element of . The symbol means is not an element of .
For example if A = {3, 5, 10}, then 3 A and 4 A.
4. The symbol means intersection and the symbol means union.
For example if A = {1, 2, 3} and B = {2, 5}, then A B = {1, 2, 3, 5} since the union
is the set of all elements that are either in A or B, and A B = {2} since the intersection
is the set of all elements that are both in A and B.
5. To denote an empty set, that is a set with no elements, we use or { }.
6. The symbol means is a subset of . For example if B = {2, 1, 3, 7, 10} and
C = {1, 3, 10}, then C B since each element in set C is also element of set B.

EXAMPLE

Given that A = {1, 4, 5, 7} and B = {all odd numbers between 2 and 8},
a. find n(A) and n(B) .
b. find A B and A B.
c. is it correct that 9 A B?
d. is it correct that A B = ?

Solution

We know that A = {1, 4, 5, 7}. If we list the elements of B, we get B = {3, 5, 7}.
a.
b.
c.
d.

106

n(A) = 4 and n(B) = 3.


A B = {1, 3, 4, 5, 7} and A B = {5, 7}.
Clearly 9 is not an element of A B. So 9 A B is correct.
Since n(A B) = 2, A B = is incorrect.
Algebra 8

Note

Certain letters are reserved for important number sets.


These are , , , and :
is the set of real numbers.
is the set of rational numbers.
is the set of integers.
is the set of natural numbers.
Note that all these sets contain an infinite amount of
elements, so it is impossible to list them.

Notation

A = [a, b] denotes the set of all real numbers between a and b making it
inclusive.
A = (a, b) denotes the set of all real numbers between a and b making it
exclusive.
A = [a, b) denotes the set of all real numbers between a and b, where b is
excluded.
A = (a, b] denotes the set of all real numbers between a and b, where a is
excluded.
Instead of A = [a, b] we can also use the notation A = {all x such that
a x b} or A = {x | a x b}.
Similar notation can be used for the other sets described above.
EXAMPLE

[a, b]
a

b
(a, b)

b
[a, b)

b
(a, b]

Given that A = [2, 3) and B = (0, 4), find A B and A B.

Solution
A

A
-2

-2

B
0

AB
-2

AB
4

Here we cannot list all of the elements of the given sets since they contain an infinite amount
of elements. A B contains elements that are in A or in B so A B = [2, 4). A B
contains elements that are both in A and B so A B = (0, 3).
Functions

107

Check Yourself 2
1. Given that A = {1, 2, 5, 9, 12} and B = {all x that are less than 14 and divisible by 3}
find n(B), A B, A B.
2. Given that A = [5, 3] and B = (2, 4], find A B and A B.
Answers
1. 4, {1, 2, 3, 5, 6, 9, 12}, {9, 12} 2. [-5, 4], (2, 3]

3. Cartesian Product
cartesian product

Definition

Let A and B be two non-empty sets. The set of all ordered pairs, whose first component is from
A and whose second component is from B, is called the cartesian product of A and B and is
denoted by A B.
For example, if A = {1, 2, 3} and B = {x, y}, then
A B = {(1, x), (1, y), (2, x), (2, y), (3, x), (3, y)} and
A B is read A cross B,

B A = {(x, 1), (x, 2), (x, 3), (y, 1), (y, 2), (y, 3)}.
Here, A B and B A are clearly different sets. The only common property between them is

not A times B.

that they have the same number of elements, which is equal to the product of number of
elements of A and B. Note that A B = B A only when A and B are equal sets.
NUMBER OF ELEMENTS OF A CARTESIAN PRODUCT
Given two sets A and B, n(A B) = n(B A) = n(A) n(B).
EXAMPLE

Solution

Given that M = {a, b, c} and N = {1, 2, 3, 4}, find the number of elements of M N and list
them.
n(M N) = n(M) n(N) = 3 4 = 12
M N = {(a, 1), (a, 2), (a, 3), (a, 4), (b, 1), (b, 2), (b, 3), (b, 4), (c, 1), (c, 2), (c, 3), (c, 4)}

The term Cartesian is used in the


name of the French mathematician
and philosopher Ren Descartes.

Listing the elements of a cartesian product, as shown above, is called the list method.
Sometimes a set may have an infinite amount of elements which will result in an endless list.
For example, think about making a list for A B where A = {yes, no} and B = {all even
numbers}.
Even though there are a finite number of elements, we may want to see the whole
representation rather than the list. For these cases the coordinate method is the most
efficient way to show the representation. To represent all the elements of a cartesian product
by the coordinate method we choose the horizontal axis for the first component and the
vertical axis for the second component.

108

Algebra 8

EXAMPLE

Given that C = {black, white} and D = {1, 2, 3, 4, 5, 6}, represent C D by:


a. the list method.

Solution

b. the coordinate method.

a. By the list method, we have


C D = {(black, 1), (black, 2), (black, 3), (black, 4), (black, 5), (black, 6), (white, 1),
(white, 2), (white, 3), (white, 4), (white, 5), (white, 6)}.
b. By the coordinate method, we have

black
white

D
6
5
4
3
2
1
C

CD

Here note that the horizontal axis denotes


elements of C and the vertical axis denotes
elements of D. Since n(C D) = 12, we
have 12 points plotted.

EXAMPLE

Solution

A chessboard is a cartesian product of


{A, B, C, D, E, F, G, H} and {1, 2, 3, 4, 5, 6, 7, 8}.

Given that A = {1, 2, 3} and B = {all x such that 3 x 5}, represent A B and B A.
Since A B and B A contain an infinite amount of elements, using the list method is not
possible. But by coordinate method we can represent all of the elements as shown below:
B
5

3
2
1
1 2 3

AB

BA

How many points are plotted on each plane?


Functions

109

EXAMPLE

Solution

Given that A = (4, 7) and B = [1, 3] represent A B.

By the coordinate method we have


B
3
1
4

AB

Note that the vertical boundaries of the rectangle are not included in A B.

4. Analytic Plane
Ren Descartes
(1596-1650)

French mathematician and


philosopher, Ren Descartes
believed science and mathematics
could explain and predict events
in the physical world. Descartes
developed the Cartesian coordinate
system for graphing equations
and geometric shapes. Modern
maps use a grid system that can
be traced back to Cartesian
graphing techniques. It is said
(although the story is probably a
myth) that Descartes came up
with the idea for his coordinate
system while lying in bed and
watching a fly crawl on the
ceiling of his room.

To graph the cartesian products whose elements are ordered pairs of real numbers, we need
a coordinate system. The rectangular or cartesian coordinate system consists of a horizontal
number line, the x-aaxis or the abscissa, which we label as x, and a vertical number line, the
y-aaxis or the ordinate, which we label as y.
The plane on which such a coordinate system is constructed is called an analytic plane or
xy-p
plane. Axes divide the analytic plane into four parts which are called quadrants. The
intersection point of axes is called the origin.
ILLUSTRATION OF ANALYTIC PLANE
y

4
quadrant 3
2
origin 1
nd

-4 -3 -2 -1 -1
-2
rd
-3
quadrant -4

1quadrant
st

(1,2)
1 2 3 4

4 quadrant
th

Just as every real number corresponds to a point on the number line, every pair of real
numbers corresponds to a point on the analytic plane. For example the pair (1, 2)
corresponds to the point that lies one unit to the right of the origin and two units up. The
first component, which lays on the x-axis, is 1. The second component, which lays on the
y-axis, is 2. The point where the components meet is called the plot. Locating this point on
an analytic plane is named plotting or graphing the plot.
110

Algebra 8

EXAMPLE

Solution

Plot the pairs (0, 2), (3, 1), (4, 2), (1, 0) on an analytic plane.
y

Note that the first component is on the x-axis and the


second component lays on the y-axis.

(-4,2)

4
3
2
1

(3,1)
(1,0)
-4 -3 -2 -1 -1
1 2 3 4
-2 (0,-2)
-3
-4

The coordinate system that maps use stems from the Cartesian coordinate system.

Check Yourself 3
1. Given that A = {1, 2} and B = {a, b, c, d}, represent A B and B A by the list method
and the coordinate method.
2. Given that M = {all x such that 1 < x 5} and N = {all x such that 2 x 7},
express M N by the coordinate method. Is it possible to use the list method?
3. Plot the pairs (1, 2), (0, 1), (3, 3), (4, 1) on an analytic plane.
Answers
B
d
c
b
a

1. {(1, a), (1, b), (1, c), (1, d), (2, a), (2, b), (2, c), (2, d)}
{(a, 1), (a, 2), (b, 1), (b, 2), (c, 1), (c, 2), (d, 1), (d, 2)}
2.

1 2

3.

7
-4

Functions

a b c d

1
-1

; no

2
1

2
M

-2

111

B. RELATIONS
1. Representation of a Relation
If A and B are two non-empty sets, then every non-empty subset of
the cartesian product A B is a relation defined from A to B.

B
Canada
Russia
Turkey
China
Ottowa
Beijing
Moscow
Ankara

For example, if A = {Ottowa, Beijing, Moscow, Ankara} and


B = {China, Turkey, Russia, Canada} we can describe a relation,
say , from A to B in the following way:
= {all pairs (x, y) such that x is the capital of y} and can be
listed as follows: {(Ottowa, Canada), (Beijing, China), (Moscow,
Russia), (Ankara, Turkey)}. Here is a subset of A B. Note that
we can choose any subset of A B to find another relation. Since
each subset is a collection of ordered pairs, a relation can simply be
defined as the collection of ordered pairs.

Definition

Above all elements of A B


is illustrated. Red ones are
elements of relation .

relation
A relation is a set of ordered pairs. The set of all the first components is called the domain
and the set of all the second components is called the range of the relation.
For example, {(0, 5), (1, 3), (1, 3), (2, 0), (3, 5)} is a relation since it is a set of ordered
pairs. Its domain is {3, 1, 0, 1, 2} and its range is {3, 0, 5}.
The relation above is represented by a list. This relation can also be represented by a table,
by a map, or by a graph as seen below:

1st component 2nd component

1st component

-1

-3

representation by a table
112

component
5

2nd

2nd component

-1
-3

-3

representation by a map

1st
component

-3

representation by a graph
Algebra 8

EXAMPLE

10

month

season

January

Winter

September

Fall

April

Spring

March

Spring

June

Summer

October

Fall

February

Winter

May

Spring

Given the following relations represented


by the table above, answer the following:
a. How is the first component related with the second component?
b. Represent it by a list.
c. Represent it by a map.
d. Represent it by a graph.
e. Find the domain.
f. Find the range.
Solution

Before we find the solution, to get rid of confusion, let us assume that we are on the
northern hemisphere since seasons are different on the southern hemisphere of the earth.
a. The first component is the month, and the second component is the season which
contains the given month.
b. {(January, Winter), (September, Fall), (April, Spring), (March, Spring), (June, Summer),
(October, Fall), (February, Winter), (May, Spring)}

Fall
Spring
Summer

season
Summer
Spring
Fall
Winter
May

Winter

d.

October
February

January
September
April
March
June
October
February
May

season

April
March
June

month

January
September

c.

month

e. The domain is {January, September, April, March, June, October, February, May}.
f. The range is {Winter, Fall, Spring, Summer}.
Functions

113

EXAMPLE

11

Given the domain {2, 1, 0, 1, 2} and the relation containing ordered pairs of the form
(x, y) such that x < y 2 and y ,
a. represent the relation by a list.
b. represent the relation by a graph.
c. find the range.

Solution

a. Choose x = 2 from the domain. Related y values should support 2 < y 2 and y .
So for x = 2, we have y = 1, 0, 1, 2. That gives us the elements (2, 1), (2, 0),
(2, 1) and (2, 2). We proceed in the same way for x = 1, 0, 1, 2 and get the final list
as {(2, 1), (2, 0), (2, 1), (2, 2), (1, 0), (1, 1), (1, 2), (0, 1), (0, 2), (1, 2)}.
b.

y
2
1
-2 -1 -1

c. The range is {1, 0, 1, 2}.


How can we represent this relation by a map?
EXAMPLE

12

Given the domain [1, 2] and the relation containing ordered pairs of the form (x, y) such
that y = x + 1,
a. if possible, represent the relation by a list.
b. represent the relation by a graph.
c. find the range.

Solution

To draw a line,
1. select two numbers
for x and find the
y value for each.
2. plot those two points of
the form (x, y).
3. connect them with a
straight line and extend
the line on each side.

a. Since the domain contains an infinite amount of


elements it is impossible to list all of the elements of that
relation.
b. We just sketch the line y = x + 1, but note that we take
the part of the line whose x values are between 1 and 2
(remember the domain!).

-1

c. As it is seen on the graph y can take any value between 0


and 3, inclusive, therefore the range is [0, 3].

Note
If the domain of a relation contains an infinite amount of elements we cannot represent it by
a list, table or map.
114

Algebra 8

EXAMPLE

13

The relation between blood types is represented


by the map on the right.

a. Explain the relation verbally.


b. Represent the relation by a list.

c. Find the domain.


d. Find the range.

Solution

AB

a. All blood types can give blood to the same type. Additionaly type 0 can give blood to any
type and type AB can get blood from any type.
b. {(0, 0), (0, A), (0, B), (0, AB), (A, A), (A, AB), (B, B), (B, AB), (AB, AB)}
c. The domain is {0, A, B, AB}.
d. The range is also {0, A, B, AB}.

Check Yourself 4
1. Represent the relation {(Monday, sunny), (Tuesday, rainy), (Wednesday, sunny),
(Thursday, cloudy), (Friday, cloudy)} by mapping and graphing.
2. Represent the relation having ordered pairs of the form (x, y) such that y2 = x in the
domain {0, 1, 4} by listing and graphing.
3. Given the domain [3, 4] and the relation containing ordered pairs of the form (x, y) such
that y = 2x + 3 represent the relation by graphing and find its range.
Answers
sunny

cloudy

rainy

rainy
sunny

cloudy

2. {(0, 0), (1, 1), (1, 1), (4, 2), (4, 2)}
2
1
-1
-2

Wednesday
Thursday
Friday

Monday
Tuesday
Wednesday
Thursday
Friday

Monday
Tuesday

1.

3.

y
9

y
x
1 2 3 4

4
x

-3
-5

Functions

; [5, 9]
115

2. Inverse of a Relation
As we learned previously, a relation is a set
of ordered pairs. The inverse of a set of
ordered pairs is obtained by interchanging
the places of the first and the second
components. Since the first and the
second components are interchanged
when we are finding the inverse of a
relation, the domain and the range are
also interchanged.

EXAMPLE

14

Given the relation {(2, 3), (0, 1), (4, 0), (2, 3)} containing ordered pairs of the form (x, y),
a. list the inverse relation.
b. find the domain and range of the inverse relation.
c. graph both of these relations.

Solution

a. We simply change places of components to find {(3, 2), (1, 0), (0, 4), (3, 2)}
b. The domain of an inverse relation is {0, 1, 3} and its range is {4, 2, 0, 2}
c. Let us plot the set of points {(2, 3), (0, 1), (4, 0), (2, 3)} and {(3, 2), (1, 0), (0, 4),
(3, 2)} on the same analytic plane using different colors:
y
y=x

4
3
2
1
-4 -3 -2 -1

-1 1 2 3 4
-2
-3
-4

Note that each point and its inverse is symmetric with respect to the line y = x.

GRAPH OF INVERSE OF A RELATION


The graph of inverse of a relation is symmetric to the original graph with respect to the
line y = x.
116

Algebra 8

EXAMPLE

15

Plot the graph of the inverse of the relation whose graph is given below.
y

Solution

Since the graph of inverse of a relation is symmetry of the original graph with respect to the
line y = x, we first draw the axis of symmetry, that is y = x, and then the reflection of the
relation. So we get the following graph:
y

y=x

Note that the domain and the


range of the relation and its
inverse contain an infinite
amount of elements.

EXAMPLE

16

Solution

Given the relation containing ordered pairs of the form (x, y) such that y = x + 2, find the
rule for the inverse relation.
If the relation was given by a list it would be enough to interchange x and y. Note that here
it is impossible to list the elements of the relation since it contains an infinite amount of
elements. However we can apply the same technique directly on the given rule for relation.
The rule for relation is y = x + 2. If we interchange x and y we have x = y + 2. Leaving y
on one side alone we get the rule for the inverse relation as y = x 2.

Note
If the rule for a relation is known, then the rule for its inverse is obtained by interchanging
x and y.
Functions

117

Check Yourself 5
1. Given the relation {(1, 2), (4, 7), (1, 0)}, find the inverse relation, its domain and the
range.
2. Plot the graph of the inverse of the relation whose graph is given below.
y

3. Given the relation containing ordered pairs of the form (x, y) such that y = 3x 1, find
the rule for the inverse relation.
Answers
1. {(2, 1), (7, 4), (0, 1)}; {0, 2, 7}; {4, 1}

2.

3. y =

x +1
3

118

Algebra 8

EXERCISES

4 .1

A. Cartesian Product and Analytic


Plane
1. Find the unknowns in the following ordered pairs.

5. For the given sets A and B list A A, B B, A B,


B A.
a. A = {all x + such that x2 < 15},
B = {all x such that x2 1 = 0}

a. (3x + 5, 4) = (2, y)
b. (x 2y, 3x + y) = (1, 3)

b. A = {months of winter in northern hemisphere},


B = {2, 5, 6}

2. Let A = {2, 3, 4}, B = {3, 7, 8, 9},


C = {1, 3, 5, 9, 10}.
a. Find A B.

b. Find n(A C).

c. Find B C.

d. Find A B C.

e. Find A B C.

f. Find n((A B) C).

6. For the given sets A and B, plot A B on an


analytic plane.
a. A = {1, 2, 3}, B = {4, 6}
b. A = {all x such that 1 < x < 4} and
B = {2, 3, 4}
c. A = {all x such that 2 < x 3} and
B = {all x such that 2 x 4}

3. For the given sets A and B find A B and A B.


a. A = {all x such that 3 < x 1},
B = {all x such that 1 < x < 2}
b. A = {all x such that x > 2},
B = {all x such that 3 x < 5}

B. Relations
7. List the following relations which have ordered
pairs in the form (x, y).
a. y = x2, the domain is {0, 1, 2, 3}.

4. Using the following find A and B.

b. 2x 3y = 1, the domain is 3 < x < 4 such


that x .

a. A B = {(1, a), (1, b), (1, c), (2, a), (2, b), (2, c)}
b.

8. Represent the following relations having ordered

pairs of the form (x, y) by mapping.

a. {(1, b), (2, d), (3, c), (5, e)}


1 2

Functions

b. x y = 1, the domain is {1, 2, 3, 4, 5}.


119

9. Represent the following relations having ordered


pairs of the form (x, y) by a graph.
a. {(1, 3), (2, 1), (1, 2), (2, 3)}

Mixed Problems
12. Plot all pairs (x, y) that satisfy the equation
y = x 1 on an analytic plane.

b. y = 2x + 1, the domain is [0, 3].


c. y = x, the domain is .
d. y x 1, the domain is .

13. In this figure a relation is


represented by mapping.
List the elements of the
inverse relation.

10. Find the inverse relation, domain and range of


the inverse relation for the following relations
having ordered pairs in the form (x, y).

a
d

b
c

a. {(winter, December), (fall, September),


(summer, June), (spring, March)}
b. y = 2x 3, the domain is [20, 34]
c. y = x2 + 2, the domain is [3, 3]

14. How many elements does A B have if A denotes


days in a week and B denotes months in a year?

11. Graph the inverse relation for the following:


a.

15. Prove that n(A B) = n(A) + n(B) n(A B).

(0,2)

(-3,0)

b.

16. Given that A = {1, 2, 3},

(2,2)
(0,0)

(4,0) x

B = {a, b}, and


C = {blue, red}, list A B C. How can you show
A B C by the coordinate method?

(2,-2)

c.

y
(-1,4)

(2,4)

17. Given that A = {1, 0, 3}, B = [5, 5], and

(0,0)

120

(3,0) x

C = [1, 4), plot the difference of A B from


A C.
Algebra 8

Many board games use principles of the cartesian product and the 10
9
cartesian coordinate system. The Game of Battleship is a navy game 8
where two players try to sink each other's hidden ships. On a piece of 76
paper two players draw 10 10 tables, the first of which represent 54
the player's ocean and the second which represents their opponent's 3
2
ocean. Horizontal grids are named by letters and vertical grids are 1
A B C DE F GH I J
named by numbers. Each player should place five ships in his own
C3 = HIT
hidden ocean as follows: Carrier(5 squares), Battleship(4 squares),
G7 = MISS
Cruiser(3 squares), Submarine(2 squares), and Destroyer(1 square). Ships may be
placed in any horizontal or vertical position - but not diagonally. Taking turns, players call
out their shots by telling an ordered pair denoting a location in their opponents ocean
and attempting to hit the opponents ships in order to sink them. When a shot is
called, the opponent immediately tells whether if it is a hit or miss. If the shot is a
hit, the opponent tells the player what kind of ship was hit. Players note shots of their
opponent in their own view and their shots in the enemys view to decide a further
strategy of play. Strategy and some luck must be combined to be the first to locate and
sink all 5 of your opponents ships in order to win the game.

The salvo game version is recommended for more experienced players. It differs mainly
in how many shots are taken in a turn by each player. Each player at the start takes a
salvo of 5 shots in his turn. Whenever a player has had one of his ships sunk, he loses
one shot for his next salvo. As his ships are removed from the game, the shots for each
salvo are reduced.
The advanced salvo game offers a challenge for the expert player. It is played as salvo,
except after a salvo of shots is called, the opponent simply announces how many hits
were made - but not where or on what ships.
Try to remember the board games you know. Do they have any relation with the cartesian
Functions
121
coordinate
system? What can be the components of an ordered pair?

A. DEFINING A FUNCTION
1. Definition
There are many variables around us. Some of them are closely related to each other and some
of them are not. For instance, the number of students in a classroom and the quantity of
oxygen in the same classroom are closely related. But the number of students in a classroom
and the temperature outside are not related at all. Functions are used to show this relation
between variables. Using that relation we can estimate the results for possible cases.
Below, the numbers on the right are related to the numbers on the left:
11
24
39
4 16
You can easily guess the rule that relates the number on the left to the number on the right.
It is: square the number. So the relation converts a number x to another number x2. We
can symbolize this as:
x x2
So,
If x = 10, the square of the number is 100

(1st sentence)

If x = 4, the square of the number is 16

(2nd sentence)

If x = 0.5, the square of the number is 0.25

(3rd sentence)

Obviously, a way of expressing the result of the rule as shown above is not very practical or
mathematical. Writing it in the form 10 100 is also not good since it is not clear which
rule we are using. That is, the meaning of can be confusing. (it may be add 90 to the
number or multiply the number by 10 as well). In order to talk about this rule, in our case
square the number, we should name it as f. When we apply this rule to x, we get x2. So f
is the rule that converts x into x2. Symbolically,
f(x) = x2.
Let us rewrite the above sentences once more:

122

f(10) = 100

(1st sentence)

f(4) = 16

(2nd sentence)

f(0.5) = 0.25

(3rd sentence)
Algebra 8

We can write each of the numbers above on the left and right as ordered pairs like
(10, 100), (4, 16), (0.5, 0.25). Note that for each number on the left, which is the first
component, there corresponds just one number on the right, which is the second
component. We know that the mathematical relation is a set of ordered pairs. Whenever the
first component of an ordered pair is associated with exactly one second component we name
that relation a function.

function

Definition

A function f is a rule that assigns to each element x in set A exactly one element y or f(x)
in set B. Set A is called the domain and set B is called the range of the function f. We name
x as the independent variable or the argument, and y as the dependent variable since the
value of y depends on x.
A function can also be

Euler's abstract
thought of as a set of
definition of function
ordered pairs whose
in 1755 in his
Institutiones Calculi first components are
Differentialis:
If some quantities so depend on
other quantities that if the latter
are changed the former undergo
change, then the former quantities
are called functions of the latter.
This denomination is of broadest
nature and compromises every
method by means of which one
quantity could be determined by
others. If, therefore, x denotes a
variable quantity, then all
quantities which depend on x in
any way or are determined by it
are called functions of it.

EXAMPLE

17

B = {(1, 2), (2, 6), (3, 9)}

FUNCTION

Different x-values
all different. The set of
all the first components
NOT A FUNCTION
of the ordered pairs is
the domain of the
Same x-values
function. The set of all
the second components
of the ordered pairs is the range of the function. Consider the set, A = {(cat, dog), (chicken,
duck), (cat, mouse)}. The set A would not be a function because the first component, cat, is
paired with 2 different second components. Consider the set B = {(1, 2), (2, 6), (3, 9)}. This
is a function since each first component has only one second component paired with it. The
domain of B is the set {1, 2, 3} and the range of B is the set {2, 6, 9}. C = {(1, 3), (1, 4), (4, 6)}
would not be a function since 1 has two components paired with it, 3 and 4.

C = {(1, 3), (1, 4), (4, 6)}

State whether the following relations define a function or not.


a. {(0, 2), (0, 3), (1, 6), (2, 4), (3, 5)}
b. {(3, 1), (1, 1), (0, 1), (1, 3), (2, 2)}
c. A relation having ordered pairs of the form (radius, area of circle)
d. A relation having ordered pairs of the form (name, surname)
e. {(1, z), (2, d), (4, f)} with the domain {1, 2, 3, 4}

Functions

123

Solution

a. In order to have a function for each value in the domain we


should have exactly one element assigned in the range.
Since 0 is assigned both to 2 and 3, this is not a function.

0
1
2
3

2
3
4
5
6

b. The domain is {3, 1, 0, 1, 2}. Since there is only one value


-2
-3
assigned for each value in the domain, this is a

-1
function. Note that in the image on the right 3 and 0 are
-1
0
both assigned to 1, but this does not prevent it from being a
1
1
function. The elements from the domain may be assigned to
3
2
the same value from the range. The important point is that
each element of the domain must not have more than one different value from the range.
c. For each given radius there is exactly one possible circle area. This relation is a function.
d. Two people with the same name can have different surnames. This relation is not a function.
e. As in the previous examples if nothing else is stated as the domain, the set of all the first
components is the domain. For this example it would be obvious to think that the domain
is {1, 2, 4}. This relation is a function if we didn't see the phrase with domain {1, 2, 3, 4}.
But the definition of a function states that every element in a domain must be assigned
by exactly one element. We can see that the element 3 from the domain remains
unassigned. Therefore this relation is not a function.

Note
Any function is a relation but any relation is not always a function.

To each person there corresponds just


one birth date.
Have you ever heard of a person without
a birth date? Or a person with two birth
dates?

124

To each satellite there corresponds just


one planet.
Have you ever heard of a satellite without
a planet? Or a satellite with two planets?

Algebra 8

Notation

If the letter f represents a function, then the notation f(x) means apply the rule f to the
number x and we read f(x) as f of x.
EXAMPLE

18

Given the function with the rule multiply a number by 3 and add 5, formulize it and find
its value when the number is
a. 3

Solution

b. 1

c. 0

d. 2

Clearly, we have a long rule for a mathematician. If we denote the number by x and the rule
by f, then we have f(x) = 3x + 5 and we need to find f(3), f(1), f(0), f(2).
a. Substituting 3 in the place of x, f(3) = 3 (3) + 5 = 4.
b. Substituting 1 in the place of x, f(1) = 3 (1) + 5 = 2.
c. Substituting 0 in the place of x, f(0) = 3 0 + 5 = 5.

f(x) does not mean


f times x.

d. Substituting 2 in the place of x, f(2) = 3 2 + 5 = 11.


EXAMPLE

19

Given the function f(x) = |2x 3|,


a. find f(1).

Solution

b. find x, if f(x) = 0.

a. f(1) = |2 1 3| = |1| = 1
b. We are looking for x value(s) for which f(x) = 0.
That means |2x 3| = 0.

| | stands for absolute value.


Absolute value of a number is
itself if it is nonnegative,
its negative if it is negative.
e.g. |25| = 25, |0.7| = 0.7

Solving this equation we get that x = 1.5.

A common way to consider functions is through the idea of


a function machine. Imagine this machine as having an input,
where items are entered, and an output, where results are
obtained. If a number is dropped into the input, a function
machine will output a single value.

Functions

125

EXAMPLE

20

Solution

Given the function f(x) = x2 3x, find f(2) + f(4).


To find f(2) + f(4) we use the formula f(x) = x2 3x twice by substituting 2 and 4 in the
place of x.
f(2) = (2)2 3(2) = 10

In general,
f(a) + f(b) f(a + b).

f(4) = 42 3 4 = 4
So, f(2) + f(4) = 10 + 4 = 14.

Notation

In mathematics, there are different notations used for functions. The expressions below are
the most frequent notations and have the same meaning:
f(x) = x2
y = x2
Since this is just a matter of notation we may also use letters like g or h to denote a function
and t, s, u, v to denote its argument. For the previous example if we had chosen g as
the function name and u as the variable, then the formula would have been g(u) = u2 3u
which would definitely give us the same result.
EXAMPLE

21

Solution

Given the function g( x) =

2 x +1
, find g(2), g(2a), g(a + 3), g(x), g(anything).
x2 1

Substituting 2 in the place of x, g( 2) =

2 2 +1 5
= .
22 1
3

Substituting 2a in the place of x, g( 2a) =

2 2a +1 4a +1
=
.
( 2a)2 1 4a2 1

Substituting a + 3 in the place of x, g( a + 3)


Substituting x in the place of x, g( x) =

2 ( a + 3) + 1
2 a +7
= 2
.
( a + 3)2 1
a + 6a + 8

2 ( x)+1 2 x+1
= 2
.
( x)2 1
x 1

Substituting anything in the place of x, g( anything ) =

2 ( anythi ng ) +1
.
(anything )2 1

Notation

f : A B means function has domain A and range B.


This notation is rarely used. When it is not used, it means that the domain is all possible real
numbers for which the function is defined and the range is all possible corresponding values.
EXAMPLE

126

22

Given the function g : such that g(u) = u + 5, find g(0), g(2), g(0.5).
Algebra 8

Solution

Substituting 0 in the place of u, g(0) = 0 + 5 = 5.


Substituting 2 in the place of u, g(2) = 2 + 5 = 3.
We cannot substitute 0.5 in the place of u since the function has integers as its domain
(note that 0.5 ), so g(0.5) is undefined.
What would you say about g(0.5) if the domain of function was ?
Expressing a function with the help of one formula may not always be possible. Sometimes
we need two, three or even more formulae to describe a function. We face these kind of
situations frequently and name such functions as piecewise defined functions.

Note
A piecewise function is defined so that for any value in the domain there corresponds
exactly one formula.
EXAMPLE

23

Solution

Given that f(2x + 1) = x + 7, find f(3).


We cannot substitute 3 in the place of x. If we do so, we get f(2 3 + 1) = 3 + 7 which gives
f(7) = 10. But we need f(3) not f(7)! To find it we will look for the value of x which will make
the inside of the expression f(2x + 1) equal to 3. Solving 2x + 1 = 3, we find x = 1.
Substituting 1 in the place of x, we get f(2 1 + 1) = 1 + 7 which gives f(3) = 8.

EXAMPLE

24

1
Given that f ( ) = x 1, find
x

a. f(2).
Solution

b. f(x).

a. We must use the same method of the previous example. To find f(2) we must find the
1
. In other words we
x
1
1
must solve = 2, which means x = . Substituting this value in the given formula we
2
x
have

value of x which will give 2 when put in the place of the expression

1
1
1
f ( 1 ) 1 or f (2) .
2
2
2
1
1
1
1
a, so x . Rewriting the formula f ( ) x 1 in terms of a we have f ( a) 1.
x
a
a
x
Since this is just a matter of notation instead of letter a we can choose any other letter
1
we like. Since we must find f(x), let us use x in place of a. So f ( x) 1.
x
Note that since we now have the formula for f(x) we are able to find any value of f
1
without repeating the procedure in part a. Try to find f(2) directly using f ( x) 1.
x
What will you get?

b. Let

Functions

127

Check Yourself 6
1. Is the relation having ordered pairs in the form of (month, its season) a function? What
about the relation with ordered pairs in the form of (season, a month of it)?
2. Given the function f(x) = x2 5x, find f(0), f(3), f(a), f(x + 1), f(a) + f(b).
x2 x if x > 0
, find f(2), f(0), f(5).
3. Given the function f ( x) =
if x 0
4
4. Given that f(2x 3) = x2 x, find f(1).

Answers
1. yes, no 2. 0, 6, a2 + 5a, x2 3x 4, a2 5a + b2 5b 3. 4, 4, 20 4. 2

2. Applied Problems
Functions are widely used for modelling real life data. In modelling functions, we must be
able to translate the verbal description into mathematical language. We do this by assigning
symbols to represent the independent and dependent variables and then finding the function
that relates these variables. Once we model a real life situation, we are able to estimate the
result for any specific value.

25

EXAMPLE

The area of a rectangle is 40 m2.


a. Express its perimeter P as a function of its width.
b. Using the required function find the perimeter of the rectangle if its width is 5 meters.

Solution

b
a
Given a rectangle with
sidelength a and b
Perimeter = 2(a + b)
Area = a b

EXAMPLE

128

26

40
m. We
x
know that the perimeter of a rectangle is twice the sum of its length and width. So the
40
perimeter can be expressed as P( x) = 2( x + ).
x

a. Let x m be the width of the rectangle. Since its area is 40 m2 , its length is

b. Simply we need to find P(5)= 2(5+


meters.

40
) = 26. The perimeter of the rectangle is 26
5

On a certain route an airline carries


8000 passengers per month, each
paying $50. The airline wants to
increase the fare. However, the
market research estimates that for
each $1 increase in fare, it will lose
100 passengers. Express the airline's
monthly income as a function of
increase in fare.
Algebra 8

Solution

Let x be the increase in fare. Then


the new fare = 50 + x,
the number of passengers = 8000 100x (each $1 increase is a decrease of 100 in passengers).
So if they increase the fare by x, the new monthly income will be
(new fare) (number of passengers) = (50 + x)(8000 100x)
Let f be the income function.
So f(x)= (50 + x)(8000 100x) or f(x) = 100x2 + 3000x + 400000.

EXAMPLE

27

A telephone company does not charge any


money for calls that lasts at most 15 seconds,
$0.2 per minute for calls that last at most 4
minutes and $0.15 multiplied by unit time after
4th minute. Express the call cost as a function in
terms of minutes.

Solution

Let x be the call length in minutes and f(x) be the cost function.
Clearly x > 0.
When x 0.25 (15 seconds = 0.25 minutes), f(x) = 0.
When 0.25 < x 4, f(x) = 0.2x.
When x > 4, besides 0.2 4 which is the cost of first 4 minutes, the caller will pay $0.15
multipled by extra time after the 4th minute.
So, f(x) = 0.8 + (x 4)0.15.
Since we have three different cases, the cost function will be expressed piecewisely:
0

f ( x) = 0.2 x

0.15 x +0.2

if 0 < x 0.25
if 0.25 < x 4
if x > 4

Check Yourself 7
1. The area of a circle is found by multiplying the square of its radius by . Write the formula
for the area of the circle function S(r) where r is the radius.
2. A gardener has 140 m of fencing for her rectangular vegetable garden. Express the area
as a function of one side length.
Answers
1. r2
Functions

2. x2 + 70x
129

3. Graph of a Function
a. Plotting Ordered Pairs
Visualization makes great use to get familiar with a specific function. The best way to
visualize a function is to graph it. The graph of a function f is the collection of ordered pairs
(x, f(x)) or (x, y) such that x is in the domain of the function.

ILLUSTRATION OF THE GRAPH OF A FUNCTION


y
y = f(x)
f(x)

(x, f(x))

Keep in mind that x is the directed distance from the y-axis, and f(x) is the directed distance
from the x-axis.

EXAMPLE

28

Given the graph of a function on the right,


a. find

f ( 2)+ 2 f (3)
.
f (0)

b. find x, if f(x) = 0.

1
-2

3
x

-1
y = f(x)

Solution

a. In this example we have no formula to use but only the graph. To find the given value we
should first find each of f(2), f(3) and f(0).
f(2) means the y-coordinate of the point whose x-coordinate is 2. Clearly the answer is
0. With the same procedure we find that f(3) = 1 and f(0) = 1.
Therefore,

f ( 2) + 2 f (3) 0 + 2 ( 1)
=
2.
f (0)
1

b. To find the values of x for which f(x) = 0, we should find all the points whose
y-coordinate is 0 and then take x-coordinates of those points as an answer. Clearly, there
are two such points (2,0) and (2,0) so the required x values are 2 and 2.
130

Algebra 8

EXAMPLE

29

Solution

Plot the graph of f(x) = x 2 with the domain {2, 1, 0, 1, 2, 3}.


In this example the domain contains a finite number of elements. Substituting 2, 1, 0, 1,
2 in the place of x, we get
f(2) = 4, f(1) = 3, f(0) = 2, f(1) = 1, f(2) = 0, f(3) = 1
which gives us the ordered pairs (2, 4), (1, 3), (0, 2), (1, 1), (2, 0), (3, 1).
So we get the following graph:
y
2
1
-4 -3 -2 -1 -1
-2
-3
-4

EXAMPLE

30

Solution

If domain is not given


with the formula, then it
is the set of all possible
real numbers for which
the function is defined.

Graph of f(x) = mx + n is
a line.

Functions

y = f(x)

1 2 3 4

Plot the graph of f(x) = x 2.


Note that the difference of this function from the one in the previous
example is that its domain is any real number for which the function is
defined. So we must have an infinite amount of points plotted on our
graph. Let us evaluate f(x) for some values of x (see the table on the
right). This will give us the following line as graph:
y
y = f(x)

2
1
-4 -3 -2 -1 -1
-2
-3
-4

2 3 4

f(x)

131

b. Intercepts of a Function
An x-intercept is the first
component of the ordered
pair (a, 0) where a is any
real number.

If the graph of a function f crosses the x-axis, then the function has an x-intercept. To find
the x-intercepts, we need to find all the x-values that support the equation f(x) = 0. Since
an equation may have more than one solution, the graph of a function can cross the x-axis
more than once.

Note
The x-intercept(s) of a function are also called the zeros or the roots of the function. A
function can have more than one x-intercept.

x-intercepts are on the x-axis.

EXAMPLE

31

Find the zeros of the following functions:


a. f(x) = 2x + 10

Solution

b. g(x) = x2 9

a. Zeros of the function support the equation f(x) = 0 which means 2x + 10 = 0. So at


x = 5 function has a zero.
b. Solving g(x) = 0 which means x2 9 = 0, we get x = 3 and x = 3.

A y-intercept is the second


component of the ordered
pair (0, b) where b is any
real number.

Similarly, if the graph of a function f crosses the y-axis, then the graph of the function has a
y-intercept. To find the y-intercepts, we need to let x = 0, and then solve for y. Since we are
talking about a function, f(0) can have at most one value. That means that the graph of a
function can cross the y-axis at most once.
Note that we used the phrase at most since it is possible to have no y-intercept if 0 is not
in the domain of function.

132

Algebra 8

Note
A function can have at most one y-intercept.

y-intercepts are on the y-axis.

EXAMPLE

32

Solution

Find the y-intercept of the function f ( x) =

x+ 3
.
x2 +1

To find the y-intercept we substitute x = 0. That gives us f (0) =

0+ 3
= 3.
02 +1

c. Vertical Line Test for Functions


By definition, a function needs to have at most one y-value
assigned to each x-value. That is, a vertical line can cross
the graph of the function at most once for any x-value.

VERTICAL LINE TEST FOR FUNCTIONS


A set of points in the coordinate plane is the graph of a function if, and only if, no
vertical line crosses the graph at more than one point.
EXAMPLE

33

Which of the following are graphs of functions? Why or why not?


a.
b.
c.
y
y
y

Functions

d.

133

Solution

a. As demonstrated below, no vertical line intersects the graph at more than one point. So
the graph belongs to a function.
y

b. Since we can find a vertical line which intersects the graph at more than one point, this
graph does not belong to a function.
y

c. Since we can find a vertical line which intersects the graph at more than one point, this
graph does not belong to a function.
y

d. Since no vertical line intersects the graph at more than one point, this graph belongs to a
function.
y

134

Algebra 8

EXAMPLE

34

Solution

Plot the graph of a function whose x-intercepts are 3, 2, 4 and whose y-intercept is 3 if the
graph passes through the point (1, 1.5).
y

Our graph should cross the x-axis at 3, 2, 4 and the y-axis


at 3. Moreover, no vertical line should intersect the graph at
more than one point and the point (1,1.5) should be on the
graph. On the right we have a possible graph. Note that we
can plot an infinite amount of different graphs using these
data.

y=f(x)

3
1.5
-3

1 2

Check Yourself 8
1. Plot the graph of the function f(x) = 2x + 4.
2. Find the x-intercept(s) of the function f(x) = 3x 1.
3. Find the y-intercept of the function g(x) = x2 2x + 7.
4. State whether the following graphs belong to functions or not. Explain your answer.
y

5. Plot the graph of any function with x-intercepts 2, 1 and y-intercept 4 so that the graph
passes through the point (3, 1).
Answers
y

1.

2.

1
3

3. 7 4. yes, no, yes 5.

-2

-2
x

3
x

-1

Functions

135

4. Domain of a Function
If a function f does not model data or come with conditions, its domain is the
largest set of real numbers x for which f(x) is a real number.
For example, an area of a circle with radius x can be modelled such that
f(x) = x2. Since the radius must be positive, the domain is the set of all
positive numbers.
The domain of a function may be stated explicitly. For example, if we write
f(x) = x + 3

x [2, 5),

then the domain is the set of all real numbers x such that 2 x < 5.
You can think of candies in the jar as
x values and the domain.

Or if we write
f ( x) =

x2 4 x + 3
2 x +1

f : .

then the domain is the set of all integers.


If the function is given by an algebraic expression and the domain is not stated explicitly, then
the domain is the set of all real numbers for which the expression is defined as a real number.
For example, the function f(x) = x 2 is defined when x 2 0. So its domain is the set
of all real numbers more than or equal to 2, that is x [2, ).
Sometimes to find the domain of a function, we find all values that make the function
undefined and throw them out of the set of real numbers.
1
is not defined at x = 0, so its domain is any real number
x
x except 0, that is, x (, 0) (0, ) or x R \ {0}.

For example, the function f ( x) =

Notation

To express the domain of a function f(x), we write x or D( f ) = .


RULES FOR FINDING THE DOMAIN OF A FUNCTION
1. The polynomial functions have any real number as their domain.
2. The rational functions have any real number except the ones that make the
denominator zero as their domain.
3. Even degreed root functions (square root, fourth degree root, etc.) have any real
number that makes the expression under the root non-negative as their domain.
4. Odd degreed root functions (cubic root, fifth degree root, etc.) have any real number
as their domain.
5. If a function contains the combination of the above functions, then the domain is
found by taking the intersection of all conditions.
136

Algebra 8

EXAMPLE

35

Find the domain of the following functions:


a. f(x) = x2 + 2x 8

Solution

b. f ( x) =

1
x+ 2

c. f ( x) = 2 x +1

d. f ( x) = 3 x 7

a. The polynomial functions are defined for any real number x. So D( f ) = .


b. We have a rational function denominator of which should not be equal to zero which
means x + 2 0, that is x 2. So D( f ) = (, 2) (2, ).
c. The square root is defined when the expression under it is nonnegative.
1
1
That means 2x + 1 0, therefore, x . So, D( f ) =[ , ).
2
2

d. The cubic root is defined for any real number x. So D( f ) = .

EXAMPLE

36

Find the domain of the following functions:


1
a. f ( x) = 2
x 2x
c.

Solution

f ( x) = 4 (12 3 x)( x2 +7) +

1
x + 2x
2

b.

f ( x) =

d. f ( x) =

1
x 5 + 3 x2 2 x
6x
1
4 2 x +7 + x3 5 x
3 x

a. Since we have a fraction, the denominator should be non-zero. That means x2 2x 0


or x(x 2) 0. That gives x 0 or x 2. So D( f ) = (, 0) (0, 2) (2, ).
b.

1
means 6 x 0.
6x
matter what x is.

x 5 means x 5 0.

x2 2 x always gives real number no

6 x 0
.
To find the domain we should solve the system
x 5 0
So D( f ) = [5, 6) (6, ).

c.

(12 3 x)( x2 +7) means (12 3x)(x2 + 7) 0.

To find the domain we should solve the system

1
means x2 + 2x 0.
x2 + 2 x

(12 3 x)( x2 +7) 0


12 3 x 0
or
since x2 +7 is always positive.

2
x( x + 2) 0
x + 2 x 0

So, D( f ) = (, 2) (2, 0) (0, 4].


d.

1
1
4 2 x +7 + x3 5 x gives a real number when
0 and 2x + 7 0.
3 x
3 x

3 x > 0
To find the domain it is enough to solve the system
.
2 x +7 0
So, D( f ) = [3.5, 3).
Functions

137

EXAMPLE

37

Write any function by using the following domains:


a. D( f ) = (, 4]

Solution

b. D( f ) = [1, 2)

a. Any real number x that is less than or equal to 4 will work for our function, that is x 4
or 4 x 0. If we think 4 x as inside of a square root function, this will give us the
domain D( f ) = (, 4]. So f ( x) = 4 x is such a function.
b. If we think the problem backwards x [1, 2) should be a solution that we get from a
x 1
x 1 0
system of inequalities
which can be a condition for the function
or
x < 2
x 2 < 0

f(x) = 2004 x 1+

1
x2

Note that these are not the only possible answers.

Check Yourself 9
1. Find the domain of the function f ( x) = 3 x2 + 2 x +8.
2. Find the domain of the function g( x) = x 1+

x+ 4
.
x2 4

3. Write any function by using the domain D( f ) = (2, 4].


Answers
1.

138

2. [1, 2) (2, )

3. f ( x) = 4 x
x+ 2

Algebra 8

B. PROPERTIES OF FUNCTIONS
1. Equal Functions
EXAMPLE

38

Solution

Are the functions f ( x) =

x2 1
and g(x) = x + 1 the same? Why?
x 1

x2 1
= x +1. But this doesnt mean that f(x) = g(x). To understand this
x 1
fact it is enough to see that D( f ) = (, 1) (1, ) and D(g) = . Since D( f ) D(g), the

We know that

functions f ( x) =

x2 1
and g(x) = x + 1 are not the same.
x 1

EQUALITY OF FUNCTIONS
Two functions f(x) and g(x) are equal if, and only if,
1. f(x) = g(x),
2. D( f ) = D(g).
EXAMPLE

39

Solution

Are the functions f(x) = (x0.5)4 (x0.25)4 and g(x) = x2 x equal? Why?
Clearly, f(x) = (x0.5)4 (x0.25)4 = x2 x = g(x)
However, note that in the original
4
f(x) = (x )4 (x
)4, which means D( f ) = [0, ).

Since D(g) = D( f ), functions f and g are not equal.

2. Even and Odd Functions


Let us consider the function f(x) = x2. Clearly,
f(2) = f(2) = 4
f(5) = f(5) = 25.
It is not hard to realize that the function f gives the same value for any number and its
negative in its domain. So we can say that f(x) = f(x) for any x D( f ). These kind of
functions are called even functions.
Let us consider the function f(x) = x3. Clearly,
f(2) = f(2) = 8
f(5) = f(5) = 125.
For this function we can say that f(x) = f(x) for any x D( f ). These kind of functions are
called odd functions.
Functions

139

Definition

even, odd functions


A function f is even if, for each x in its domain, f(x) = f(x).
A function f is odd if, for each x in its domain, f(x) = f(x).

EXAMPLE

40

Classify whether the following functions are even or odd.


a. f(x) = 3x4 4

b. f(x) = 5x3 2x
d. f ( x) =

Solution

x2 +1
3x

c. f ( x) =

2
2x 7

e. f ( x) = x + 2

We should evaluate f(x) and decide if it is the same of the given function f(x) or its negative.
a. f(x) = 3(x)4 4 = 3x4 4 = f(x). This function is even.
b. f(x) = 5(x)3 2(x) = 5x3 + 2x = (5x3 2x) = f(x). This function is odd.
2
2
=
2( x) 7 2 x 7

c. f ( x) =

This function is neither even nor odd since f(x) f(x), nor is f(x) f(x).
x2 +1
( x)2 +1 x2 +1
=
=
= f ( x). This function is odd.
3( x)
3x
3x

d. f ( x) =

e. f ( x) = ( x)+ 2 = x + 2. This function is neither even nor odd.

Note
A function is even, odd or neither. Generally functions that we face are neither even nor odd.
EXAMPLE

41

Classify whether the following functions are even or odd.


a.

Solution

f ( x) =

x3 2 x
xx

a. f ( x) =

b.

f ( x) =

2x 1
2 x +1

c.

f ( x)

x7
x 1

x7
x 1

( x)3 2( x) x3 + 2 x ( x3 2 x) x3 2 x
=
=
=
= f ( x).
( x) x
x x
x x
xx

This function is even. Here note that |x| = |x| since the absolute value of a number is
equal to the absolute value of its negative.
1
1 2x
1
x
x
2 1 2
1 2x
2 x 1
b. f ( x) = x
=
= 2 x=
= x
= f ( x). This function is odd.
x
1
1+
2
2 +1
1+
2
2
+1
+1
2x
2x
x

c. f ( x) =

x 7
x +1

x +7
x 1

x +7
( x 1)

x 7
( x +1)

x +7
x 7
=
+
= f ( x).
x
1
x +1

This function is odd.


140

Algebra 8

If f is even, then the points (x0, y0) and (x0, y0) are on the graph of the function f, which
means the graph of an even function is symmetric with respect to the y-axis.
GRAPH OF AN EVEN FUNCTION
y

y=f(x)

y0

-x0

x0

If f is odd, then the points (x0, y0) and (x0, y0) are on the graph of the function f which
means the graph of an odd function is symmetric with respect to the origin.
GRAPH OF AN ODD FUNCTION
y
y=f(x)

y0
-x0

x0

-y0

EXAMPLE

42

Classify whether the following functions are even or odd:


a.

b.

Solution

c.

d.

a. The graph is symmetric with respect to the y-axis making this function even.
b. The graph is symmetric with respect to the origin making this function odd.
c. Using the vertical line test we can easily state that the graph does not belong to a
function. So we cannot talk about the function being even or odd.
d. The graph has no symmetry with respect to the y-axis or the origin. The function is
neither even nor odd.

Functions

141

EXAMPLE

43

Complete the following graphs if they belong to an even or


an odd function.
a.

b.

Solution

In order to complete the graph for an even function just imagine the y-axis as a mirror and
draw the reflection of each point on the other side.
In order to complete the graph for an odd function connect each point with the origin and
extend that line to the other side of the origin until you have the same distance.
a.

b.

Check Yourself 10
1. Are the functions f ( x) = ( x 3)( x + 4) and g( x) = x 3 x+ 4 equal? Explain, why.
2. Classify whether the following functions are even or odd:
f(x) = x7 + 4x

f ( x) =

x2 + 2x4
x4

f(x) = 3x + 3x 2|x|

3. Complete the graph on the right if it belongs to

a. an even function.
b. an odd function.

Answers
1. no 2. odd, neither, even

3. a.

b.
x

142

Algebra 8

3. Increasing, Decreasing and Constant Functions


It is very useful to know where the graph of a function rises and where it falls. The graph
shown below rises, falls, then rises again as we move from left to right, that is as the argument
gets larger.
y

y=f(x)
C

A
a

It rises from A to B, falls from B to C, and rises again from C to D. The function f is said to
be increasing when its graph rises and decreasing when its graph falls. If there is no rise or
fall in the graph, then we say the function is constant.
Definition

increasing, decreasing, constant function


A function f is increasing on an interval I if f(x1) < f(x2) for any x1 < x2 in I.
A function f is decreasing on an interval I if f(x1) > f(x2) for any x1 < x2 in I.
A function f is constant on an interval I if f(x1) = f(x2) for any x1 < x2 in I.
GRAPH OF INCREASING, DECREASING, CONSTANT FUNCTION
y

y=f(x)

y=f(x)

y
y=f(x)

f(x1)
x1

x2

f(x1)

f(x2)
x

An increasing function

Functions

f(x2)
x1

f(x1)
x2 x

A decreasing function

x1

f(x2)
x2

A constant function

143

EXAMPLE

44

Investigate the following functions for increase and decrease:


a. f(x) = 4

Solution

b. f(x) = 2x + 5

c. f(x) = x + 4

Let us plot their graphs to see the rise and fall.


y

y
5

y=4

-2.5
x

x
y = x + 4

y = 2x + 5

a. f(x) = 4 is constant on D( f ).
b. f(x) = 2x + 5 is increasing on D( f ).
c. f(x) = x + 4 is decreasing on D( f ).

Note
A line in the form
y = mx + n
has m as its slope.

A linear function is always


1. increasing if its slope is positive,
2. decreasing if its slope is negative,
3. constant if its slope is zero.

EXAMPLE

45

State the intervals on which the function, whose graph


is given on the right, is increasing, decreasing or
constant.

-5

Solution

-2

Note that when the question is about the intervals of increase and decrease we are interested
in the argument, that is, the x-axis. Looking at the graph we see that there is a rise until
x = 5 (note that since the beginning of the graph is not fixed by a point, this rise begins at
minus infinity), there is a constant behaviour until x = 2, there is a fall until x = 1, and
finally there is a rise until plus infinity. Mathematically our answer is as follows:
The function is increasing on (, 5] and [1, ), decreasing on [2, 1], constant on [5, 2].
Note that we find the largest possible interval of increase and decrease. So the intervals have
closed brackets whenever possible.

144

Algebra 8

EXAMPLE

46

Solution

Prove that f ( x) =

5
is decreasing on (, 0.5).
2 x +1

In this problem we will use the definition. Let x1 < x2 < 0.5, then we must show that
f(x1) > f(x2) on (, 0.5).
f ( x1 ) f ( x2 ) =

10( x2 x1 )
5
5

=
2 x1 +1 2 x2 +1 (2 x1 +1)(2 x2 +1)

x2 x1 > 0

10( x2 x1 )
Since x1 < x2 < 0.5, 2 x1 +1< 0 . So,
> 0.
x
(2
1 +1)(2 x2 +1)

2 x2 +1< 0

It means that f(x1) f(x2) > 0 or f(x1) > f(x2). Thus, f ( x) =

EXAMPLE

47

Solution

5
is decreasing on (, 0.5).
2 x +1

Investigate f(x) = x2 for increase and decrease.


Note that f(x) is an even function. So lets investigate it in [0, ).
If 0 x1 < x2, then f(x2) f(x1) = x22 x12 = (x2 x1)(x2 + x1).
x2 x1 > 0
. So (x2 x1)(x2 + x1) > 0.
Since 0 x1 < x2, then
x2 + x1 > 0
It means that f(x2) f(x1) > 0 or f(x1) < f(x2).

Thus, f(x) = x2 is increasing on [0, ). Since f(x) is even, it is decreasing on the other side
of the y-axis (Think about the symmetry with respect to the y-axis). So the function is
increasing on [0, ), decreasing on (, 0].

Check Yourself 11
1. Find a if f(x) = (a 3)x + 4 is constant on D( f ).
2. State the intervals on which the function whose
graph is given on the right is increasing, decreasing
or constant.
3. Prove that f ( x) =

y
2
3

4
is increasing on (2, ).
2x

4. Investigate f(x) = x3 for increase and decrease.


Answers
1. 3

2. increasing on [2, 3], decreasing on (, 2], [5, ), constant on [3, 5]

3. consider f(x1) f(x2) when 2 < x1 < x2


Functions

4. increasing on
145

4. Reading Graph of a Function


Graphs give us the opportunity to see
the general details of a function at one
glance. In order to get the necessary
information it is important to read a
graph properly. We read a graph the
same way that we would read a book,
from left to right. The behaviour of the
function is determined by the y-values,
but the intervals are reported in terms
of the x-values.

Notation

1. If a point is plotted by a full circle, it means that the point is included in the graph.
2. If a point is plotted by an empty circle, it means that the point is not included in the graph.
3. If the endpoint of a graph is a full or empty circle, it means the part of the graph on that
side stops at that point; otherwise it means that the graph continues up to infinity.

EXAMPLE

48

Answer the following using the graph below:


y

y=f(x)

2
-6
-7

-4

5
-3

-1

-3

a. Find f(6), f(0), f(3), f(5), f(4).


b. Find the domain and the range of f.
c. Find the x and the y-intercepts of f.
d. Solve f(x) = 3.
e. Solve f(x) 0.
f. Find the intervals on which f is increasing, decreasing and constant.
g. Is f even or odd?
h. Find the minimum and the maximum value of f .
146

Algebra 8

Solution

a. We just find the corresponding y coordinate on the graph for each given x value, that is
for 6, 0, 3, 5, 4. Clearly, f(6) = 1, f(0) = 3, f(3) = 3.
f(5) and f(4) are undefined since no part of the graph has x coordinate being equal to
5 or 4.
b. Graphically finding the domain means writing all of the values of x for which a vertical line
intersects the graph of f . So, D( f ) = (, 6] (4, ).
Similarly finding the range means writing all of the values of y for which a horizontal line
intersects the graph of f . So, E( f ) = [3, ).
c. Finding the x-intercepts means finding the values of x for which the graph crosses the
x-axis. So, the x-intercepts are 7, 3 and 6.
Finding the y-intercept means finding the value of y for which the graph crosses the
y-axis. So, the y-intercept is 3.
y
y=f(x)

d. To solve f(x) = 3 means finding values


of x for which y is equal to 3. We see
that there are an infinite amount of
solutions: x [0, 5].

5
x

-3

e. To solve f(x) 0 graphically, you must


ask for which values of x is the graph of
f above or at the same level with the line
y = 0. So, the answer is
x (, 7] [3, 0) [6, ).

f. Recall that intervals are reported in


terms of the x-values and for increase,
decrease or being constant we write the
largest possible interval.

-7

y=f(x)

-3

y
y=f(x)
-6

-4

5
x

f is increasing on (4, 0) and [5, )


f is decreasing on (, 6]
f is constant on [0, 5]

g. f is neither even nor odd since its graph has no symmetry with respect to the y-axis nor
the origin.
h. The maximum value of f is the highest possible y value that the graph can reach. In our
graph since it moves as high as possible we say that the maximum value doesnt exist or
the maximum value is plus infinity.
The minimum value of f is the lowest possible y value that the graph can reach. In our
graph the minimum value is 3.
Functions

147

EXAMPLE

49

Answer the following using the graph below:


y
3
2
y=f(x)

1 2
-6

-4

-2 -1

-1

-3

a. Find the domain and the range of f.


b. Find the x and the y-intercepts of f.
c. Solve f(x) = 1.
d. Solve 3 < f(x) 1.
e. Solve f(x) > 2.
f. Solve f(x) 2.
g. Find the intervals on which f is increasing, decreasing and constant.
h. Is f even or odd?
i. Find the minimum and the maximum value of f .
j. When does f have its minimum and maximum value?
Solution

a. D( f ) = [6, 7) and E( f ) = [3, 3].


b. The x-intercepts are 1, 3 and the y-intercept is 1.
c. x {0, 2}.
d. x [0, 1) (1, 2].
e. x [6, 2).
f. x [6, 2] {5}.
g. f is increasing on [1, 5], decreasing on [4, 1] and [5, 7), constant on [6, 4].
h. f is neither even nor odd.
i. The maximum value of f is 3 and the minimum value is 3.
j. f has its maximum value when x [6, 4] and has its minimum value when x = 1.

148

Algebra 8

Graphs are widely used in many areas like statistics, economics, engineering, medicine,
meteorology, etc. Interpreting information from them in real situations is the same as reading
a graph. In fact those graphs belong to the functions that are the models of daily situations.

EXAMPLE

50

Describe a situation that can be modelled by


the graph shown.
Heart Rate

Time

Solution

EXAMPLE

51

The graph shows the heart beat rate of a person in a time interval. It increases for a period
of time and then decreases back to its initial condition. The person may have dealt with a
sport activity during the interval of increase and after finishing that activity, the heart beat
rate decreased to its normal condition.

Answer the following questions using the graph below:


a. In what period did the car sales occur?
b. How many cars were sold in April?
c. In which month were 15 cars sold?
d. In which month were the sales the best?
e. In which month were the sales the worst?
f. In which two months were the sales same?
g. In which months did the sales increase?

Car Sales
35
30
25
Number 20
of cars
15
10
5
0

Jan Feb Mar Apr May Jun


Month

h. In which months did the sales decrease?


Solution

a. The first six months of the year, that is from January to June, inclusive.
b. 25

c. March

d. May

e. June

f. January and February

g. April and May

h. March and June


Functions

149

Check Yourself 12
Answer the following using the graph below:
y

y=f(x)

(5,1)

(-4,1)
(2,0)

(0,0)

(1,-3)

1. Find the domain and the range of f .


2. Find the x the and the y intercepts.
3. Find the intervals on which f is increasing and decreasing.
4. Solve f(x) 1.
Answers

150

1. D( f ) = (, 2) (2, 5), E( f ) = [3, )

2. 0; 0

3. increasing on (1, 2) and (2, 5), decreasing on (, 1)

4. x [4, 2) (2, 5)

Algebra 8

EXERCISES

4 .2

A. Defining a Function
1. State whether the following relations are functions
or not.
a. {(1, x), (2, y), (3, x)} with the domain {1, 2, 3}
b. {(a, b), (a, c), (b, a), (c, a)} with the domain
{a, b, c}
c. {(1, 1), (2, 2), (3, 1), (4, 1)} with the domain
{1, 2, 3, 4, 5}

2. Express the following rules in a function notation.

2 x +1

5. Given that f ( x) = 3

3x

find

if x >1
if x =1,
if x <1

f (2)+ f (1)
.
f ( 1)

6. Plot the graphs of the following functions within the


given domain.
a. f(x) = x2, D( f ) = {2, 1, 0, 1, 2}
b. g(x) = 2 x, D(g) = {1, 0, 1, 2, 3}

a. Multiply by 5, then add 2.

c. h(x) = x, D(h) = {0, 1, 4, 9}

b. Square the number then subtract twice the


number.

d. f(x) = 2x + 1, D( f ) =

c. Divide by two, then add three times its cube.

7. Find the x and the y intercepts of the following


functions.

3. Express the following rules in words.


a. f(x) = 2x 4
c. h( x) =

b. g( x) = x +1

x
3x +1

4. Find the required values for the following functions.


2

a. If f(x) = 2x 5x, find


f(1), f(3), f(x), f(x), f(x), f(x + h).
b. If g( x) =

a. f(x) = 3x 7
b. f(x) = (x 6)(x + 1)
c. f ( x) =

2x 4
x +5

d. f ( x) = x + 4 5

8. State whether the following graphs belong to a


function or not.
a.

b.

3x 1
, find
x2 + 2

c.

g(0), g(4), g(3x), 2g(x), g(x2), g(2a + b).


u 2 +1
c. If h( u ) = 3
, find
u + 2u

d.

e.

f.

h(1), h(0), h(2u), h(x + 1), h(u2), h2(u).


(notation h2(u) is used in the place of (h(u))2 which means

x
x

the square of the function.)

Functions

151

9. Find the domain of the following functions.


h. f ( x) =

a. f(x) = 2x + 1

5 x
x 1

b. f(x) = x 2x + 5
c. f ( x) =

x +1
x4

d. f ( x) =

x
2
x 9

i.

f ( x) =

j.

f ( x) =

1
3

2 5x
x2 7 x +10
x2 +6 x 9

e. f ( x) = 3x 6
f.

f ( x) =

g. f ( x) =

1
2

x x+ 3
1
x 5
6x

h. f ( x) = x 5 + 30 3 x
i.

f ( x) =

j.

f ( a) =

1
5x 1

+ 1 x

1
+ 1+ a 3 2 a
a 1
3

3u 4
1
k. f ( u ) =
+
+ u3
u 2 3u 10

11. Write a formula for the function by using the

following domains.
a. [3, 5)
b. [4, 2) (2, 0)
c. [2, 1] {3}

12. For which values of x are the following functions


equal?
a. f(x) = 2x 3 and g(x) = 5x + 1
b. f(x) = x + 22 and g(x) = x2 + x + 6
c. f ( x) =

10. Find the domain of the following functions.

a. f ( x) = 15 2 x 8 x2
1
x+ 4
b. f ( x) = 2
+
x x2
5 3x

c. f ( x) =
d. f ( x) =

x+ 3

B. Properties of Functions
13. Classify whether the following functions are even,
odd or neither.
a. f(x) = x3 + x

b. f(x) = x5 + x3 1

c. f(x) = 5 x2 + x4

d. f ( x) =

x3 x2 x +1
4x + 3
x 2x 8

e. f ( x) =

g. f ( x) =

e. f ( x) = x2 +6 x +5 + x2 + x + 2
f.

f ( x) = x2 (1 x)( x 2)

g. f ( x) =

152

1
( x 1)2 (3x x2 )

3x 1
and g(x) = 3
x2

1
x 3
4

x 2x
x2 +1

f.

f ( x) =

h. f ( x) =

x 1
x2 +1
x
1+ x2
10 x 10 x
10 x +10 x

if x 0

i.

x + 2
f ( x) =
x 2

j.

3x2 +1 if x > 0
f ( x) =
2

3x 1 if x < 0

if x < 0

Algebra 8

14. Given their graphs classify whether the following

16. Plot the graph of a function by the following


intervals of increase and decrease.

functions are even, odd or neither.


a.

b.

c.

e.

f.

b. Increasing on [5, 2] and [6, 9], decreasing on


[2, 6] and [9, 11]

d.

a. Increasing on (, 2] and [5, ), constant on


[2, 5]

c. Increasing on (, 3] and [7, ), decreasing on


[4, 5], constant on [3, 4] and [5, 7]

17. Prove that:

15. Complete the following graphs to get an even


function and to get an odd function.
a.

(-4,6)

a. f(x) = 3 2x is decreasing on .
4
b. f ( x) =
is increasing on (2; ).
2x
c. f ( x) =

21x 9
1
is increasing on (; ).
3x 1
3

d. f ( x) =

4x + 31
is decreasing on (7; ).
x +7

e. f(x) = x4 8x is increasing on [2; ).


(-7,4)
(0,0)

18. Given that a f(x) = x f(x) + bx + 2 and

(-2,-2)

g(x) = x + 5, find g(ab) if f is a constant function.

19. f(x) = ax3 + bx + 1, a 0, b 0 and f(10) = 4,

b.

find f(10).

(4,4)

(0,4)

20. Answer the following using the graph below:


y

(7,-4)

y = f(x)
5

-6

(5,-6)

-2

a. Find the domain and the range.

c.

b. Find the intervals on which the function is


increasing and decreasing.

(2,4)
(7,2)

c. Find the x and the y intercepts.

(-2,0)
(0,-2)

(5,0)

d. Solve f(x 2) = 0.
e. Solve f(x) < 0.

Functions

153

21. Answer the following using the graph below:


y

23. Two ships leave the port at the same time, one
sailing east at a rate of 9 km/h and the other
sailing south at 12 km/h. If t is the time (in hours)
after their departure, express the distance d
between the ships as a function of t.

y = f(x)

2
-10

-6

-5

-3

10 x

-7

a. Find the domain and range of f(x).


b. Solve f(x) = 0.

c. Find f(0).
d. Find f(10).

e. Solve f(x) 0.
f. Solve f(x) > 2.

24. The freezing point of water is 0 C, or 32 F, and

g. Solve f(x) = 2.

the boiling point is 100 C, or 212 F.

h. Is f(x) even or odd?

a. Express the Fahrenheit temperature F as a


linear function of the Celsius temperature C.

i. Write down the intervals where f(x) is


increasing, decreasing or constant.
j. Find the minimum and the maximum value of
f(x).
k. Find all the x values for which f(x) = 7.
l. Find all the x values for which f(x) (7; 2).

b. What temperature increase in F corresponds


to an increase in temperature of 1 C?

25. A cross section of a rectangular pool with the

dimensions 60 m by 20 m is shown in the figure.


The pool is being filled with water at a rate of
10 m3 / min.
60
1

Mixed Problems
x 1

22. Given that f ( x) = x2 2 x 3

x + 4
find f ( f ... f (0)...).

2004 times

154

15

if x > 3
if 1< x 3,
if x 1

a. Express the volume V of the water in the pool


as a function of time t.
b. Express V as a piecewise function of the depth
h at the deep end for 0 h 2 and then for
2<h3
c. Express h as a piecewise function of t.
Algebra 8

26. The graph below gives the weight of a certain


person as a function of age. Describe in words how
this person's weight has varied over time. What do
you think happened when this person was 40
years old?

31. If f(x) = 2x9 3x7 + ax 3 and f(11) = 2, find

100
Weight
(kg)

x +1
= x, find f(1), f(3), f(0).
x 1

30. If f

f(11).

75
50
25
10 20 30 40 50 60 70

Age
(years)

32. Find the intervals on which the function

f(x) = x2+|5x+1|+5 is increasing and decreasing.

27. The graph gives a salesmans distance from his


home as a function of time on a certain day.
Describe in words what the graph indicates about
his travels on this day.

1
1
33. If f x = x3 3 , find f(x).

Distance
from home
(km)

34. If f(x) = ax2 + bx + c and


8 A.M.

10

NOON

6 P.M.

Time
(hours)

f(x 1) + f(x) + f(x + 1) = x2 + 1, find f(2).

28. Find the range of the following functions.


a. f(x) = 4 x2

b. f ( x) = x +1+ 3

1
c. f ( x) = 2
x +6

x3
d. f ( x) =
x+ 4

29. If f(x) + f(x + 1) = 2x + 3 and f(2) = 3, find

f(99).

Functions

35. If f : + , f(25) = 1, f ( x) =

2 f ( x +1)+1
,
2

find f(3).

1
36. If g( x)+ 2 g = x, find g(x) in terms of x.

155

A. BASIC OPERATIONS
Two functions f and g can be combined to form new functions f + g, f g, f g, and f / g in
a similar way that we add, subtract, multiply, and divide the real numbers.
For example, we define the function f + g as (f + g)(x) = f(x) + g(x). This new function is
called as a sum of functions f and g. Its value at a given x value is found by adding the value
of f and g at that x value. The operation is similar for the case of the difference, the product
and the quotient. Clearly, the new function is defined only when f and g is defined, that the
domain of the new function is also the intersection of the domains of f and g. In case of the
quotient the values that make denominator equal to zero must be excluded from the domain.
SUM, DIFFERENCE, PRODUCT AND QUOTIENT OF FUNCTIONS
Let f and g be two functions with domains A and B respectively. Then

EXAMPLE

52

Solution

Operation

Definition

Domain

Addition

( f + g)(x) = f(x) + g(x)

AB

Subtraction

(f g)(x) = f(x) g(x)

AB

Multiplication

(fg)(x) = f(x)g(x)

AB

Division

(f/g)(x) = f(x)/g(x)

A B, g(x) 0

Given that f(x) = x2 4 and g(x) = x + 3, find


a. f + g, f g, fg, f / g and their domains.

b. (f + g)(1), (f g)(1), (fg)(1), (f / g)(1).

a. Note that D( f ) = and D(g) = . So we have


( f + g)( x) = f ( x)+ g( x) = x2 4 + x + 3 = x2 + x 1 with the domain ,
( f g )( x) = f ( x) g( x) = x2 4 ( x + 3) = x2 x 7 with th e domain ,
( fg)( x) = f ( x)g( x) = ( x2 4)( x + 3) = x3 + 3 x2 4 x 12 with the domain ,
( f / g )( x) =

f ( x ) x2 4
=
with domain ( , 3) (3, ) since 3 makes t he
g( x)
x+ 3

denominator of the new function equal to zero.


2
b. ( f + g)(1) = f (1)+ g(1) = 1 4 + 1+ 3 =1

( f g )(1) = f (1) g(1) = 12 4 (1+ 3) = 7


( fg )(1) = f (1)g(1) = (12 4)(1+ 3) = 12
( f / g )(1) = f (1) / g(1) = (12 4 ) /(1+ 3) = 3 / 4
156

Algebra 8

EXAMPLE

53

Solution

Given that f(x) = 2, g(x) = 3x2 11, h(x) = 2 x 6 and m( x) =


find m(5) and the domain of m.

m( x) =
m(5) =

f ( x)+ g( x)
+ f ( x)g( x),
h( x)

f ( x) + g( x)
2 + 3x2 11
3 x2 9
+ f ( x)g( x) =
+ 2(3x2 11) =
+6 x2 22.
h( x)
2x 6
2x 6
3 52 11
2 56

+6 5 2 22 =160.

The domain of function m can directly be found by considering the final formula for m(x).
The only condition that we should deal with is 2x 6 > 0. So, D(m) = (3, ).

B. COMPOSITION OF FUNCTIONS
Aside from the four basic operations there is a very important way of combining functions to
get a new function. We call this a composition. This operation is illustrated as follows:
Here we see two different functions f and
g. Their common point is that the range
of one is the domain of the other.
Because of that fact the domain of the
function g is linked with the range of the
function f. Consider x = 2:
g(2) = 3
f(g(2)) = f(3) = 0

f(g(x)) means
g is applied first,
f is applied second.

Read f(g(x)) as
f of g of x.

In other words, 2 finally maps to 0. Note


that to find the final mapping we first use
g and then f. If we name that final
mapping as h, then h(2) = 0 where
h(x) = f(g(x)).
That final mapping h is another function
and is the composition of f and g as
shown below on the right:

g
Range of g
2
0
1
3
Domain of g

Range of f

Domain of f
f(g(x))
h
2
0
1
3

Domain of h

1
0
1
Range of h

As an algebraic example if f(x) = x and g(x) = 3x + 1, we know that f(x) is the rule take
the square root of the number x and g(x) is the rule multiply the number x by 3 and add
1. In that case f(g(x)) means you must first multiply the number x by 3 and add 1, then take
the square root of the result so we can define h( x) = f ( g( x)) = g( x) = 3x +1. Here we
say that h(x) is a composition of f(x) and g(x), that is h(x) is a composite function.
The domain of f(g(x)) is the set of all x in the domain of g such that g(x) is in the domain of f.

Functions

157

Composite functions are composed of bridges that link the domain of first with the range of last.

composition

Definition

f(g(x)) is defined as composition of f and g. It is also denoted by (f g)(x) or simply f g.


fog

g
x

f
g(x)

f(g(x))

Arrow diagram for (f o g)(x)


EXAMPLE

54

Solution 1

Given that f(x) = x2 + 3x, g(x) = x 4, find f(g(x)) and g(f(x)).


f(g(x)) = g(x)2 + 3g(x) = (x 4)2 + 3(x 4) = x2 8x + 16 + 3x 12 = x2 5x + 4
g(f(x)) = f(x) 4 = (x2 + 3x) 4 = x2 + 3x 4
So f(g(x)) = x2 5x + 4 and g(f(x)) = x2 + 3x 4.
What can you say about f(g(x)) and g(f(x))? Is it true that they are the same?

Solution 2

f(g(x)) = f(x 4) = (x 4)2 + 3(x 4) = x2 5x + 4


g(f(x)) = g(x2 + 3x) = (x2 + 3x) 4 = x2 + 3x 4

Note
In general, f(g(x)) g(f(x)).
158

Algebra 8

EXAMPLE

55

Let f(x) = x + 1, g(x) = x3. Find:


a. (f g)(2)
b. (g f)(1)
c. (f f)(5)
d. (g g)(1)

Solution 1

a. (f g)(x) = f(g(x)) = g(x) + 1 = x3 + 1


(f g)(2) = 23 + 1 = 9
b. (g f)(x) = g(f(x)) = (f(x))3 = (x + 1)3
3

(g f)(1) = (1 + 1) = 8
f g means g is applied
first, f is applied second
and in general f g g f.

Composite functions are formed of functions that are


inside another function.

c. (f f)(x) = f(f(x)) = f(x) + 1 = x + 2


(f f)(5) = 5 + 2 = 7
d. (g g)(x) = g(g(x)) = (g(x))3 = x9
(g g)(1) = (1)9 = 1

Solution 2

Note that these values can be calculated without finding the formula.
a. In case of ( f g)(2) we may first calculate g(2) = 8 and then f(8) = 9 to find the answer
since ( f g) (2) = f(g(2)) = f(8) = 9.
b. (g f )(1) = g( f(1)) = g(2) = 8
c. ( f f )(5) = f( f(5)) = f(6) = 7
d. (g g)(1) = g(g(1)) = g(1) = 1

EXAMPLE

56

Solution 1

Let f ( x) =

x
, g(x) = x5 and h(x) = x2 + x. Find f g h.
x 1

Here we have a composition of three functions.


( f g h)( x) = f ( g( h( x))) =

Solution 2
Functions

g( h( x))
( h( x))5
( x2 + x)5
=
= 2
.
5
g( h( x)) 1 ( h( x)) 1 ( x + x)5 1

( f g h)(x) = f(g(h(x))) = f(g(x2 + x)) = f((x2 + x)5) =

( x2 + x)5
.
( x2 + x)5 1
159

EXAMPLE

57

Solution

Write the function h( x) = 5x + 3 as a composition of two functions.


Let us read what the formula tells us to do: first add 3 to five times the number x and then
take the square root of all. Let f(x) = 5x + 3 and g(x) = x. Note that f is applied first, g is
applied second. So h(x) = (g f)(x).
The composite function form is never unique.
For example, consider the previous example:
If n is any nonzero integer, we could choose
f(x) = (5x + 3)1/n and g(x) = xn/2.
Thus, there are an infinite amount of composite
function forms. Generally, our aim is to choose a
formula such that the expression for the function
is simple, as in the above example.

EXAMPLE

58

Solution

Given f(x) = 3x + 1 and g(x) = 2x m such that (f g)(x) = (g f )(x), find g(

1
).
10

( f g)(x) = f(g(x)) = 3 g(x) + 1 = 3(2x m) + 1 = 6x 3m + 1


(g f )(x) = g( f(x)) = 2 f(x) m = 2(3x + 1) m = 6x + 2 m

1
Since ( f g)(x) = (g f )(x), 6x 3m + 1 = 6x + 2 m which gives m = .
2
1
1
7
So we have g(x) = 2 x + . Therefore, g( ) = .
2
10
10
EXAMPLE

59

Solution

If the rule for function f g h is first take the square root of a number plus twenty-five,
second divide the new number plus 2 by itself, third take the cube of the newest number
and p(x) = 3x2 + 6x. Find (f g h)(x) and (h p f)(2).
First of all let us find formulae for f, g and h.
Since f g h means h is applied first, g second and f third we have
h( x) = x + 25, g( x) =

x+ 2
, f ( x) = x 3.
x
3

x + 25 + 2
According to the rule, ( f g h)(x) =
.

x + 25

Now let us find (h p f)(2):

(h p f )(2) = h(p( f(2))) = h(p(8)) = h(144) = 13.


160

Algebra 8

EXAMPLE

60

Solution

Given that f(g(x)) = 4x 1 and g(x) = x + 2, find f(x).


Since f(g(x)) = 4x 1 and g(x) = x + 2 we have f(x + 2) = 4x 1.
Let x + 2 = a. Then x = a 2.
Substituting x = a 2 in f(x + 2) = 4x 1 we get
f(a 2 + 2) = 4(a 2) 1 or f(a) = 4a 9.
Since this is just a matter of notation in the final formula instead of a we choose x to get
f(x) = 4x 9.

Check Yourself 13
x +1
and g(x) = x, find f + g, f g, fg, f / g.
2x 4
2. Given that f(x) = x2 and g(x) = 3x + 4, find (f g)(1) and (g f)(1).

1. Given that f ( x) =

3. Write the function h(x) = (2x 7)5 as a composition of two functions such that
h(x) = (f g)(x).
Answers
2
2
2
1. 2 x 3x +1 , 2 x +5 x +1 , x + x , x2+1
2x 4
2x 4
2x 4 2x 4x

Functions

2. 49, 7 3. f(x) = x5, g(x) = 2x 7

161

C. INVERSE OF A FUNCTION
1. One-to-one Functions
Consider the two functions represented by a map as shown below:
f

4
5

Each number in the domain of f takes a different value


from the range. But it is not the same for the function g
since 1 and 2 take the same value in the range. We
name a function one-to-one if each element in the
domain corresponds to exactly one element from the
one and g is not one-to-one.
range. So here f is one-tto-o
To understand whether a given formula belongs to a
one-to-one function or not we need an algebraic tool.
The following definition for one-to-one function meets
our needs:
one-tto-o
one function

Definition

one if, for each x1 x2 in its domain, f(x1) f(x2).


A function f is one-tto-o
EXAMPLE

61

Which one of the following is a one-to-one function?


a. f(x) = 2x 1

Solution

b. f(x) = x2

Let us use the definition for one-to-one function to find out whether they support the
definition or not:
a. Let x1 x2. Then f(x1) = 2x1 1 and f(x2) = 2x2 1. Lets assume that f(x1) = f(x2), so
2x1 1 = 2x2 1 which means x1 = x2. But this is wrong since we let x1 x2! So our
assumption that f(x1) = f(x2) is also wrong. That means when x1 x2, we have f(x1) f(x2).
Therefore, the function is one-to-one.
b. Clearly, if we choose 2 2 we get f(2) = f(2) = 4. Although we can find an infinite
amount of such examples, just one of them is enough to decide that function is not
one-to-one.

162

Algebra 8

We know that the graph of a relation is a function if any vertical line


one if
crosses the curve at most once. Similarly, a function is one-tto-o
any horizontal line crosses the curve at most once. For example:
y

Graph of
a function which is not one-to-one

Graph of
a one-to-one function

HORIZONTAL LINE TEST FOR THE ONE-TO-ONE FUNCTIONS


A function is one-to-one if, and only if, no horizontal line crosses its graph at more than
one point.
Note that in the previous example we could have found the answers without using the
definition but just by plotting their graphs and applying the horizontal line test.
EXAMPLE

62

Which one of the following graphs belong to one-to-one functions?


a.

b.

Solution

c.

a. This graph does not belong to a one-to-one function.


b. This graph does not belong to a function, so we can not talk about one-to-one function.
c. This graph belongs to a one-to-one function.

Note
A function that is only increasing or decreasing in its domain is one-to-one.
Functions

163

Check Yourself 14
3x 1
is a one-to-one function.
5
2. Draw a graph which belongs to a one-to-one function.

1. Prove that f ( x) =

3. Draw a graph which does not belong to a one-to-one function.


Answers
1. Compare f(x1) and f(x2) when x1 x2.
2. Consider a function which is either increasing or decreasing.
3. Consider a function which is sometimes increasing and sometimes decreasing.

2. Definition for The Inverse of a Function


As mentioned earlier in this book, many mathematical relations can be modelled as functions.
For example,

C = f(r) = 2r

V = g(a) = a3

The circumference of a circle


is a function of the radius r.

The volume of a cube is


a function of the edge a.

In many cases we are interested in reversing this correspondence determined by a function:

r m C

C
2

The radius of a circle is a function


of the circumference C.

a n V 3 V

The edge of a cube is a


function of the volume V.

As illustrated above, reversing the relation between two quantities produces a new function.
164

Algebra 8

Recall that a function is a relation that assigns to each element in the domain exactly one
element from the range.

f
x0

y0

Domain

Range

In the figure above we see an arrow diagram for function f. So Using the rule f we say that
x0 becomes y0. We symbolize this as f(x0) = y0.

Read f (x) as
f inverse of x.

Now consider this question: Using the rule f what gives us y0? Of course the answer is x0. So
Using the rule f to get y0 we need x0. We need a new notation so that x0 will be our
answer. And we symbolize this fact as f 1(y0) = x0.

1
.
f ( x)
1
The reciprocal
is
f ( x)

f 1(x) doesnt mean

f 1

written as (f(x))1.

x0

y0

Range

Domain

Note
The domain of a function is the same as the range of its inverse.
The range of a function is the same as the domain of its inverse.

The domain and the range change their places in an inverse of a


function just as the server and the opponent change their places
in a table tennis game.

Functions

165

EXAMPLE

63

Given the function f = {(0, 2), (1, 4), (4, 6), (5, 5)},
a. find f 1.

Solution

b. find f 1(4).

a. Interchange the x and y-coordinates of each ordered pair of f to find f 1:


f

= {(2, 0), (4, 1), (6, 4), (5, 5)}

b. To find the value of f 1(4), notice that f 1(4) is the second coordinate when the first
coordinate is 4 in the function f 1. So f 1(4) = 1.
EXAMPLE

64

Solution

If f(x) = 2x 3, find f 1(5).


If we substitute 5 in place of x we find f(5) = 2 5 3 = 7 but not f 1(5)!
Finding f 1(5) means finding the x value for which f(x) = 2x 3 gives 5 as a result.
2x 3 = 5, so x = 4 or f 1(5) = 4.
If f 1 also supports the definition for a function, we will call f 1 as the inverse of f. But this is
not always possible. For example, consider the relation below:
-3

-1

-2
-1

1
2

Domain

Range

Clearly, this is a function since each element is assigned to exactly one element. Now lets
consider its inverse:
-3

f 1

-1
0
1

-2
-1
1

Range

Domain

As we see f 1(1) = 3 or f 1(1) = 0 is not possible for a function (just one element must be
assigned for each element in the domain). So although f is a function, f 1 is not a function.
That means the inverse of f doesnt exist.

Note
It is not true that every function has an inverse.
For the inverse of a function to be defined, it is necessary that different elements in the domain
always give different values. As we know such functions are called one-to-one functions.
166

Algebra 8

As another example let us consider the following function graph of which is given below:
y
y = f(x)

To find the inverse, we use the same procedures that we used for relations. Drawing the
reflection with respect to y = x we get the following picture (below left):
y

y = f 1(x)

y = f 1(x)

y = f (x)
x

y=x

We know that a set of points in the coordinate plane is the graph of a function if, and only if,
no vertical line crosses the graph at more than one point. But that rule doesnt hold for f 1
(look at the picture above right). That means the inverse of f doesnt exist. Note that the
inverse we draw will be a function if f has no horizontal line crossing the graph at more than
one point, that is if f is one-to-one.
We know that the range of a function becomes the domain of its inverse. We also know that
no element from the domain of a function must be left unassigned.
Sometimes the range of a function is explicitly so that it is larger than the functions its real
range, that is some elements of the range are not used. In that case when we talk about the
inverse some elements in the domain of inverse will be unassigned. This will result in an
absence of the inverse function.
For example, if f(x) = x + 2 such that f: , although the range seems to be it is in
fact (Note that when we put any integer in x + 2, we always get other integers).
To guarantee that we will not face such functions we must be sure that any element in the
range is assigned by an element from the domain. Such functions are called onto functions.
CRITERIA FOR EXISTENCE OF INVERSE OF A FUNCTION
A function has an inverse if, and only if, it is one-to-one and onto.
If a function is given by only formula or graph (where no explicit range is given), it is
naturally an onto function. So there is no need to think about this detail to define its inverse.
Functions

167

Now we can give the formal definition for the inverse of a function:
inverse function

Definition

Let f be a one-to-one and onto function with the domain A and the range B. Then its inverse
function f 1 has the domain B and the range A such that f(x) = y f 1(y) = x.
By definition the inverse function f 1 undoes what f does. That is, if we take x, apply f, and
then apply f 1, we arrive back at x where we started. Similarly, f undoes what f 1 does. That
is why f and f 1 are the inverses of each other.

Note
f(f 1(x)) = x and f 1(f(x)) = x.
EXAMPLE

65

Solution

Prove that f(x) =x3 and g(x) = 3 x are inverses of each other.
When we find f(g(x)) = ( 3 x )3 = x, we can see that f(x) and g(x) are the inverses of each
other. Note also that g(f(x)) = x.

EXAMPLE

66

Solution

If f(x) = x + 2 and f 1(x) = x a, find a.


We know that f(f 1(x)) = x. So f(x a) = x or (x a) + 2 = x. That gives a = 2.

3. Finding The Inverse of a Function


Given the graph of a function, to find the inverse we use the following procedure:
1. Verify that the function is one-to-one by applying the horizontal line test.
2. Take symmetry of the graph of the function with respect to the line y = x.
EXAMPLE

67

Answer the following using the graph on the right:

(7,11)
y = f(x)

a. Does f have an inverse? Why?

(6,6)

b. Draw the graph of the inverse function f 1.


c. Find f 1(-6), f 1(-3), f 1(0), f 1(6), f 1(11).

(0,-3)

(4,0)

d. Find the domain and the range of f .


Solution

(-10,-6)

a. Applying the horizontal line test we can see that f is one-to-one. So it has an inverse
y

168

Algebra 8

b. We draw the line y = x and reflect f with respect to it to draw the graph of f 1:
y
(11,7)
(0,4)

(6,6)

y = f (x)

(-3,0)
x

(-6,-10)

c. f 1(6) = 10, f 1(3) = 0, f 1(0) = 4, f 1(6) = 6, f 1(11) = 7.


d. f 1 can take any x between 6 and 11, inclusive, so the domain is [6, 11]. The value y of
f 1 can be any number x between 10 and 7, inclusive, so the range is [10, 7].

Given the formula for a function, to find the inverse we use the following procedure:
1. Verify that the function is one-to-one.
2. Solve the equation y = f(x) for x and interchange x and y in the end.

EXAMPLE

68

Given that f(x) = 2x + 5, answer the following:


a. Does f have an inverse? Why?
b. Find the formula of the inverse function f 1.
c. Find f 1(19), f 1(1).
d. Find the domain and the range of f 1.

Solution

a. f is a linear function which is always increasing. That means it is one-to-one and so it has
an inverse.
y5
.
b. Let y = 2x + 5. Then x =
2
Interchanging x and y we have the inverse function as y =

x5
x5
or f 1(x) =
.
2
2

1 5
x5
19 5
1
= 2.
we get f 1(19) =
= 7 and f (1) =
2
2
2
Note that we could also solve equations 2x + 5 = 19 and 2x + 5 = 1 to find f 1(19) and

c. Using the formula f 1( x) =

f 1(1), respectively.
d. Clearly, D( f 1) = and E( f 1) = .
Functions

169

FINDING THE INVERSE OF A FUNCTION


To plot the graph of the inverse of a function, reflect the graph of the function with
respect to the line y = x.
To find the formula for the inverse of a function, solve the equation y = f(x) for x and
interchange x and y.
EXAMPLE

69

If possible draw the graphs of inverses for the following functions:


a.

b.

y
y = g(x)

(0,4)

y = f(x)

(-6,0)

Solution

(-3,3)

(2,3)

(0,0)

a. Clearly (horizontal line test), f is one-to-one. So the inverse is the symmetry of f with
respect to the line y = x:
y
y = f 1(x)

y = f (x)
(0,4)
(-6,0)

(4,0)

(0,-6)

b. g is not one-to-one (horizontal line test). We cannot draw the graph of the inverse
function since it doesnt exist.
EXAMPLE

70

Given that the following one-to-one functions, find their inverses.


a. f(x) = 4 3x

Solution

b.

f ( x) =

2x 6
3

c. g(x) = x3 5

Since we know that the functions are one-to-one, we can find their inverses directly
a. Let y = 4 3x. Then x =
b. Let y =

4 y
4 x
4 x
. Interchanging x and y: y =
or f 1( x) =
.
3
3
3

2x 6
3y +6
3x +6
3 x +6
. Then x =
. Interchanging x and y: y =
or f 1( x) =
.
3
2
2
2

c. Let y = x3 5. Then x = 3 y +5. Interchanging x and y: y = 3 x +5 or g 1( x) = 3 x +5.


170

Algebra 8

EXAMPLE

71

If possible find the inverses of the following functions:


a.

Solution

f ( x) =

2x + 4
3x 5

a. We can verify that f ( x) =

b.

f ( x) = 3 2 x 1+7

c. f(x) = x2 2x

2x + 4
is one-to-one as follows:
3x 5

Let x1 x2. Then assume that f(x1) = f(x2). So,


2 x1 + 4 2 x2 + 4
=
3x1 5 3x2 5
6 x1x2 10 x1 +12 x2 20 = 6 x1x2 +12 x1 10 x2 20
22 x2 = 22 x1
x2 = x1.

But this is not correct since we let x1 x2. This means that if x1 x2, then f(x1) f(x2).
So f is one-to-one. Now lets find its inverse:
f ( x) = y =

2x + 4
3x 5

3xy 5y = 2x + 4
3xy 2 x = 4+5 y
x(3y 2) = 4+5 y
x=

4+5 y
3y 2

Interchanging x and y we have y =

4+5 x
4+5 x
or f 1( x) =
.
3x 2
3x 2

b. We leave the verification of the fact that f is one-to-one to student and proceed to finding
its inverse.
y = 3 2 x 1+7
y 7 = 3 2x 1
( y 7)3 = 2 x 1
( y 7)3 +1
=x
2
( x 7)3 +1
( x 7) 3 +1
or f 1( x) =
.
2
2
c. Choosing x = 0 and x = 2 we can realize that f will give the same value. So f is not
one-to-one. Proof of this using definition is left to the student in exercises. As a result f
does not have an inverse.

Interchanging x and y we have y =

Functions

171

Check Yourself 15
1. If f(2) = 3, f(5) = 6, f(6) = 1, find f 1(1), f 1(3), f 1(6).
2. If possible draw the inverse of f graph of which is given on the right.
3. Find the inverse of f ( x) =

(-3,2)

2x + 3
.
7

(0,0)

Answers
1. 6, 2, 5

y = f(x)

x
(1,-3)

2.
(3, 1)

3. f 1( x) =

(0, 0)

7x 3
2

(2, 3)

The functions which are not one-to-one can


have their domains restricted so that they
become one-to-one. As a result their inverses
will then be functions.

EXAMPLE

72

Given that f(x) = x2 + 1,


a. find the largest possible domain so that the inverse exists.
b. find f 1(x) using the new domain.

Solution

a. The inverse exists if we have a one-to-one function. Note that for each value of x and its
negative we have the same value of y. So, if we ignore the negative x values, we will have
a one-to-one function. That means to have an inverse, domain must be restricted to [0, ).
b. Let y = x2 + 1 and we will try to solve the equation for x: x2 = y 1, so x = y 1.
If we interchange x and y we have y = x 1. Note that the range of the inverse
function is the domain of the original function, that is [0, ). So y = x 1. Therefore,
1
the inverse function is f ( x) = x 1.

172

Algebra 8

EXAMPLE

73

Solution

Prove that f(x) = x2 2x 2 has an inverse if D( f ) = [1, ) and then find its inverse.
f(x) = x2 2x 2 = x2 2x + 1 3 = (x 1)2 3.
Here (x 1)2 is equal to the same number for x 1 = x0
and x 1 = x0 for any x0 > 0.
This fact prevents f(x) = (x 1)2 3 from being a
one-to-one function. But if we guarantee that x 1 is
never negative, then the function will be one-to-one. And
that is possible when x 1 0, x [1, ).
To find the inverse, we solve y = (x 1)2 3 for x:
y = ( x 1)2 3
y + 3 = ( x 1)2
y + 3 =| x 1|
Since x 1 0 we have

y+ 3 = x 1 or

y+ 3 +1= x.

Interchanging x and y we have f 1( x) = x + 3 +1.

EXAMPLE

74

Solution

x +1
1
3
If f 1
= x , find f (2) + f(8).
x

Let us find f 1(2). We dont have the formula for f 1(x) but for f 1(

x +1
). The expression
x

inside the brackets must be equal to 2 since we are looking for f 1(2).
When we solve

x +1
= 2, we find x = 1.
x

x +1
) = f 1(2) =13 1.
x
1 x +1
) = x3
Finding f(8) is simple. Note that f is inverse of f 1. So if we need f(8), the result of f (
x
x +1
x +1
must be equal to 8, that is f 1(
will be f(8) for the corresponding x value.
) = 8, where
x
x

That means when x = 1, f 1(

So, x3 = 8, x = 2 and

x +1 2+1 3
=
= .
x
2
2

3 5
Finally, f 1(2)+ f (8) =1+ = .
2 2
Functions

173

EXAMPLE

75

Using the graph below find (f g1 f 1)(5) + (f g f)(3).


y
y = f(x)

1
1

x
y = g(x)

Solution

Note that we dont have the graph of any inverse function and we dont need them at all. To
find the value of a function just find the y-value for the given x-value on the graph. To find
the value of an inverse function just find the x-value for the given y-value on the graph.
(f g1 f 1)(5) = f(g1(f 1(5))) = f(g1(4)) = f(0) = 2
(f g f)(3) = f(g(f(3))) = f(g(0)) = f(4) = 5
So (f g1 f 1)(5) + (f g f)(3) = 2 + 5 = 7.

EXAMPLE

76

Solution

If (f g)(x) = 5x 2 and f(x) = 4x 3, find g1(x).


(f g)(x) = f(g(x)) = 4 g(x) 3 = 5x 2
Solving the last equation for g(x) we have g( x) =
To find the inverse, we solve y =

5x +1
4y 1
for x: x =
4
5

1
Interchanging x and y we have g ( x) =

EXAMPLE

77

Solution

4x 1
.
5

Given f(x) = 2x 7, find f 1(4x + 1).


Let us find f 1(x): f ( x) = 2 x 7 = y
y +7
x=
2
x +7
f 1( x) =
2
Now let us find f 1(4x + 1): f 1( 4x +1) =

174

5x +1
4

(4 x +1)+7
= 2x+ 4
2
Algebra 8

EXAMPLE

78

Given that f(x) = 3x + 1 and g(x) = 2x + 2, find


a. (f g)1

Solution

b. g1 f 1

a. Let us find f g.
y = (f g)(x) = f(g(x)) = 3(2x + 2) + 1 = 6x + 7
To find the inverse, x =
(f g)1(x) =

y7
6

x7
.
6

b. Let us find f 1 and g1.


y = f(x) = 3x + 1
To find the inverse, x =
f 1(x) =

y 1
3

x 1
.
3

y = g(x) = 2x + 2.
To find the inverse, x =

y2
2

x2
.
2
x1
2
x7
(g1 f 1)(x) = g1( f 1(x)) = 3
=
2
6

g1(x) =

Note that we have (f g)1(x) = (g1 f 1)(x).


Is it always correct? Why?

Check Yourself 16
1. Given that f(x) = x2 4, restrict the domain so that the inverse exists and find its formula.
2. Given that f(2x + 1) = x 5, find f 1(0).
3. Given that f(g(x)) = 2x + 1 and g(x) = x + 4, find f 1(x).
4. Given that f(x) =

x 1
and g(x) = 3x, find (f g)1 and g1 f 1.
4

Answers
1. [0, ),
Functions

x+ 4

2. 11 3.

x +7
2

4.

4x +1 4 x +1
,
3
3
175

EXERCISES

4 .3
5. Find the required values using the given data:

A. Basic Operations
1. Find f + g, f g, fg, f / g for the following functions.

a. f ( x) =

a. f(x) = x + 1, g(x) = x 1

3x +5
x+ 2
and g( x) =
,
x +1
x 1

( f g f )(0) = ?

b. f(x) = x3 + 3x2, g(x) = x2 + 5x

x +1
2
b. f(2x + 1) = 3x 1 and g
= x +1,
x 1
( f g)(1) = ?

c. f ( x) = x + 3, g( x) = x + 2
d. f ( x) = x 1, g( x) = x +1

c. f (3x 1) =

2. If

1
f ( x) = 3x + 4, g( x) = x 2 + x, h( x) = ,
x

( f g)(1) = ?

find

2 x +1 if x > 2
d. f ( x) =
and
if x 2
x 2

the following functions.

b. f h ( 1)
g

a. (f + g)(3)

if x > 0
x
g( x) =
if x 0
x
( f g f )(2) = ?

c. (hg f)(4)

B. Composition of Functions

6. If f(x) = x3 3x2 + 3x 1 and g( x) =

3. Find f f, f g, g f, g g for the following

solve ( f g)(x) = 0.

functions.
a. f ( x) =

x +1
, g( x) = x2 + x
x 1

(g f )(x) = 3f(x) 1,

b. f(x) = x 2x + 1, g(x) = x + 1

find a such that ( f + g)(a) = 19.

8. If f ( x) =

4. Express h(x) in terms of f(x) and g(x) such that


h(x) = (f g)(x).

C. Inverse of a Function
functions or not.
a. {(x, 1), (y, 1), (z, 2), (k, 3)}

x4
b. h( x) =

b. {(1, 1), (2, 2), (3, 3), (4, 1)}


3

c. h(x) = (2x x) (2x x) + 4


176

1 x
, find f f ... f ( x).

1+ x
2004 times

9. State whether the following relations are one-to-one

a. h( x) = 2 x 4

2 x +1
,
x+ 3

7. If ( f g)(x) = 5g(x) + 4 and

2x + 3
x+ 3
c. f ( x) =
, g( x) =
x
3x 2

x+ 3
x2
and g( x+1) =
,
2x 1
x 1

c. {(a, b), (b, a), (c, d), (d, c)}


Algebra 8

10. State whether the following functions are one-to-one

13. Plot the graphs of inverse functions for the

or not.

following functions, if possible:

a. f(x) = x2 1, D( f ) = {0, 1, 2, 3}

a.

b.

(4,5)

b. f(x) = x2, D( f ) = +

(0,4)

c. f(x) = x2
d. f(x) = 3x + 5
e. f ( x) =

x 1
x2

(2,-2)

(-8,-3)

c.

(-5,-5)

(5,7)

d.

11. State whether the following graphs belong to


b.

(0,3)

(0,0)

(2,-3)

(-6,-3)

one-to-one functions or not.


a.

(-6,7)
(-5,4)

(4,1)
(-2,-1)

(5,-5)

e.

(6,-5)

f.

y
(0,5)

(-5,3)

(4,-2)

(6,-5)

c.

14. Find the required values using the given data for
the functions with the inverse.

2 x +5 1
, f (3) = ?
5
b. f 1(x) = x2 3, D(f 1) = [0, ), f(3)= ?

a. f ( x) =

12. Find the inverses of the following one-to-one


functions:
3x + 4
x 1

a. f(x) = 6 4x

b.

f ( x) =

c. f(x) = 4x3 5

d.

f ( x) = 3 5 x 1+9

Functions

c. 2 f ( x) 1=

f ( x)+1 1
, f (2) = ?
x2

d. f (3x +1) =

4 x 1 1
, f (3) = ?
x2

e. f(x2 + x + 1) = x 2, D( f ) = , ,
4

f 1(0) = ?
177

15. Find the following using the given data for

18. Find the following using the graph of functions

functions with inverses:

y = f 1(x) y

2 x +1
1
a. f
= x +1, f ( x) = ?
x 1

b. f ( x 1) =

below:

3x 2 1
, f ( x) = ?
x + 21

3x 5
1
c. f
= x, f (2 x) = ?
x +1

Mixed Problems

a. (g f 1)(2) + ( f 1 g f )(2)
b. ( f 1 g)(2) + ( f g)(3)

16. Find the following using the given data for

functions with inverses:


a. f(x) = 3x + 1, g(x) = 2x + 3, (f g1)(x) = ?

19. Prove that

1
x+ 2
b. f ( x) = , g( x) =
, ( g 1 f )( x) = ?
x
2 x +1

c. f 1( x) =

2 x +1 1
3x +1
, g ( x) =
, ( f 1 g1 )( x) = ?
x3
x 2

d. f 1( x) =

x 2 1
2
, g ( x) =
, ( f g)1( x) = ?
x
x2

f. g( x) =

3x +1
, ( f g)( x) = 2 x+ 3, f 1( x) = ?
x 1

g. g( x) =

2 x +1
, ( g 1 f )( x) = 2 x 1, f 1( x) = ?
x

17. Find the required values using the given data for
functions with inverses:

20. If f and g are even functions such that


f(0) = g(4) = 2, g(0) = f(4) = 4,
f(2) = g(2) = 0, find

a. f(x) = x + 1, g(x) = 3x 4, ( f

b. f(x) = x 2, g(x) = 2x + 1, ( f g1)(3) = ?


x+ 3
, g( x) = 6x +1, ( f g)1(2) = ?
c. f ( x) =
x2

2x + 3
1 2 x +5
d. f
= x, g
= 3x 2,
x +1
x+ 3
( f g )(3) = ?
178

21. Prove that if f and g are even, then f + g, f g, fg,

f / g are also even.

22. Prove that if f and g are odd, then f + g, f g are


odd and fg is even.

g)(2) = ?

( g f )(4)+( f g)(4)+( g f )(4)


.
( f g)(2)+( g f )(2)+( f g)(4)

a. f(x) = x2 2x is not one-to-one.


b. f(x) = 2x3 + 4 is one-to-one.

e. f(x) = 2x + 3, (g f )(x) = 4x 5, g(x) = ?

x
y = g(x)

23. Find the inverses of the following functions:

a. f(x) = x2 + 6x, D( f ) = (, 3]
b. f(x) = x2 8x + 5, D( f ) = [4, )

24. Prove that (g f )1 = f 1 g1.

Algebra 8

Cryptography is the art of secret writing. Its aim is to protect information by


transforming it into a coded language which unwanted eyes are unable to
use. This transformation, however, must be done in a reversible way so that
individuals intended to view the information may do so.
Some of the classical methods of cryptography are as follows (However, none
of them are used today, because they are considered either insecure or
impractical).

Simple Substitution:
When we encode the message MATH IS COOL , it becomes
NBUIAJTADPPM. Here, the rule is that every letter is substituted by the
letter that follows it. That is, A by B, B by C, space by A, etc.

Mixing:
Take a key sequence consisting of the first few natural numbers in mixed
order, for example 3, 5, 1, 2, 4. The rule is that the first letter will move to
the 3rd place, the second to 5th place, the third to 1st place, the fourth to
2nd place, the fifth to 4th place, and so on for the next five letters.
So MATH IS COOL becomes THM A CIOS OL.
Note that the space character is also treated as a letter.

Method of Vigenere:
Pair each letter with a number with its order in the alphabet as follows:
A B C D E F G H I J K L M N O P Q R S T U V W X Y Z
1 2 3 4 5 6 7 8 9 10 11 12 13 14 15 16 17 18 19 20 21 22 23 24 25 26 27
Functions

179

Our message MATH IS COOL becomes 13 1 20 8 27 9 19 27 3 15 15 12


Choose a key word, for example, FUNCTION which is 6 21 14 3 20 9 15 14.
Now, add 6 to the first letter of our encoded message, 21 to the second, 14 to the
third, and so on.
The new message will be read as 19 22 34 11 47 18 34 41 9 36 29 15.
If the number is more than 27, divide it by 27 and take the remainding
number. Finally, the message will become SVGKTRGNIIBO. Find the pair for
these numbers from the table above. The whole procedure is summarized in
the table below.
13 1 20 8 27 9 19 27 3 15 15 12
6 21 14 3 20 9 15 14 6 21 14 3

convert the message using table.


convert the keyword and write it as the
same length as the original message.

19 22 34 11 47 18 34 41 9 36 29 15

add the top two rows.

19 22 7 11 20 18 7 14 9

divide the previous row by 27 and


write the remainding number.

2 15

Note that all of these methods are functions that are defined from a limited
domain to a range. We can invent even more complicated methods by
applying a few of them together for the same message (Think about
the composition of functions!). Here it is very important to know the rules.
To decode a crypted message we just apply the rule in reverse order (Think
about the inverse of a function!).
Below is an encoded message. We first applied the Method of Vigenere with
the key word CAT (each letter is paired by its order in the alphabet as space
being the 27th letter), and then we applied the simple substitution.

GQHKTVXWFEVCSPM
Try to decode it!
What will happen if the rule is a function that has no inverse? What happens
if we assign the same number to two different letters? If a few of the rules are
applied in order, by which rule should we start the decoding procedure? Try
to develop your own method of encryption.
Algebra 8
180

Definition

quadratic function
A function f(x) is a quadratic function if
y = f(x) = ax2 + bx + c,

a0

where a, b, and c are real numbers.


For example, f(x) = 2x2 + 3x + 1, y = x2 1, y = 2x2 and f(x) = 2x x2 are all quadratic
functions. y = 3x 4 is not a quadratic function, because a = 0.
The domain of a quadratic function is the set of all real numbers.
The graph of a quadratic function f(x) = ax2 + bx + c is called a parabola.
a>0
y

a<0
y

axis of symmetry

c
The sign of a has no
effect on the size or
shape of the parabola,
the sign of a determines
whether the parabola
opens upward or
downward.

x-intercept
x1

x2

x1
x

x2
x

x-intercept
c
y-intercept

a>0

a<0

vertex

When a > 0, the parabola opens upward.

When a < 0, the parabola opens downward.

The point V in the parabolas above is called the vertex of the parabola. The vertex is the
lowest or the highest point of the parabola.
The vertical line drawn through the vertex is called the axis of symmetry. It divides the curve
into two symmetrical halves.
The points x1 and x2 are called the x-iintercepts of the parabola. They are the zeros of the
function.
The point c is called the y-iintercept of the parabola.
182

Algebra 8

A. GRAPHING y = ax2
How can we find the graph of the quadratic function y = ax2? If we have the function, we can
plot the graph by making a table of values. To find the values, we substitute different values
of x into the equation to obtain the corresponding y values. These x and y values provide the
coordinates for points which we can plot to form the shape of the graph.
Let us graph the function y = ax2.
If a > 0, we get the table of
ordered pairs opposite.
Then we plot the points

4a

4a

on a graph and draw a


parabola through them.
y
4a
3a
2a
a
-2

-1

y = ax2, a > 0

We can see that the vertex of the parabola is at the origin (0, 0), and the axis of symmetry
lies along the y-axis (the line x = 0).
If a < 0, we get a different
set of ordered pairs.

4a

4a

y
-2

-1

2a
3a
4a

Quadratic Functions

183

EXAMPLE

Sketch the graphs of the functions.


a. y = x2 , y = 2x2 and y = 1 x2
2
a.

Solution

As |a| increases, the


parabola becomes
narrower. As |a|
decreases, the parabola
becomes wider.

y = x2

y = 2x2

1 2
x
2

1
2

1
2

y=
y

ax2 x2

b. y = x2 , y = 2x2 and y = 1 x2
2

1 x2
2 x2 2x2

y
4

a>1

x
y

x2 ax2

1
-4

-3

-2

-1

y = x2

y = 2x2

1
2

1
2

0<a<1

b.

y=

ax2 +

bx

b
a

x
a>0

y=

b
a

1 2
x
2

-4

-3

-2

-1

4
x

b x
b
a
a
a<0

2x2

184

x2 1 x2
2

Algebra 8

Check Yourself 1
Graph the functions.
1. y = 3x2

2. y = 1 x2
3

4. y = 1 x2
3

3. y = 3x2

B. GRAPHING y = ax2 + bx + c
1. Vertex Point
a>0

a<0

Then ax2 + bx+ c = a(x x1 )(x x2 )

b
2a

b
2a

b
2a

b
2a

Let x1 and x2 be zeros of the polynomial ax2 + bx + c.

b
2a

b+
b
= a x
x

2a
2a

b
b+ b 2
= a x2 x
x

2a + 4a2

2a

xb+ x + xb x
b2

= a x2 +
+ 2 2
2a
4a
4a

bx b 2
= a x2 + + 2
a 4a 4a

b
2a

= a x+

2
4
a
a

= a(x h)2 + k where h =

If a quadratic function has


two known x-intercepts,
we
can
find
the
x-coordinate of the vertex
point by averaging them.
h=

x1 + x2
b
=
2
2a

and k = .
2a
4a

This is called the completed square form of the quadratic function.


COMPLETED SQUARE FORM
The completed square form of a quadratic function is f(x) = a(x h)2 + k.
The vertex of a parabola of a function in complete square form is the point (h, k).
b
4ac b 2
and k = f(h) or k =
.
2a
4a
Since (h, k) is the vertex of a parabola, then the x-coordinate of the vertex point is the equation of the axis of symmetry, x = h.

At the vertex point V(h, k), h =

Quadratic Functions

185

If a > 0, then the y-coordinate of the vertex is the minimum value of the function. If a < 0,
then the y-coordinate of the vertex is the maximum value of the function.
a>0

a<0
y

D
4a

maximum value of
the function
V(h,k)

b
2a
x

The trajectory of a shell


fired from a cannon is a
parabola. To reach the
maximum value range with
a cannon, it is shown in
calculus that the barrel of
the cannon must be set at
45.

EXAMPLE

D
4a

V(h,k)
minumum value
of the function

Find the coordinates of the vertex point of each function. Write the equation of the axis of
symmetry, and determine the maximum or minimum value of the function.
a. y = x2 2x 3

Solution

b
2a

b. y = x2 + 4x + 5

c. y = x2 + 3x + 1

a. V(h, k) is the vertex point.


b
2
1
h
2a 2
k = f(h) = 12 2 1 3 = 4
So the vertex is V(1, 4).
The equation of the axis of symmetry is x = 1.
Since a = 1 > 0, the vertex is the minimum value and k = 4 is the minimum value of
the function.

b. h =

b
4
=
=2
2a
2 (1)

k = f(h) = 22 + 4 2 + 5 = 9
So the vertex is V(2, 9).
The equation of the axis of symmetry is x = 2.
Since a = 1 < 0, the vertex is the maximum value and k = 9 is the maximum value of
the function.
186

Algebra 8

c. h =

b
3
3
=
=
2a
2 1
2
2

5
3
3
k = f (h) = + 3 + 1 =
4
2
2
3 5
So the vertex point is V , .
2 4

3
The equation of the axis of symmetry is x = .
2

Since a = 1 > 0, the vertex is the minumum value and y =


the function.
EXAMPLE

Solution

5
is the minimum value of
4

The line 3x + 1 = 0 is the axis of symmetry of the parabola y = 2x2 + mx 1.


Find the maximum value of the function.
3x + 1 = 0 ; x =

1
is the equation of axis of symmetry.
3

1
b
m
4
h= =
=
; m=
3
2a
2 (2)
3

Since a = 2 < 0, the vertex is the maximum value.


2

4 1
7
1
k = f (h) = 2 1 =
3 3
9
3

So k =

EXAMPLE

Solution

7
is the maximum value.
9

The minimum value of the function y = x2 8x + n is 4. What is the value of n?


Given k = 4 and h =
k = f (h) ;

b
8
=
= 4 ; h = 4.
2a
2 1

4 = 42 8 4 + n ;

n = 12

Check Yourself 2
Find the vertex of each parabola.
1. y = x2 10x + 20

2. y = 1 x2 + x 8
2

3. y = 4 x2

4. y = 4x + x2

Answers
1. (5, 5) 2. (1, 8.5)
Quadratic Functions

3. (0, 4)

4. (2, 4)
187

2. Intercepts
Consider the function y = f(x) = ax2 + bx + c,

a 0.

If x = 0, then y = c. The point (0, c) is called the y-intercept of the graph. We mean that the
parabola intersects the y-axis at this point.
If y = 0, then ax2 + bx + c = 0. Let x1 and x2 be roots of this equation. The points (x1, 0)
and (x2, 0) are called the x-intercepts of the graph. We mean that the parabola intersects the
x-axis at these points. There are three possibilities for the x-intercepts.
1. If > 0, the parabola intersects the x-axis at two distinct points.
2. If = 0, the parabola is tangent to the x-axis.
3. If < 0, the parabola does not intersect the x-axis.
D<0
D=0

a<0
D>0

x
D>0

a>0

D=0
D<0

EXAMPLE

Solution

Find the x- and y-intercepts of each function.


a. y = x2 9

b. y = 4x2 + 5x

d. y = x2 + 3x + 7

e. y = 4x2 + 20x + 25

c. y = 3x2 + 4x 2

a. x = 0 ; y = 9
So (0, 9) is the y-intercept.
y = 0 ; x2 9 = 0
x1 = 3, x2 = 3
So (3, 0) and (3, 0) are the x-intercepts.
b. x = 0 ; y = 0
So (0, 0) is the y-intercept.
y = 0 ; 4x2 + 5x = 0 ;

x1 = 0 , x2 =

5
4

So (0, 0) and , 0 are the x-intercepts.


4

188

Algebra 8

c. x = 0 ; y = 2
So (0, 2) is the y-intercept.
y = 0 ; 3x2 + 4x 2 = 0
= 42 4 3 (2) = 40
x1 =

4 2 10 2 10
4 + 2 10
2 + 10
, x2 =
=
=
6
3
6
3

So (

2 10
2 + 10
, 0) and (
, 0) are the x-intercepts.
3
3

d. x = 0 ; y = 7
So (0, 7) is the y-intercept.
y = 0 ; x2 + 3x + 7 = 0
= 32 4 1 7 = 19 < 0
So the parabola does not cut the x-axis, i.e. there are no x-intercepts.
e. x = 0 ; y = 25
So (0, 25) is the y-intercept.
y = 0 ; 4x2 + 20x + 25 = 0
= 202 4 4 25 = 0
x1 = x2 =

20
5
=
8
2

= 202 4 4 25 = 0
5
So the parabola is tangent to the x-axis at the point ( , 0).
2
EXAMPLE

The function y = x2 (m + 2)x + 5 + m is given. Find the value(s) of m for which the
parabola
a. does not cut the x-axis.

b. is tangent to the x-axis.

c. intersects the x-axis at two distinct points.


Solution

= (m + 2)2 4(5 + m)
= m2 16
= m2 16 = 0 ;

m = 4

Let us consider the sign of .


m
m2 16

Quadratic Functions

+
189

a. If m (4, 4), the parabola does not cut the x-axis.


b. If m = 4, the parabola is tangent to the x-axis.
c. If m (, 4) (4, ), the parabola cuts the x-axis at two distinct points.

Check Yourself 3
Find the x- and y-intercepts of each parabola.
1. y = x2 + 3x 4

2. y = 3x2 6x + 3

3. y = 3x x2

Answers
1. no x-intercepts, (0, 4)

2. (1, 0), (0, 3)

3. (0, 0), (3, 0)

3. Sketching a Graph
To sketch the parabola y = ax2 + bx + c, follow the steps.
1. Check a.
When a > 0, the graph has a minimum point and the parabola opens upward.
When a < 0, the graph has a maximum point and the parabola opens downward.
2. Find the coordinates of the vertex point, V(h, k).
3. Find the x- and y-intercepts.
4. Sketch the graph of the parabola.

EXAMPLE

Solution

Graph the functions.


a. y = x2 3x 10

b. y = 2x2 8x

d. y = 3x2 + 12

e. y = x2 22x + 2

c. y = x2 + 4x + 5

a. a = 1 > 0, so the parabola opens upward.


h=

3
b
= ,
2a 2

k = f (h) = (

3
2

-2

3 2
3
49
) 3 10 =
;
2
2
4

3 49
) is the vertex point.
V( ,
2
4

x = 0 ; y = 10, (0, 10) is the y-intercept.

f(x)=x2 3x 10
-10
49
4

y = 0 ; x2 3x 10 = 0 ; x = 2, x = 5 ;
(2, 0) and (5, 0) are the x-intercepts.
190

Algebra 8

b. a = 2 < 0, so the parabola opens downward.


h=

8
= 2,
2 (2)

k = f (h) = 2(2) 8 (2) = 8 ;


2

V(2, 8)

y
8

f(x)=2x2 8x

x = 0 ; y = 0, (0, 0) is the y-intercept.


y = 0 ; 2x2 8x = 0 ;
x = 0, x = 4 ; (4, 0) and (0, 0) are the x-intercepts.

-4

c. a = 1 > 0, so the parabola opens upward.


h=

4
= 2,
2 1

k = f (h) = (2)2 + 4 (2) 2 + 5 = 1 ;

-2

f(x)=x2+4x+5

V(2, 1)

x = 0 ; y = 5, (0, 5) is the y-intercept.


2

y = 0 ; x + 4x + 5 = 0 ;

-2

= 42 4 1 5 = 4 < 0 ; the parabola does not cut the x-axis.


d. a = 3 < 0, so the parabola opens downward.
h=

0
= 0,
2( 3)

k = f (h) = 3(0)2 +12 = 12 ;

V 12

V(0, 12)
f(x)=3x2+12

x = 0 ; y = 12, (0, 12) is the y-intercept.


y = 0 ; 3x2 + 12 = 0 ;
x = 2 ; (2, 0) and (2, 0) are the x-intercepts.

-2

e. a = 1 > 0, so the parabola opens upward.


h=

y
f(x)=x2 22x+2

2 2
= 2
2 1

k = f (h) = ( 2 )2 2 2 2 + 2 = 0 ;

V( 2, 0)

x = 0 ; y = 2, (0, 2) is the y-intercept.

2
V
2

y = 0 ; x 22 + 2 = 0 ;
x1 = x2 = 2 ; the parabola is tangent to the x-axis at the point (2, 0).
Quadratic Functions

191

EXAMPLE

Solution

Sketch the graph of the function y = x2 2x + 8 for |x| 3. Find the range of the function.
a = 1 < 0, so the parabola opens downward.
h=

2
= 1,
2 (1)

9
8

k = f (h) = (1)2 2 (1) + 8 = 9 ;

V(1, 9)

x = 0 ; y = 8, (0, 8) is the y-intercept.


y = 0 ; x2 2x + 8 = 0 ;
x1 = 4, x2 = 2 ; (4, 0) and (2, 0) are the x-intercepts.
|x| 3 ; 3 x 3
x = 3 ; y = 9 + 6 + 8 = 5, so (3, 5) is on the parabola.
x = 3 ; y = 9 6 + 8 = 7, so (3, 7) is on the parabola.
So the range of the function is [7, 9].

EXAMPLE

Solution

-4
2

-3 -1

-7

Find the values of p for which px2 + 4x + p is greater than zero for all real values of x.

px2 + 4x + p > 0 for all real numbers x.

y = px2+ 4x+p

This is possible only if p > 0 and < 0.


(1)

p>0
2

4 4p < 0 (2)

(1) p > 0
(2) 42 4p2 < 0 ; 42 4p2 = 0 ; p = 2
p

(1)

(2)

system

So p (2, ).

Check Yourself 4
Sketch the graph of each function.
1. y = x2 5x + 4
192

2. y = x2 + 4x 3

3. y = 4x2 20x + 25

4. y = 3x2 2x
Algebra 8

Answers
1. y

2.

3.

4.

25

2/3

-3
4 x

5/2

4. Shifting Graphs
Let y = f(x) be a function.
Vertical Shifting
y = f (x ) + k

k > 0 shift the graph of y = f(x) k units upward. (see Figure 1)

k < 0 shift the graph of y = f(x) |k| units downward. (see Figure 1)

Horizontal Shifting
y = f (x+ h)

h > 0 shift the graph of y = f(x) h units to the left. (see Figure 2)

h < 0 shift the graph of y = f(x) |h| units to the right. (see Figure 2)

Reflection
y = f(x)

Reflect the graph of y = f(x) in the x-axis. (see Figure 3)

Horizontal and Vertical Shifting


y = f(x + h) + k Shift the graph of y = f(x) |k| units upward or downward and |h| units
to the left or to the right.
k>0

x2+k

h>0

x2 (x h)2

(x+h)2

x2
k

x2 k

-h

x2

-k
x2

Figure 1
EXAMPLE

10

Quadratic Functions

Figure 2

Figure 3

Sketch the graph of the each function using the shifting method.
a. y = x2 + 3

b. y = x2 3

c. y = (x 3)2

e. y = (x 4)2

f. y = (x + 2)2 3

g. y = (x 2)2 + 1

d. y = (x + 3)2

193

Solution

a.

b.

x2+3

c.

x2

d.

(x+3)2

x2 (x 3)2

x2

y
x2

x2 3

3
x

-3

-3

e.

f.

g.

(x+2)2
x2

x2

x2

(x+2)2 3

-2

-3

(x 4)2

x2

(x 2)2+1

Check Yourself 5
Sketch the graph of the each function by shifting.
1. y = x2 + 4
Answers
1.
y

2. y = (x + 1)2
2.

x2+4

3. y = (x 2)2 2
3.

y
-1

x2

(x+1)2

x2

4.

(x2)2

y
1

-1

x2

4. y = (x + 2)2 + 1

(x2)22

4
2
-2

-2
(x+2)2+1

-3

-4

x
(x+2)2

x2

5. Parabolas with Absolute Value (Optional)


EXAMPLE

194

11

Sketch the graph of each function.


a. y = |x2 3x + 2|

b. y = x2 + 2|x| + 3

c. y = x|x + 2|

d. y = |2x2 12x + 16| + 2


Algebra 8

Solution

a. First graph the function y = x2 3x + 2.

Then, take above the part of the graph which is below

the x axis, symmetric with respect to x-axis.


a = 1 > 0, so the parabola opens upward.
h=

3
3
= ,
2 1 2

f(x)=|x2 3x+2|
1
1/4
3/2

-1/4

3
1
3
k = f (h) = 3 + 2 = ;
2
4
2
1
3
V , is the vertex point.
4
2

x = 0 ; y = 2, (0, 2) is the y-intercept


y = 0 ; x2 3x + 2 = 0 ;
x = 1, x = 2 ; (1, 0) and (2, 0) are the x-intercepts.
b. y = x2 + 2|x| + 3
Case
1
____________________________________________________________________
If x 0, |x| = x ; y = x2 + 2x + 3
a = 1 < 0, so the parabola opens downward.
h=

2
= 1,
2 (1)

k = f (1) = 12 + 2 1+ 3 = 4 ;

V(1, 4) is the vertex point.

x = 0 ; y = 3, (0, 3) is the y-intercept.


y = 0 ; x2 + 2x + 3 = 0 ;
x = 1, x = 3 ; (1, 0) and (3, 0) are the x-intercepts.
Case
2
____________________________________________________________________
If x < 0, |x| = x ; y = x2 2x + 3

y
4

a = 1 < 0, so the parabola opens downward.

2
h=
= 1,
2 (1)
k = f (1) = (1) 2 2 (1) + 3 = 4 ;

f(x)=x2+2|x|+3

-3

-1

V(1, 4) is the vertex point.

x = 0 ; y = 3, (0, 3) is the y-intercept.


y = 0 ; x2 2x + 3 = 0 ; x = 3, x = 1 ; (3, 0) and (1, 0) are the x-intercepts.
Quadratic Functions

195

c. y = x|x + 2|
Case
1
________________________________________________________________________
If x + 2 0, x 2, |x + 2| = x + 2 ; y = x2 + 2x
a = 1 > 0, so the parabola opens upward.
h=

2
= 1, k = f (1) = (1) 2 + 2 (1) = 1 ;
2 1

V(1, 1)is the vertex point.

x = 0 ; y = 0, (0, 0) is the y-intercept.


y = 0 ; x2 + 2x = 0 ; x = 2, x = 0 ; (2, 0) and (0, 0) are the x-intercepts.
Case
2
________________________________________________________________________
If x + 2 < 0 ; x < 2, |x + 2| = (x + 2) ;

y = x 2x
2

a = 1 > 0, so the parabola opens upward.

f(x)=x|x+2|

2
h=
= 1,
2 (1)

-3

-2

-1

-1

k = f (1) (1) 2 2 (1) 1 ;

V(1, 1) is the vertex point.


x = 0 ; y = 0, (0, 0) is the y-intercept.
y = 0 ; x2 2x = 0 ; x = 2, x = 0 ; (2, 0) and (0, 0) are the x-intercepts.
d. y = |2x2 12x + 16| + 2

y
2

First graph the function y = |2x 12x + 16|.


Then shift this graph 2 units up.

18

f(x)=|2x2 12x+16|+2

16

a = 2 > 0, so the parabola opens upward.


h=

y=|2x2 12x+16|

12
= 3,
22

k = f (h) = 2(3)2 12 3 +16 = 2 ;


V(3, 2)is the vertex point.

x = 0 ; y = 16, (0, 16) is the y-intercept.

y = 0 ; 2x2 12x + 16 = 0 ;

x = 2, x = 4 ;
(2, 0) and (4, 0) are the x-intercepts.

196

2 3 4
-2

Algebra 8

Check Yourself 6
Sketch the graph of each function.
1. y = |x2 6x + 5|

2.

y = x|x| + 4

Answers
1. y

2.

5
4

4
2
1

C. EQUATION OF A PARABOLA
We have learned how to construct a parabola if we are given its equation. But if we are given
the graph of a parabola, how can we write its equation?
There are three different approaches to finding the equation of a parabola. The approach we
use depends on the information we know.
1. If we know the vertex point of the parabola, V(h, k) then we use the formula y = a(x h)2 + k
to write the equation of the parabola. We need to know another point on the parabola to
write the equation.
2. If we know the x-intercepts of the parabola then we use the formula y = a(x x1)(x x2)
to write the equation of the parabola. We need to know another point on the parabola to write
the equation.
3. If we know any three points on the parabola, then we use the formula y = ax2 + bx + c
to write the equation of the parabola.
EXAMPLE

12

Write the equation of each parabola.


a.

b.

y
2

x
-3

Quadratic Functions

y=g(x)

y=f(x)
2

c.

y=h(x)

2
1

197

Solution

a. The vertex point of the parabola is V(2, 2), so y = a(x 2)2 + 2.


Also, the parabola passes through the origin (0, 0), so this point satisfies the equation of
the parabola.
0 = a(0 2)2 + 2 ; 0 = 4 a+ 2 ;

a=

1
2

1
y = (x 2)2 + 2
2
1
y = x2 + 2 x
2

This is the equation of the parabola.


b. Let y = ax2 + bx + c.
Since the points (0, 3), (3, 3) and (1, 4) are on the graph of the parabola, these points
satisfy the equation of the parabola.
(0, 3), 3 = c

b b2 4ac
2a

(3, 3), 3 = 9 a 3 b+ 3 ;
(1, 4), 4 = a+ b+ 3 ;

9a 3b = 0
a+ b = 1

9a 3b = 0 (1)

(2)
a+ b = 1
From (1) and (2),

1
3
a= , b= .
4
4

So the equation is y =

1 2 3
x + x+3 .
4
4

c. Let y = a(x 3)(x 4)


(0, 2) is on the parabola.
2 = a(0 3)(0 4) ;

a=

1
6

1
y = (x 3)( x 4)
6
So the equation is y =
198

1 2 7
x x+2 .
6
6
Algebra 8

EXAMPLE

13

x1 and x2 are the roots of the parabola below. Find the value of x1 + x2.
y
y=g(x)

-6 x1

x2 2

-2

Solution

g(2) = g(6) = 3

y
B

y=g(x)

AB = BC, so the x-coordinate of the vertex point is


h=

x2 2

-6 x1

-2

6 + 2
= 2.
2

h =

x + x2
b
; 1
= 2 ;
2a
2

x1 + x2 = 4.

EXAMPLE

14

Solution

A parabolic concrete bridge support needs


to pass through the points (50, 0),
(0, 30), and (50, 0). Write the equation of
the parabola formed by the bridge.

Let y = ax2 + bx + c. Since the points (50, 0), (0, 30), and (50, 0) are on the graph of the
parabola, these points satisfy the equation of the parabola.
(0, 30), c = 30
(50, 0), 0 = 2500a 50b + 30 ;

250a 5b = 3

(1)

(50, 0), 0 = 2500a + 50b + 30 ;

250a + 5b = 3

(2)

From (1) and (2),


a=

3
, b = 0.
250

So the equation is y =

Quadratic Functions

3 2
x +30 .
250
199

EXAMPLE

15

The shape of the famous Gateway to the West arch, which is


the spirit of the City of St. Lois can be modeled by a parabola.
The equation for the parabola is y =

1 2 21
x + x.
150
5

a. Sketch the graph of the arch's equation on a coordinate axis.


b. What is the distance from one side of the arch to the other?
Solution

a. a =

1
< 0, so the parabola opens downward.
150
21
5

h=

2 (

1
)
150

y
661.5

= 315,

1
21
315 2 + 315
150
5
= 661 .5 V(315, 661.5)

k = f (315) =

630
315

x = 0 ; y = 0 (0, 0) is the y-intercept.


1 2 21
x + x=0 ;
150
5
x = 0, x = 630 ; (0, 0) and (630, 0) are the x-intercepts.
y=0 ;

b. From the x-intercepts, we can see that the distance along the bridge is 630 m.
EXAMPLE

16

Solution

A farmer has 120 m of fencing. He wants to put


a fence around three sides of a rectangular plot of
land, with the side of a barn forming the fourth
side. Find the maximum area the farmer can
enclose. What dimensions give this area?
Let x represent the width of the plot. Then, since
there are 120 m of fencing,
x + x + length = 120 ; length = 120 2x.
The area of the plot is A = (120 2x) x = 120x 2x2.
We can see that a < 0, so the graph of the function of the area has its maximum value at the
vertex point.
To make the area as large as possible, let us therefore find the vertex of the graph of the function
A = 2x2 + 120x.
b
120
=
= 30. This is the width of the plot at its maximum
The x-coordinate of the vertex is
2a
4
area. Now we have the solution: A = 2 302 + 120 30 = 1800 m2 and the dimensions of
the plot are 30 m by (120 2 30)m, i.e. 30 m by 60 m.

200

Algebra 8

EXAMPLE

17

Solution

Find the maximum vertical distance d between the parabola and the line in the figure.
The distance between the vertex point of the parabola
and the midpoint of the AB line segment in the figure
will be the maximum distance.

b
4
=
=1
So h =
2 a 4

k = f (1) = 5 ,

5
4

f(x)=x 2
d

V (1, 5).

Let us find the coordinates of point A and point B.

-2

-1

2 x + 4x+ 3 = x 2

-1

-2

2 x2 3x 5 = 0
A

5
(2 x 5)(x+1) = 0 ; x = , x = 1
2

-3
-4

5 1
A(1, 3) and B( , )
2 2

5
1
1+
3 +
2,
2 ) = ( 3 , 5 )
M(
2
2
4 4
So d = (

f(x)=2x2+4x+3

B
( 5 , 1)
2 2

(1, 3)

3
5
1 625
626
626
1)2 +(
5) 2 =
+
=
=
.
4
4
16 16
16
4

Check Yourself 7
Write the equation of each parabola.
1.

2.

3.
3

-1

-3

-2

-4

-1

Answers
1. y =

1 2 2
x x1
3
3

2. y =

1 2 3
x + x3
4
4

3. y =

1 2
x x+ 2
4

The things of this world cannot be made known without a knowledge of mathematics.

Quadratic Functions

201

P ARABOLIC R

EFLECTORS

Parabolic curves are used in the design of lighting systems, telescopes, and radar antennas, mainly because of the
reflective property you can see in the figures below.
axis

axis

Figure a . A reflecting telescope:


light rays parallel to the
axis are concentrated at
the focus.

Figure b . A parabolic flashlight:


a light source at the focus
sends out beams of light
parallel to the axis.

Figure a shows the application of the reflective property of a parabola to create a reflecting telescope. The eyepiece
of the telescope is placed at the focus F of a parabolic mirror. Light enters the telescope in rays that are parallel to
the axis of the parabola. We know from physics that when light is reflected, the angle of incidence equals the angle
of reflection. Hence, the parallel rays of light strike the parabolic mirror so that they all reflect through the focus,
which means that all the parallel rays are concentrated at the eyepiece. This maximizes the light-gathering ability of
the mirror.
Flashlights and automobile headlights (see Figure b) simply reverse this process. A light source is placed at the focus
of a parabolic mirror. The light rays strike the mirror with an angle of incidence equal to the angle of reflection, and
each ray is reflected along a path parallel to the axis. As a result, the light emits a light beam of parallel rays.

Radar utilizes both of these properties. First, a pulse is transmitted from the
focus to a parabolic surface. As with a reflecting telescope, parallel pulses are
transmitted in this way. The reflected pulses then strike the parabolic surface and
are sent back to be received at the focus.

202

Algebra 8

5 .1

EXERCISES

6. The equation of the axis of symmetry of the

A. Graphing y = ax2
1. Graph each set of functions in the same plane.
a. y = 2x2, y 4x2 = 0, 2x2 = 3y

parabola y = 2x2 + (m + 1)x 4 is 4x 3 = 0.


Find the minimum value of the function.

b. y = 3x2, y + 4x2 = 0, 3x2 = 2y


c. y = 3x2, 3y = x2, 2y + x2 = 0, y + 2x2 = 0

7.
2. Determine whether each point lies on the graph

4
is the minimum value of the function
3

y = 3x2 + 2x n. Find the value of n.

of the function 4y + 5x2 = 0.


45

a. 3,

4
b. , 1
5

c. (2, 5)

d. (4, 10)

8. The vertex point of the parabola


y = 3x2 2(2k + 5)x + 4 is on the y-axis.

3. If the parabola y = px2 passes through the following

Find k.

points, find p.
a. (3, 3)

4
b. 2,
5

c. (3, 7)

d. , 2
5

4. Graph the function y = x if x 4.


2

9. Find the x- and and y-intercepts of each function.


a. y = x2 + 4x

b. y = (x 3)2

c. y = 8(x + 7)2 + 4

d. y = x2 8

e. y = 2x2 + 5x + 3

f. y = x2 4x 5

g. y = 16x2 + 24x + 9 h. y =

B. Graphing y = ax2 + bx + c

2 2
x 4
3

5. Find the coordinates of the vertex point of each


function. Write the equation of the axis of symmetry
and determine the maximum or minimum value
of the function.
a. y =

3 2
x
4

c. y = 3x2 + 2

d. y = x2 1

e. y = 2x2 + 5x

f. y = 0, 7x2 + 0,8x

g. y = 3x2 4x + 3

h. y = x2 + x + 1

i. y = 3(x + 1)(x 4) j. y = 2(x + 4)2


k. y = (x + 3) 5
Quadratic Functions

Find the values of m for which the parabola


a. does not cut the x-axis.

b. y = 3x2

10. The function y = x2 + (m 2)x + m 3 is given.

l. y = (x 1) + 3

b. is tangent to the x-axis.


c. cuts the x axis at two distinct points.

11. The parabola y = mx2 (m + 1)x + 2m 1 passes


through the point A(2, 9). Find the value of m.
203

12. Sketch the graph of each function.

22. Sketch the graph of each function by shifting.

a. y = 3x2 9

b. y = 2x2 + 5

a. y = x2 + 4

b. y = x2 2

c. y = 3(x + 4)2

d. y = 2(x + 7)2

c. y = x2 2

d. y = (x + 5)2

e. y = x2 2x 3

f. y = x2 4x

e. y = (x 4)2

f. y = (x 3)2

g. y = x2 4x + 1

h. y = x2 5

g. y = (x + 1)2 2

h. y = (x 2)2 + 3

i. y = (x 3)2 + 4

j. y = (x + 3)(4 x)

13. Sketch the graph of the function


y = x2 2x 8 if 1 < x < 5.

23. Sketch the graph of each function.


a. y = |x2 4x + 3|

b. y = 2x2 + 4|x| + 1

c. y = x|x 2|

d. y = |x2 6x + 8| 1

14. Sketch the graph of the function


y = 2x2 + 4x + 1 if |x 1| < 2.

C. Equation of a Parabola
15. Find the minimum and the maximum values of
the function y = x + 4x + 7 if x [4, 1].
2

24. Write the equation of each parabola.


a.

b.

3
2

16. Find the minimum or the maximum value of the

function if m + n = 16.

6 x

17. Find the minimum or the maximum value of the


function y = m2 + n if m 2 = n.

c.

d.

y
4
3

18. The parabola y = 2ax + bx 3b passes through


2

the points (1, 3) and (2, 5). Find the values of


a and b.

19. Write the equation of the parabola which is


symmetric to y = 3(x + 1)2 with respect to the
line x = 2.

20. Given f(x) = 3(x 1)2 + 2, show that f(x) f(1).

-3

e.

f.

y
3

-1

x
-3

-3

21. f(x) = x2 + 3mx + 4m and f(x) 0 are given.


Find m.
204

-8

Algebra 8

25. Write the equation of the parabola through each

30. The figure shows the graph

set of points.

of the function

a. A(1, 0),

B(0, 4),

C(1, 2)

y = mx2 7mx + 5.

b. A(0, 4),

B(2, 0),

C(4, 4)

If |AB| = 3, find m.

26. Write the equation of the parabola through each


vertex point V and point A.
a. V(0, 4), A(3, 5)

b. V(2, 0), A(4, 4)

31. Find the area of the

1 15
c. V(1, 4), A(2, 5) d. V , , A(0, 4)
4
2

trapezoid OABC in
the figure.

5
2

y=2x2+6x
C

B
A

27. In the figure,

3 |OB| = |AO|.
Find c.

y=x2 4x c
A

B
x

32. Find the area of the

|OB| = 5 |AO| and


V is the vertex point
of the parabola.
Find k.

B
O

29. The figure shows the

graph of the function


y = ax2 + bx + c.
Find the value of

33. Find the area

square OABC in the figure.


24

28. In the figure,

-5

of the
rectangle
ABCD in
the figure.

C
y=x2 6x+5

a + b + c.
Quadratic Functions

205

39. The figure shows the

Mixed Problems

34. x1 and x2 are the x-intercepts of the function

y = mx2 4x 2m 2. If x1 x1 < 0, find the


possible values of m.

graph of the function


y = x2 + bx + 1.
Find the value of
a + b + c + d.

y=x2+bx+1

5
c

35. The perimeter of a rectangle is 32 cm. Find its

maximum possible area.

40. In the figure,


36. The function y = 5x 3x + k is tangent to the
2

line y = 2. Find k.

the area of the


triangle ABC is
6 cm2.
Find k.

y=k(x2+5x+4)
C
A

B O

37. 3 is the minimum value of the function

y = mx2 2mx + 2m + 1. Find m.

38. The figure shows the

graph of the function

41. In the figure, the

y = x2 + bx + c.
of the point A.

2
A

206

maximum value of the


49
function is
.
4
Find the area of the

Find the coordinates


x

y
C
A
-2

B
O

triangle ABC.
Algebra 8

An equation is a statement that says two expressions are equal. For example, a = b is an
equation. An inequality is a statement that says two expressions may or may not be equal. For
example, a > b, a b, and a < b are all inequalities. An equation usually has a finite number of
solutions, but an inequality may have an infinite number of solutions.
We can show the solutions of an inequality as an interval and as a graph. Look at the following
examples of intervals and their graphs.
Interval

Inequality

Graph

the interval (, a)

x<a

the interval (, a]

xa

the interval (a, )

x>a

the interval [a, )

xa

For any two real numbers


a and b, exactly one of
the following is true:

the open interval (a, b)

a<x<b

a < b, a = b or a > b.

the closed interval [a, b]

axb

the half-open interval [a, b)

ax<b

the half-open interval (a, b]

a<xb

the interval (, )

Trichotomy Property:

Notice in the table that an open circle on a graph shows an open interval, i.e. a line segment
with an open endpoint (a point which is not included in the segment). A filled circle shows
a closed endpoint (a point which is included in the segment).

EXAMPLE

Write each inequality using interval notation and show it as a graph.


a. 1 x < 5

Solution

c. (3, 4)

d. x 9

c. 3 < x < 4
1

a. The interval notation is [1, 5) and the graph is


b. (7, )

208

b. x > 7

d. (, 9]

.
9

Algebra 8

linear inequality

Definition

A linear inequality is an inequality that can be written in one of the forms


ax + b > 0,
ax + b < 0
ax + b 0,
ax + b 0
for the real numbers a and b, a 0.
If both sides of an inequality are multiplied or divided by a negative number, the direction of
the resulting inequality must be reversed.

Property

For example, if we multiply both sides of the inequality a < b by 2, we obtain 2a > 2b.
The order of the inequality is reversed.
EXAMPLE

Solution

Solve the inequality 2x + 3 5x and graph this solution.


2x + 3 5x
3x 3
1x

Therefore, x [1, ), or

EXAMPLE

Solution

Solve the inequality 2 < 3x 1 8 and graph this solution.


2 < 3x 1 8
3 < 3x 9
1<x3
Therefore, x (1, 3], or

For k > 0, the following statements are true.

Property

1. |x| k means k x k.
2. |x| > k means x < k or x > k.
EXAMPLE

Solution

Solve the inequality |2x 1| < 5 and graph this solution.


|2x 1| < 5
5 < 2x 1 < 5
4 < 2x < 6
2 < x < 3
Therefore, x (2, 3), or

Quadratic Inequalities

.
209

Check Yourself 1
Solve the inequalities.
1. x + 7 > 2x + 1
2. 11 x+ 3 17
4
3. |2x 1| + 2 7
Answers
1. x < 2 2. 47 x 65 3. 2 x 3
FIND THE MISTAKE!
a>4
4a > 16
4a a2 > 16 a2
a(4 a) > (4 a)(4 + a)
a>4+a
0>4
Can you find the mistake in this working?

Sign Chart
In general, to solve a linear inequality such as ax + b > 0 or ax + b 0 we need to know the
sign of the polynomial ax + b, a 0.
Look at the steps.
First we find the zero of the polynomial:
b
ax+ b = 0 ; x = .
a

Then we construct a sign chart.


x
ax+b

x0=

ax+b has sign opposite to a

b
a

ax+b has the same sign as a

This sign chart shows us:


b

if x , , the sign of the polynomial is opposite to the sign of a,


a

if x , , the sign of the polynomial is the same as the sign of a.


a

210

Algebra 8

EXAMPLE

Solution

Solve the inequality 3x 2 0.


2
Find the zero: 3 x 2 = 0 ; x = .
3
Draw the sign chart:
x

x0=

2
3

3x 2

If x , , then 3 x2 is negative.
3

If x , , then 3 x2 is positive.
3

Since 3 x 2 0, x is in the interval , , i. e. x , .


3
3

EXAMPLE

Solution

Solve the inequality 6 2x 0 by using a sign chart.


6 2x = 0 ; x = 3
x
6 2x

Since 6 2x 0, the sign is positive. So x (, 3].

Check Yourself 2
Solve the inequalities by using a sign chart.
x+ 14 x 12
1. 5 2x < 0
2.

3
6
8
Answers
5
1. x >
2. x 20 3. x 10
2

3. 6x + (x 2)(x + 2) (x + 4)2

Mathematics, of course, is not the only cornerstone of opportunity in todays


world. Reading is even more fundamental as a basis for learning and for life. What
is different today is the great increase in the importance of mathematics to so
many areas of education, citizenship, and careers.

Quadratic Inequalities

211

A rectangular room has dimensions 30 m 12 m 12 m. A spider is in the horizontal center of one end wall,
one unit away from the ceiling. A fly is in the horizontal center of the opposite wall, one unit away from the floor.
What is the shortest distance the spider needs to travel (without leaving a surface) to get to the fly (which
remains stationary)?

EXERCISES

6 .1

1. Write each inequality using interval notation.


a. x > 2

b. x 6

c. x < 7

d. x 4

e. 2 x < 4

f. 1 < x 0

g. 5 < x <

1
2

h. 0 ,5 x

a. 2x + 3 < 4

3
2

2. Determine the sign of each polynomial.


a. 4x + 1

b. 3x 5

x 7
c. +
2 3

d. 3x 6

212

3. Solve and graph the inequalities.

b.

x x x x
+
2 3 4 5

c.

3x
x
+ 5 < 1+
2
2

d.

x+ 3 x 2 2x

4
2
3

e. (3x + 1)2 (x + 2)(4x 1) > 5(x 1)2 + 6x


f. (a + 3)x 5 1

(a > 3)

g. (a4 + 4)x + 3 > 0 (a )


Algebra 8

Definition

quadratic inequality
A quadratic inequality is an inequality that can be written in one of the forms
ax2 + bx + c > 0,

ax2 + bx + c < 0,

ax2 + bx + c 0,

ax2 + bx + c 0,

for the real numbers a, b, and c, a 0.

We will never
be equals.

We have seen how to solve linear inequalities such as


2(x + 3) + 3 5(x 1).
But how do we solve quadratic inequalities such as
x2 5x 6?
First we write the inequality in standard form, leaving only zero on the right side:
x2 5x + 6 0.

In this example, we are looking for values of x that will make the quadratic on the left side
greater than or equal to zero.

Note
If and are used in the inequality, then remember that the zeros of the polynomial are
included in the solution set.
First, let us find the zeros of the polynomial x2 5x + 6:
(x 2)(x 3) = 0
positivepositive=positive
positivenegative=negative
negativenegative=positive

x = 2 or x = 3.
Then we construct a sign chart for each linear factor of the polynomial, and their product.
x

x2

x3

(x 2)(x 3)

Since x2 5x + 6 0, we need to take the positive intervals.


Therefore, the solution is the union of intervals (, 2] and [3, ), i.e. x (, 2] [3, ).
Quadratic Inequalities

213

We can also construct the sign chart in one step. The zeros of the polynomial divide the real
number line into three intervals, (, 2], [2, 3] and [3, ). We know that the polynomial has
constant sign in each of these three intervals. If we select a test number in each interval and
evaluate the polynomial at that number, then the sign of the polynomial at this test number
must be the sign for the whole interval.
Let us try testing each interval in our problem. Choose a number from each interval, and
substitute for x in the original inequality. For example, we could choose the numbers 1, 2.5,
and 4.
Test number

2.5

Value of the polynomial

0.25

Sign of the polynomial

With this information, we can draw the sign chart.


x
x2

5x + 6

This is the same as the last line of the previous chart.


We can also use the discriminant of a quadratic to help complete the sign chart. We have
already seen that the discriminant of a quadratic equation ax2 + bx + c = 0 tells us it the
equation has two real roots ( > 0), one double root ( = 0), or no real solutions ( < 0).
If < 0, then the polynomial ax2 + bx + c always has the same sign as a.
x

ax2 + bx + c

same sign as a

If = 0, then of the polynomial ax2 + bx + c has the same sign as a but we must consider
the zero of the polynomial.
x

x1=x2=

ax2 + bx + c

b
2a

same sign as a

same sign as a

If > 0, the polynomial ax2 + bx + c has the opposite sign to a between the zeros of the
polynomial and the same sign as a in other intervals.
x
ax2 + bx + c

214

x2

x1
same sign as a

opposite sign to a

same sign as a

Algebra 8

EXAMPLE

Solution

Solve the inequality x2 4 < 5x.


x2 4 < 5x
x2 + 5x 4 < 0
(x + 4)(x 1) = 0
x = 4 or x = 1
x

x2 + 5x 4

Therefore, x (, 1) (4, ).
EXAMPLE

Solve the inequalities.


a. 3x + 4 x2

Solution

c. 9x2 12x + 4 0

b. x(x + 2) > 35

d. x2

2x 1
<0
4

a. 3x + 4 x2 ; x2 + 3x + 4 0
= 9 + 16 = 25 ; x1 = 1, x2 = 4
x

x2 + 3x + 4

Plot the roots in the sign


chart line in ascending
order.

So x [1, 4].
b. 35 < x(x + 2) ; x2 2x + 35 < 0
= 4 + 140 = 144 ; x1 = 7, x2 = 5
x

x2 + 3x + 4

So x (, 7) (5, ).
c. 9x2 12x + 4 0
= 144 144 = 0 ; x1 = x2 =

12 2
=
18 3
2
3

x
9x2 12x + 4

2
So the only solution is x = .
3
Quadratic Inequalities

215

c. x2

2x 1
< 0 ; 4x2 2x + 1 < 0
4

= 4 16 = 12 < 0.
x
4x2

2x + 1

4x2 2x + 1 is always positive, so there is no solution.

EXAMPLE

Consider the equation x2 2(m + 1)x + 1 = 0. For which values of m does the equation have
a. no real root?
b. one double root?
c. two distinct real roots?

Solution

Let us check the sign of discriminant.


= 4(m + 1)2 4 = 4m2 + 8m + 4 4 = 4m2 + 8m
4m2 + 8m = 0
m = 2 or m = 0
m
4m2 + 8m

Now we can answer the question.


a. For < 0, there is no real root, and m (2, 0).
b. For = 0, there is one double root, and m = 2 or m = 0.
c. For > 0, there are two real roots, and m (, 2) (0, ).

EXAMPLE

10

Solve the inequalities.


a. (3 x)(x3 2x2 8x)(x2 + 3) < 0

216

b.

(3x+ 2)(x 5)
0
x(x 1)(x2 + x +1)

c.

(x2 x 6)(x 1)17


0
x8 (1 x2 )55
Algebra 8

Solution

a. First we find all the zeros of the polynomials, then we determine the sign for each polynomial and multiply the signs of each polynomial.
(3 x)(x3 2x2 8x)(x2 + 3) = (3 x)x(x2 2x 8)(x2 + 3)
3x=0; x=3
x=0
x2 2x 8 = 0 ; x = 2 or x = 4
x2 + 3 = 0 ; no real solution.
x

(3

x)x(x2

3x

x2 2x 8

x2 + 3

2x

8)(x2

+ 3)

We need a value less than zero, so x (, 2) (0, 3) (4, ).

positive
= positive
positive
positive
= negative
negative
negative
= positive
negative

b. First we find all the zeros of the polynomials. The equality part of the original inequality is satisfied
for these zeros and they must be included in the final solution set. On the other hand, since
division by zero is never allowed, the zeros of x4 x must not be included in the solution set.
(3x+ 2)(x 5)
x(x 1)(x2 + x+ 1)
2
3x+ 2 = 0 ; x =
3
x5=0 ; x=5
x=0
x 1= 0 ; x = 1
x2 + x+ 1 = 0; no real solution.
x

2
3

3x + 2

x5

x1

+x+1

(3x + 2)(x 5)

x2

x4 x

We need a value greater than or equal to zero, so x , (0 , 1) [5 , ).


3

Quadratic Inequalities

217

c.

(x2 x 6)(x 1)17


0
x8 (1 x2 )55

x2 x 6 = 0 ; x = 2 or x = 3
(x 1)17 = 0 ; x 1 = 0 ; x = 1
x8 = 0 ; x = 0 (double root)
(1 x2)55 = 0 ; 1 x2 = 0 ; x = 1 or x = 1 (1 is also a double root)
If the power is an odd
number, you can ignore
it when you calculate the
zero. If the power is an
even number, consider it
just as 2 when you calculate
the zero.

x
x2 x 6

(x 1)17

x8

(1 x2)55

(x2 x 6)(x 1)17


x8(1 x2)55

We need a value less than or equal to zero, so x [2, 1) [3, ).


The signs of M N and
M
, N 0 are the same.
N

Let us summarize the key steps to solving any inequality.


1. Write the polynomial inequality in standard form.
2. Find all zeros of the polynomial(s).
3. Determine the character of the roots.
4. Determine the sign of the coefficient of leading term of the polynomial(s).
5. Construct a sign chart.
6. In the sign chart, from right to left start with the sign of the coefficient of the leading term
a. After each root change the sign, but if there is a double root do not change the sign.
Let us solve Example 10c in another way.

EXAMPLE

11

Solution

Solve the inequality

(x2 x 6)(x 1)17


0.
x8 (1 x2 )55

x2 x 6 = 0 ; x = 2 or x = 3
(x 1)17 = 0 ; x 1 = 0 ; x = 1
x8 = 0 ; x = 0 (double root)
(1 x2)55 = 0 ; 1 x2 = 0 ; x = 1 or x = 1 (1 is also a double root)

218

Algebra 8

x
(x2

x 6)(x

1)17

x8(1 x2)55

Therefore, x [2, 1) [3, ).


EXAMPLE

12

Solution

Solve the inequality (x2 2x 8)(x2 + x 12) 0.


Find the zeros of the polynomials.
x2 2x 8 = 0 ; x = 4 or x = 2
x2 + x 12 = 0 ; x = 4 or x = 3
x

(x2 2x 8)(x2 + x 12)

So, x [4, 2] [3, 4].


EXAMPLE

13

Solution

Find the domain of the function y =

x+ 2
3x 12 x2

3x 12x2 > 0
3x 12x2 = 0
x = 0 or x =

1
4
x

3x

1
4

12x2

1
So x 0 , .
4
EXAMPLE

14

Solution

Solve the inequality (x 1)2(x 2)3(x 3)4(x 4)5 0.


(x 1)2 = 0 ; x 1 = 0 ; x = 1 (double root)
(x 2)3 = 0 ; x 2 = 0 ; x = 2
(x 3)4 = 0 ; x 3 = 0 ; x = 3 (double root)
(x 4)5 = 0 ; x 4 = 0 ; x = 4
x

(x 1)2(x 2)3(x 3)4(x 4)5

So x (, 2] {3} [4, ].
Quadratic Inequalities

219

EXAMPLE

15

Solution

x4 3x3 + 2x2
< 0.
x3 5x2

Solve the inequality


x4 3x3 + 2x2
<0
x3 5x2
x2 (x 1)(x 2)
<0
x2 (x 5)

x = 0 (double root)
x = 1, x = 2, x = 5
x

x4 3x3 + 2x2

x3 5x2

So x (, 0) (0, 1) (2, 5).

EXAMPLE

16

Solution

x 1 x+ 1

< 2.
x
x 1

Solve the inequality

x 1 x+ 1

2 < 0 ; x 0 and x 1
x
x 1
(x 1)2 x(x+1) 2 x( x 1)
0
x(x 1)
2 x2 x+ 1
<0
x(x 1)
2 x2 x+ 1 = 0
x = 1 or x =

1
2

x(x 1) = 0
x = 0, x = 1
x
2x2

x+1

x(x 1)

1
2

1
So x ( , 1) 0 , (1, ).
2
220

Algebra 8

EXAMPLE

17

Solution

Solve the inequality

1
2
1 2x

.
x+ 1 x2 x+ 1 x3 + 1

1
2
1 2x

0 ;
x+ 1 x2 x+ 1 x3 + 1

x 1

x2 x+ 1 2(x+ 1) (1 2 x)
0
(x+ 1)(x2 x+ 1)
x2 x 2
0
(x+ 1)(x2 x+ 1)
x2 x 2 = 0 ; x = 1 or x = 2

x + 1 = 0 ; x = 1 (double root)
x2 x + 1 = 0 ; no real solution
x
x2

x2

(x + 1)(x2 x + 1)

So x (, 1) (1, 2].

EXAMPLE

18

Solution

Solve the inequality

2 x2 + 3x 2 . (1 x2 )
(x2 + 3x) . x 2

Since |2x2 + 3x 2| and |x 2| are non-negative, just check their roots.


2x2 + 3x 2 = 0 ; x = 2, x =
x2=0; x=2

The absolute value of a


number is never negative.
|x| 0, x

0.

1
2

1
satisfy the inequality, so these values are in the solution set. However, 2 is not in
2
the solution set because it makes the denominator zero.

2 and

1 x2 = 0 ; x = 1, x = 1
x2 + 3x = 0 ; x = 0, x = 3
x
|2x2

+ 3x 2|(1

x2)

(x2 + 3x)|x 2|

So x (, 3) [1, 0) {2,
Quadratic Inequalities

1
} [1, 2) (2, ).
2
221

EXAMPLE

19

Solution

Find the domain of the function y = 4


7

x2 6x 16
+ 7 3x4 4x3 + x 1 if y is a real number.
x2 12 x+ 11

3x4 4x3 + x 1 is a real number for all values of x because the index is an odd number.

x2 6 x 16
x2 6 x 16
is
a
real
number
if
0
x2 12 x+ 11
x2 12 x+11

Therefore, y is a real number if


x2 6x 16
0.
x2 12 x+ 11
x2 6 x 16 = 0 ; x = 2 , x = 8
x2 12 x+ 11 = 0 ; x = 1, x = 11
x
x2

6x 16

x2 12x + 11

11

So x (, 2] (1, 8] (11, ).

EXAMPLE

20

Solution

Solve the inequality


3

If ab < ac then b < c.

3 x 1
x 1

3 x 1
3x 1

<8

<2

x 3
3 x7

3 x 1

x 3

2 x1 < 8 3 x7 .
1

x 3
3 x 1 3
; 2 x1 < (2 3 ) 3 x7

3x 3
3 x 7

3x 1
3(x 3)
<
3(x 1)
3x 7
3x 1 3 x 9
<0

3x 3 3x 7
(3x 1)(3 x 7) (3 x 9)(3 x 3)
<0
(3x 3)(3 x 7)
12 x 20
<0
(3x 3)(3 x 7)
12 x 20 = 0 ; x =

5
3

3x 3 = 0 ; x = 1
3x 7 = 0 ; x =
222

7
3
Algebra 8

12x 20

5
3

(3x 3)(3x 7)

7
3

5 7
So x ( , 1) , .
3 3

Check Yourself 3
Solve the inequalities.
1. x2 + 5x 6 > 0

2. (x + 3)3(x 1)2(x 4) 0

2
x2 + 4)
4. (x 2)(
<0
x2 4
Answers

2
1
5. 4x x
2

1. (, 6) (1, )

2. (, 3] {1} [4, )
1
5. (, 0] [ , )
2

3.

2
3
4

+
2 x 2 + x 4 x2

4. (2, 2) (2, 2)

3. (2, 2) [6, )

Obvious is the most dangerous word in mathematics.

EXERCISES

6 .2

1. Determine the sign of each polynomial.

3. For which values of k does the equation

a. x2 5x + 4

b. 2x2 + x 6

x2 + 2(1 k)x + 1 = 0 have

c. 2x2 3x + 4

d. 16x2 + 8x 1

a. no real solution?

e. 4x2 + 10x 25

f. 12x2 + 43x + 1

c. two distinct real roots?

b. one double root?

4. For which values of b does the equation


3x2 + (b + 1)x + 1 = 0 have two distinct real
roots?

2. Solve the inequalities.


a. x2 < 9x 20

b. 4x 7x2 > 0

c. x(6x + 7) 0

d. 2x 6 < 3x2

e. 3 x2 + 8 x

16
3

g. (2x 1)2 > (x 5)2


Quadratic Inequalities

f. x(x 1) + 1 <

5. For which values of a does the equation


ax2 + (a + 1)x + 2 = 0 have no real root?
4
1
x+
5
5

6. The product of a number and four plus the


number is less than 15. Find all possible integer
values of the number.
223

7. Solve the inequalities.

8. Solve the inequalities.

a. x(x 1)2 > 0

a. 1

b. (2 x)(3x + 1)(2x 3) > 0


c. (3x 2)(x 3)3(x + 1)3(x + 2)4 < 0
d.

e.

(x 1)(3 x 2)
>0
5 2x

x4 + x2 + 1
<0
x2 4x 5

g.

3x 2
<3
2x 3

2 x2 + 18 x 4
h.
>2
x2 + 9x+ 8

j.

c.

(x+ 1)(x+ 2)( x+ 3)


>0
(2 x 1)( x+ 4)(3 x)

f.

i.

b.

d.

1
3

k. x x2 0
x
2

m.

(x2 + 4x+ 4)(x 2)1001


<0
(x 2)2004 x+ 4

n. |6x2 2x + 1| 1
o.

x+ 2
<3
2x 3

p.

x2 3x+ 2
>1
x2 + 3x+ 2

q.

x2 3x 1
1
x2 + x+ 1

224

3 x (x2 + x+ 3)
2 x+ 2 x2 5x+ 2
x3
2

x 5x+ 6

a. y =

2x
1 2x
3
3
2
x +x
x 3x2

1
3
>
2
3x 2 x
7x 4 3x2

(a2 + a+ 1)x2 + 4
>0
a 4 x a 4 x2

<0

9. Find the domain of each function.

x+ 1
3
1

x2 x2 2

l.

1 1
1
1
1
+

+
<0
x x2 x3 x4 x5

2
3

x 49

b. y =

1
x 4

c. y =

3x 6
x+ 2

d.

(x4 5x3 + 6 x2 )(1 x2 )

10. Solve the inequality

12
3
2
> 1.
x + 2x x + 2x 2
2

4
3
4x2 8
11. Solve the inequality x + 3x +
< 0.
2

Algebra 8

We saw in Chapter 1 that a set of simultaneons equations to solve is called a system of equations.
A system that of equations includes more than one inequality is called an inequality system.
To find the solution of a system, we solve each inequality separately and then find the
intersection of the solutions.
EXAMPLE

21

Solution

x2 7 x 8 > 0
Solve the inequality system
.
2
x 4x+ 3 > 0

First find the zeros of each polynomial.


(1) x2 7x 8 > 0
x2 7x 8 = 0 ; x = 1, x = 8
(2) x2 4x + 3 > 0
x2 4x + 3 = 0 ; x = 1, x = 3
We need to find values so that both polynomials are greater than zero. Let us check the chart.
x

(1)

(2)

system

We can see that both polynomials are greater than zero when x (, 1) (8, ). This
is the intersection of the solutions.
EXAMPLE

22

Solution

x2 + x 4
<1

Solve the inequality system


.
x
x2 < 64

(1)

x2 + x 4
x2 + x 4 x
<1;
<0 ;
x
x
x2 4 = 0 ; x = 2 ; x = 0

x2 4
<0
x

(2) x2 < 64 ; x2 64 < 0


x2 64 = 0 ; x = 8
x

(1)

(2)

system

This time, both polynomials need to be less than zero, so x (8, 2) (0, 2).
Quadratic Inequalities

225

EXAMPLE

23

Solution

x5 100 x3

Solve the inequality system


.
(x+ 9)(5 x x2 18)

x2 18 x+ 45

(1) x5 100x3
x5 100x3 0 ; x3(x2 100) = 0
x = 0, x = 10
(2)

(x+ 9)(5 x x2 18)


0
x2 18 x+ 45
x + 9 = 0 ; x = 9
5x x2 18 = 0 ; no solution
x2 18x + 45 = 0 ; x = 3, x = 15
x

10

10

15

(1)

(2)

system

So x [10, 9] [10, 15).


EXAMPLE

24

Solution

Solve the inequality


2x 1 0

x+ 2 > 0

2
2 x 1 < (x+ 2)

(1) 2x 1 0

If

f ( x) g( x)

2x 1 = 0
1
x=
2

f ( x) 0

then, g( x) 0

2
f ( x) g (x)

2 x 1 < x+ 2.
(1)
(2)
(3)

(2) x + 2 > 0
x+2=0
x = 2

(3) 2x 1 < x2 + 4x + 4
x2 + 2x + 5 > 0
x2 + 2x + 5 = 0
since < 0, there is no real solution
1
2

(1)

(2)

(3)

system

So x [
226

1
, ).
2
Algebra 8

25

Solve the inequality

Solution

Case 1
__________________

Case 2
__________________

(1)
x < 0
2
x + x 2 0 (2)

(1)
x 0
2
x + x 2 > x2 (2)

(1) x < 0

(1) x 0

EXAMPLE

If

f ( x) g( x) then

x2 + x 2 > x.

(2) x + x 2 = 0 ;

g( x) < 0

f ( x) 0

(2) x 2 > 0 ;

x1 = 2, x2 = 1

x 2 = 0, x = 2

or
g( x) 0

f ( x) g2 (x)

Case 1

(1)

(2)

(1)

(2)

system

Case 2

system

So x (, 2] (2, ).

EXAMPLE

26

Solution

Solve the inequality (x2 + x + 1)2 4(x2 + x + 1) + 3 < 0.


For the inequality, we let t = x2 + x + 1. Then the original inequality becomes
t2 4t + 3 < 0. First let us solve the inequality for t.
t2 4t + 3 = 0 ; t = 1, t = 3
x

t2 4t + 3

So 1 < t < 3. Now solve for x.


1 < x2 + x + 1 < 3
0 < x2 + x < 2, which gives us the system of inequalities
x2 + x > 0
(1)
.

x2 + x 2 < 0 (2)
Quadratic Inequalities

227

(1) x2 + x > 0
x2 + x = 0 ; x = 0, x = 1
(2) x2 + x 2 < 0
x2 + x 2 = 0 ; x = 2, x = 1
x

(1)

(2)

system

So x (2, 1) (0, 1).

27

Solve the inequality |x2 3x + 2| 2x x2.

Solution

Case 1
________________________________________

EXAMPLE

x2 3x+ 2 0
(1)

x2 3x+ 2 2 x x2 (2)

(1) x2 3x + 2 0
x2 3x + 2 = 0 ; x = 1, x = 2
(2) x2 3x + 2 2x x2 ; 2x2 5x + 2 0
1
2x2 5x + 2 = 0 ; x = , x = 2
2
Case 2
_______________________________________
x2 3x+ 2 < 0
(1)

(x2 3x+ 2) 2 x x2 (2)

(1) x2 3x + 2 < 0
x2 3x + 2 = 0 ; x = 1, x = 2
(2) (x2 3x + 2) 2x x2
x2 + 3x 2 2x x2
x20
x2=0; x=2
228

Algebra 8

Case 1

1
2

(1)

(2)

(1)

(2)

system

Case 2

system

1
So x , 1 (1, 2) {2}, i.e.
2
1
x , 2 .
2

Check Yourself 4
Solve the systems.
x2 4x 0
1.
x 3 0

x2 4x+ 3 > 0
2.
x2 (x 7)2 > 0

3.

x2 x > 1+ x

Answers

1. [4, ) 2. x 0, x 7, (, 1) (3, ) 3. ( , )
3

In the center of a square pond whose side measures 10 m


grows a plant whose top reaches 1 m above the water level. If
we pull the plant toward the bank, its top becomes even with
the waters surface. What is the depth of the pond and the
length of the plant?

Do not worry about your difficulties in mathematics. I can assure


you that mine are still greater.
Einstein
Quadratic Inequalities

229

EXERCISES

6 .3

1. Solve the inequality systems.

4. Solve the inequalities.

x+ 2 > 0
a.
2 x 3 < 0

x 1 > 0
b.
x2 2 x 3 < 0

x2 + 3x 10 > 0
c.
2 x2 + 11x 6 < 0

x 2
x+ 1 0

d.
x2 1
x 3 > 0
2 x2 + 2 < 5x

x2 x

x2 4x+ 3 < 0
e.
2 x 4 < 0

f.

x+ 2
x+ 1 > 1

g.
x
x 2 > 2

x 1
<0

3x
h.
x 1
x+ 1 < 0

a.

2 x+ 10 < 3x 5

b.

x+ 3
2
4x

c.

x2 x 12 < x

d.

17 15 x 2 x3
>0
x+ 3

e.

x+ 1 x 2 1

f.

x+2

<x8

5. Solve the inequalities.


a.

x2 9 < x+ 2

b.

2 3x x2 2 x+ 3

c.

x2 x 2 x 1

d.

4 1 x < 2 x

e.

x2+ x3<4

2. Solve the inequality systems.


x+ 3
3 x <2

a. x3 < 16 x

4 x2

x4

6. Solve the inequalities.


b. 4x 2 < x2 + 1 < 4x + 6

c. 5x 7 < 4 x + 2 3x < 4
x5
5 x x 25

b. |x2 3x 15| < 2x2 x

2+ x x 1

c.

6 x2 < x 1

b.

1+ x x 1

7. Solve the inequality

x+ 3 4 x 1 + x+ 8 6 x 1 > 1.

3. Solve the inequalities.


a. |4 3x| 2 x

a.

8. For which values of m are the following inequalities

satisfied for any real number x?

c. |2x2 + x + 11| > x2 5x + 6

a. 2x2 + 5x + m > x2 x 7

d. |x 4| + |2x + 6| > 10

b. 2x2 + mx2 2x + m 1 > (1 + m)x2 4x m

e.

x3
x2 5x+ 6

f. |2x + 1| |5x 2| 1
g.
230

x2 x 12
x3

c. (m + 2)x2 3x + m 2 < 0

2x

d. x2 6x + 4m2 > 7

9. Solve the inequality

(x2 + 6x + 14)2 9 (x2 + 6x + 15) + 9 < 0.


Algebra 8

CHAPTER REVIEW TEST

6A

1. What is the solution of the inequality

5. What is the solution of the inequality

2 3(1 x) < x + 1.8?


A) x > 1.4

B) x < 0.4

D) x > 0.7

C) x > 0.7

E) x < 1.4

1
1
<
?
x+ 2 x 1

A) x < 0 or x > 2

B) 2 < x < 1

C) x < 2 or x > 1

D) 0 < x < 2

E) 2 < x < 2

2. What is the solution of the inequality


x2 + 5x 14 0?
A) [2, 7]

B) [7, 2]

D) [2, 7)

C) (7, 2)

E) (2, 7]

6. What is the greatest integer value of x which satisfies


the inequality x2 13x + 36 < 0?
A) 13

3. What is the solution of the inequality

B) 7

C) 8

D) 8

E) 13

x2 4
> 0?
x2 1

A) (, 2) (1, )

B) (, 2) (2, )

C) (, 1) (1, )

D) {1, 1}

E) (, 2) (1, 1) (2, )

7. What is the solution of the inequality


1 1
1
+ + < 0?
x x 2 x3

4. What is the solution of the inequality

A) (, 0)

B) (0, )

C)

D)

E) {0}

(2 3x)(3 x 3)
0?
3x +1
1 2
A) , [1, )
3 3

1 2

B) ; ; 1
3 3

2
C) (1, + )
3

1 2
D) , (1, + )
3 3

1 2

E) ; ; 1
3 3

Chapter Review Test 6A

8. How many integer values of x are there which


satisfy the inequality |x + 2|

2 x +7 ?

A) 1

D) 5

B) 3

C) 4

E) 7
231

9. What is the solution of the inequality

13. What is the domain of the function

(1+ x2 )2001 (1 x)2002


0?
( x + 2)2003 (3 x)2005

A) x > 2

y=

B) x < 2

D) x > 3

C) x < 3

E) 2 < x < 3

x2

20 8 x x2

A) [1, 7]

B) [10, 2]

D) [5, 4)

C) (10, 2)

E) (2, 10]

14. What is the solution of the inequality system


10. The equation 2x(ax 4) x + 6 = 0 has no real
2

roots. What is the smallest possible integer value


of a?
A) 1

B) 2

C) 3

D) 4

E) 5

11. What is the solution of the inequality

B) (2, 1)

D) (5, 1) (1, )

A) (2, 3)

C) (1, 3)

E) (1, 1) (5, )

D) [3, 2)

12. What is the sum of the integer values of x which


x2 8x +7
< 0?
( x + 2)2

A) 32

C) 24

232

B) 28

D) 20

C) (2, 3)

E) (3, 2)

x < 2
?

( x 3)( x 2) 0

A) (2, 2)
D) [3, 2)

satisfy the inequality

B) [2, 3]

15. What is the solution of the inequality system

( x 3)( x + 2)
<1?
x2 1

A) (2, 1) (1, 3)

x2 5x +6 0
?

x2 + x 2 0

B) [2, 3]

C) (2, 3)

E) (3, 2)

16. What is the solution of the inequality


3 x x + 3 >1?

A) (2, 2)
E) 16

D) (, 2)

B) (1, )

C) (2, 3)
E) (2, )
Algebra 8

6B

CHAPTER REVIEW TEST


1. What is the solution of the inequality

5. What is the solution of the inequality

x +1 x 1

> x?
2
3

A) x > 1

x2 > x +1?

B) x < 3
D) x < 1

C) x < 3

B) 1,
2

A) (2, 4)

E) x > 6

C) (, 2)

E) ,
2

D) [2, )

2. What is the solution of the inequality


(x 6)(x + 3) 2 2x?
A) [5, 4]

6. What is the smallest possible integer value of x

B) [4, 5]

D) [5, 4)

C) (4, 2)

E) (4, 5]

3. What is the solution of the inequality


x2 + x
<1?
5x 3

which satisfies the inequality (x + 7)2(x 4) 0?


A) 3

B) 4

C) 8

D) 8

E) 13

7. What is the solution of the inequality


(x2 + 8x 9)(x2 4) 0?

A) , (1, 3)
5

3
B) , 1 (3, )
5

C) (, 1)

D) {1, 1}

A) (, 2) [1, )

B) (, 2] (2, )

C) (, 1] [1, )

D) R {1, 1}

E) (, 9] [2, 1] [2, )

E) (1, 1) (3, )

8. Which inequality is represented in this sign


chart?

4. What is the solution of the inequality


1
5
+
<1?
2 x 2+ x

-2

A) [, 1) (1, 3)

B) [3, ]

A) x2 3x + 10 < 0

B) x2 > 3x 10

C) (, 2) (2, )

D) (1, 3)

C) x2 < 3x + 10

D) x2 2x + 5 0

E) (1, 3)
Chapter Review Test 6B

E) x2 < 2x + 5
233

9. What is the solution of the inequality

13. What is the domain of the function

( x 1)3( x 2)
0?
( x 3)2

y = 4 x x2 +

B) [2, ]

A) (, 1)

C) [1, 2]
E) (2, )

D) [1, 2)

What are the possible values of b?


B) (, 12]

D) (, 12] [12, )

x2

A) (2, 4]

B) [1, 2]

D) [2, 4)

10. The equation 3x2 + bx + 12 = 0 has real roots.


A) [12, 12]

C) [12, )

E) (1, 1) (5, )

E) (2, 4]

14. What is the solution of the inequality system

x5
>0
x+ 3
x+ 7
<0
x3

A) (2, 3)

B) [7, 3]

D) [3, 3)

11. What is the solution of the inequality


1

A) ,
2

x
1?
x +1

C) (4, 2)

C) (7, 3)
E) (7, 2)

15. How many integer values of x are there which


satisfy the inequality (x + 8)(x - 1)2(x 5) < 0?

B) (2, 1)

D) ,
2

C) (1, 3)

A) 8

B) 9

C) 10

D) 11

E) 16

E) (1, 1)

16. x1 and x2 are the roots of x2 3mx + m 3 = 0.

12. What is the sum of the integer values of x which


( x 2)( x 4)x2
satisfy the inequality
0?
( x +1)2

A) 8
234

B) 9

C) 10

D) 11

1
1
+ > 4 is given.
x1 x2

What are the possible values of m?


A) (, )

E) 16

D) (3, 12)

B) [2, ]

C) (, 3)
E) (0, 12)
Algebra 8

EXERCISES

1 .1

1. a. 6 b. 10 c. 11 d. 4x e. 5y f. 11a2
c. 2 d. 3x e.

y
2 3xy
f.
3
2x

2. a. 3 b. 5 c. 6 d. 9 e. 4x f. 12xy g. a15 h. 72xy

4. a. 22 b. 62 c. 93 d. 1010 e. 55 f. xyx 5. a. 53 b. 75 c. 0 d. 26

e. 212 f. 25 g. 103 h. 13xx i. 1 j.

4
3

6. a. 2 + 1 b. 3 2 c. 2 + 6 d. 22 + 3 e. 7 1

f. 3 + 2 g. 2 3 h. 22 i. 6 j. 2 k. 3 + 1

7. a. 6 b. 2 c. 4 d. 4

6 3
12

11
d.
11

11. a.

2 33 2
6

e. 1 2 f. 23 4 g. 2 + 3 h.
22
5 3 1
c. 63 + 711 d.
5
2

b.

EXERCISES
1

1. a. 212 b. 7 c. 5 7 d. xa 3

e. 215 f. 3 5 g. 3

c. 2 d.

5
4

e. 2 + 1 f. 2

l. 7 m. 9 n. 9 o. 6 + 1 p. 16 q. 1 r.
7. a. 4 b.

1
5

d. 2x e. 81 f.
13. ax+2y

a
c. a d. ( )x 1 e. 2 f. 5
b
89
30

14.

1
4

g. 1 h. 18
15. 2

28. 57122 57121

Answers to Exercises

j. 3 6 k. 5 + 1 l. 1

12. a. 2 b. 1 c. 9

1
3

2
c. ( )3
3

2. a. a b. 3 b2

d. c xab

e. 4 a

18
7

j. 2 k. 3 l. 9 m.

1
5

n. 12 3 o.

16. 128

3. a. 6 3

b. 10 a

s. 2 6. a. 27 b. 55 c. 46 d. 24 e. 32x+1 f. 30x g. y x h. 2100

8. a. 256 b. 729 c. 4 d. 64 e. a9 f. a

9. a. 2x+1 b. 0 c. 110 3x 2

4
10. a. 6 b. 8 c. 5 d. 9 e. 4 f. { , 4} g. 3 h. 4 i. 3 j. 3
3

17. 2

18. x 2

19. 4

20.

22
l. 0 m. 14 22. 15 23. 0
3
19
29.
30. 2651 31. 121
8

f. 20 g. 27 h. 23 i. 22 j. 0 k.
27. 9

2 70 5 10
3

3 2 2 3
6

27
p. 24 313 q. 24 231
4. a. 1
5
39
17
5. a. 4 b. 26 c. 0 d. 54 e. 44 f. 17 g.
h.
i. 2 j. {4, 4} k. 10
2
5

c. 100 d. 5 e. 3 f. 5 g. 3 h. 2x i. a2 6 a
b. 6 24 33

i.

10. a. 3 b.

1 .2
2

12 3 8
23

9. a. 15 + 10

8. a. 2 b. 9 c. 24

b. 7 + 7 c. 4 6 2 d. 12 e. 6(3 + 2) f. 4 g. 1 h. 8 i. 2 j. 2 k. 1 l. 3 m. 13
c.

3. a. 5 b. 3

16
5

24. 3

11. 2

12. 268

21. a. 7 b. 83 c. 2.8 d. 5 e. 3
25. 32

26. 8

2
2

235

EXERCISES

2.

frequency (f)

1
2
1
3
3
5
3
3
1
2

3.

Percent

judo
karate 10%
5%

35
30

wrestling
7%

25

football
20%
basketball
12%

20
15

swiming
29%

10

volleyball
17%

Other

Expenses

5. a. 2705000 tons b. 2001 c. 1080000 tons

Other

EXERCISES

5
d. 5 e. 2 5
3
5

b. 0,

500 000
400 000
300 000
200 000
100 000

3 .1

1. a. 0 b. 0 c. 0 d. 0 2. a. 0,
c.

600 000

2003

Pichard
Whiting
Hake

Hazelnut production in Turkey (tons)

2002

Blue fish

700 000

2001

d.

Scad
Gray mullet

2000

Anchocy
Horse mackerel

1999

4.

Books

Clothing

5
Food

1
2
3
4
5
6
7
8
9
10

Entertaiment

scores (x)

Rent

1.

2 .1

f.

5
8
3
1
b. 0, c. 0, d. 0,
2
3
7
9
4
5

3. a. 0, 2 4. 0,

5
2

5. 1, 2 6. a. 4 b. no real solution

7. a. 1,5 b. no real solution c. (1 5), (1 + 5) d. 2, 4 8. 3

5 3
1 2
2 3
1
c. 7 d. 5 e. 1, 2 f. 1, 2 g. 1 h. ,
i. ,
j. ,
3 2
4 3
5 2
3

9. a. 0, 1

10. a. a, 1 b. no real solution c.

5b
,1
2a

11. a. (2 + 5), (2 5), b. (7 2), (7 2) c. (3 22) d. no real solution e. no real solution f. no real solution
3
1
g. 3 22 h. 1 7 12. a. 2 2 b. 1,
c. x
d. 5 13 e. no real root f. 1 2 7 g. no real root
2
2
6
3
6
4
1
7
8
2
, 1 l. 1, 23
14. a. two real roots
h. x
i. , 4 j. , 1 k.
13. a. 1 33 b. 3 73 c. 1, 5 d. , 5
5
26
4
5
3
8
4
9
b. two real roots c. one double root d. one double root e. no real solution f. no real solution
15. a. a
40
9
7
7
7
9
1
b. a
c. a
16. a. m
b. m
c. m
17. , 1 18. 8 cm, 10 cm
40
2
2
2
40
4

19. 12 cm, 20 cm
236

20. 4, 6, 8, 10

21. 256 cm2, 441 cm2

22. 7, 49
Answers to Exercises

EXERCISES

72
40
4 5
11 6
10 5
18 8
24
c.
e. , f. (32 2), 0 2.
, b. ,
, d. ,
49
21
3 6
10 5
3 3
7 7
7

1. a.

153
256

d.
10.

3 .2

5. m1 = 5, m2 = 4, x2

14
,4
3

19. 7

11. 5

20. c = 2, 1,

7 5
12. ,
3 3

1 5
2

EXERCISES

5
4

6. n1 = 4, n2 = 2, x2

13. 6, 2
21. 13

14. 1

15. k > 6

22. 1 , 1 13
2
2

2
3

7. 0

16. 6, 7

3.

2
5

4. a.

8. m = 2n
17. a b

17
21
b. 53 c.
9
4

9. k = 3

18. 0

23. 4 + 23 cm

3 .3

1. a. x2 1 = 0 b. 4x2 8x + 3 = 0 c. x2 4x = 0 d. x2 4x + 2 = 0 e. x2 23x + 1 = 0 f. p2q2x2 (p2+q2)x+1=0


2. x2 + 2x + 5 = 0 3. x2 8x 12 = 0 4. x2 + x + 27 = 0 5. a. x2 4 = 0 b. x2 + 8x + 7 = 0
c. 4x2 25 = 0 d. x2 2x + 1 = 0 e. x2 + 2mx 8m2 + 27m 18 = 0 f. mx2 2mx 8m 9 = 0

6. 5

4
13. 18(2 + 1)
7
14. from A to B 60 km/h, from B to A 80 km/h 15. 160 km/h 16. 16 km/h 17. 7 women 18. 80 km/h
19. 24 hours, 48 hours 20. 20 km/h 21. height = 6 m, base = 12 m 22. 40 kg 25%, 20 kg 40%
23. 60 hours, 84 hours 24. 4 hours, 8 hours 25. 6 m, 8 m

7. 7, 6, 5 or 5, 6, 7 8. 9 9. 15 hours, 10 hours 10. 3 hours, 2 hours 11. 6 hours 12.

EXERCISES

3 .4

1. a. 2, 3 b. 3 c. 10, 23 d. 5 7 e. 5

1
3 5
f. 79 g. 4 h. 2 3 i. 2 2 2. a.
b. (1 10), 1, 3
2
2

c. 2, 1, 0, 1 d. 1, 2 e. 6, 1

7
6 42
1
3. a. 0, , 1, 5 b. 10, (4 2), 2 c. 2 d. 5 e.
f. 5, , 1 g. no solution
3
3
4

h. 1, 2, 4 i. 0 j. 3 15

4. a. 2 b.

11 21
c. 7 d. 1, 2 e. 12 f. 3 g. 1, 2 h. 0 i. 1, 7 j. no solution
2

8
5. a. 7, 8 b. 2 c. 0 d. 0, 2 e. 0 f. no solution g. , 1 h. 2
3

g. 3 7, 2 h. 2 i. no solution
d. 1, 4
Answers to Exercises

7. a. 0 b.

6. a. 1 b.

7
, 3 c. 2 d. 1, 5 e. 1, 3 f. no solution
3

2 1
3
3
c. 2, 0 d. 2 e. , , 2 f. x ( , ] 8. a. 1 b. 0, 3 c.
3 2
2
4
2

237

EXERCISES

3 .5

5
7
1. a. (6, 18), (18, 6) b. ( , ) c. (3, 6), (10, 7) d. (10, 15), (15, 10) e. (4, 1), (4, 1) f. (3, 3), (3, 3)
2
2
g. (3, 4) h. (0, 5), (1, 4)
3
2. a. (4, 4), (6, 2) b. (0, 0), ( , 3) c. (0, 0) d. (1, 0) e. (2, 4), (4, 2), (22 4, 22 4), (22 4, 22 4)
2
7
7
7
,
)(
,
2
2
2

f. (0, 2), (2, 0) g. (0, 2), (0, 2) h. (2, 1), (2, 1), (

EXERCISES
1. a. x = 1, y = 4

4 .1

b. x = 1, y = 0

2. a. {3}

b. 1

3. a. (1, 1], (3, 2) b. (2, 5), (3, )

e. {3} f. 2

7
)
2

c. {1, 3, 5, 7, 8, 9, 10}

d. {1, 2, 3, 4, 5, 7, 8, 9, 10}

4. a. A = {1, 2}, B = {a, b, c} b. A = {1, 2}, B = [2, 4]

5. a. {(1, 1), (1, 2), (1, 3), (2, 1), (2, 2), (2, 3), (3, 1), (3, 2), (3, 3)}, {(1, 1), (1, 1), (1, 1), (1, 1)}, {(1, 1),
(1, 1), (2, 1), (2, 1), (3, 1), (3, 1)}, {(1, 1), (1, 2), (1, 3), (1, 1), (1, 2), (1, 3)} b. {(December, December),
(December, January), (December, February), (January, December), (January, January), (January, February),
(February, December), (February, January), (February, February)}, {(2, 2), (2, 5), (2, 6), (5, 2), (5, 5), (5, 6), (6, 2),
(6, 5), (6, 6)}, {(December, 2), (December, 5), (December, 6), (January, 2), (January, 5), (January, 6), (February, 2),
(February, 5), (February, 6)}, {(2, December), (2, January), (2, February), (5, December), (5, January), (5,
February), (6, December), (6, January), (6, February)}
6. a.

b.

B
6

1 2 3

1
2
3
5

b
c
d
e

b.

1
2
3
4
5

0
1
2
3
4

7. a. {(0, 0), (1, 1), (2, 4), (3, 9)}

B
4

4
3
2

8. a.

c.

b. {(2, 5/3), (1, 1), (0, 1/3), (1, 1/3), (2, 1),
(3, 5/3)}

2
-2

9. a.

b.

y
3
2
1

1 2

d.

1
1

10. a. {(December, winter), (September, fall),


1
(June, summer), (March, spring)}, {December,
September, June, March}, {winter, fall, summer, spring}
b. x = 2y 3, [43, 65], [20, 34] c. x = y2 + 2, [2, 11], [3, 3]
238

c.

1
-1

3 x

Answers to Exercises

11. a.

b.

c.

(0, 4)
(2, 0)

(-2, 2)

12.

(0, 0)

(4, 2)

(2, 2)

(0, -3)

(0, 3)

(0, 0)

(4, -1)

-1

13. {(a, a), (a, b), (b, b), (b, c), (c, a), (c, b), (c, c), (c, d), (d, a), (d, d)} 14. 84

17.

5
4

15. use the fact that n(A) + n(B) includes n(A B) twice. 16. {(1, a, blue), (1, a, red), (1, b, blue),
(1, b, red), (2, a, blue), (2, a, red), (2, b, blue), (2, b, red), (3, a, blue), (3, a, red), (3, b, blue),
(3, b, red)}, use three coordinate axes that are perpendicular to each other in the space

1
-1

-5

4 .2

EXERCISES

x
2. a. f(x) = 5x + 2 b. f(x) = x2 2x c. f ( x) = + 3x3
2
3. a. multiply by 2, then subtract 4 b. add 1, then take the square root of all c. divide by three times itself increased by 1

1. a. function b. not c. not

+ 2 3 x2 1
4. a. 3, 33, 2x2 + 5x, 2x2 + 5x, 2x 5x, 2x2 + 4hx 5x + 2h2 5h b. 1 , 11 , 9 x2 1 , 6 x
,
,
2 18 9x + 2
x2 + 2 x4 + 2

6a + 3b 1
2
4a + 4ab + b2 + 2

6. a.

b.

y
-2 -1

2
2
4
4
2
2
c. , undefined, 4u3 +1 , 3 x +22 x + 2 , 6u +1 2 , 6u + 2 4u +1 2
3
8u + 4u x + 3x +5x + 3 u + 2 u
u + 4u + 4u

x
-1
-1

3
1 2

d.

3
2
1

3
2

1 2
-1

c.

7. a.
1

1/2

5.

8
3

7
4
d. 21; 3
; 7 b. 6, 1; 6 c. 2;
3
5

8. a. function b. not c. function d. function


e. not f. not

-4

c. (, 4) (4, ) d. (, 3) (3, 3) (3, ) e. [2, ) f. (, 3) (3, 0) (0, )


5
10 10
1
3

3 5
g. [5, 6) (6, ) h. [5, 10] i. ,1 j. [1, 1) 1, k. 3,
, 10. a. , b. [4, 1) ( 1, 3 )
3 3
2
4
5

c. (1, 1) (1, ) d. (, 2) (4, ) e. [1, 2] f. [1, 2] {0} g. (0, 1) (1, 3) h. [5, 1) (1, 5] i. (, 1] [11, )
9. a.

b.

2
2
11. a. f ( x) = 3 x b. f ( x) = x 4x c. f ( x) = x 3 ( x + 2)(1 x)
2
x + 2x
x5
b. 4, 4 c. 13. a. odd b. neither c. even d. neither e. even f. even g. odd h. odd i. neither j. odd

j. [5, 2] [2, 3) (3, 5]

12. a. 4
3
14. a. even b. odd c. even d. odd e. neither f. even
Answers to Exercises

239

15. a.

(-4,6)

b.

(4,6)

c.

x
(-2,-2)

(-4,6)

(-5,-6)

(-7,4)

(0,0)

(4,4)

(0,2)
(-5,0) (-2,0)

(0,4)

(-2,-2)

(7,-4)
(4,-6)

(5,0)

(-4,-4)

(0,-4)

(7,-4)

(0,-2)

(5,-6)

(-5,6)

(2,2)

(-7,4)

(2,0)

(-5,0)

(7,-4)

(-7,-4)

(2,-2)

(7,2)
(-2,0)

(0,0)

16. a.
(2,4)

(-7,2)

(7,4)

(-7,4)

(-2,4)

(0,4) (4,4)

(-4,4)

b.

(2,4)

(7,2)

(2,0) (5,0)
(0,-2)

(-7,-2)

x
-5

9 11 x

(-2,-4)

c.

(5,-6)

17. consider f(x2) f(x1) where x1 < x2 on the given interval 18. 3 19. 2
20. a. , b. increasing on (, 2] and [5, ), decreasing on [2, 5] c. 6, 3, 6; 3
d. 4, 5, 8 e. (, 6) (3, 6)

345

21. a. (-, 10) (10, 10) (10, ), [7, ) b. 5, 5 c. 2 d. undefined e. [5, 0) (0, 5] f. (-, 10) (10, )
g. (10, 6] {0} [6, 10) h. even i. increasing on [3, 6] and (10, ), decreasing on (, 10) and [6, 3],
constant on (10, 6], [-3, 0), (0, 3] and [6, 10) j. 7, k. [3, 0) (0, 3] l. (6, 3) (3, 6) 22. 0 23. d(t) = 15t
24. a. F(x) = 1.8x + 32 b. 1.8

10t
if 0 t 90
225h2

if 0 h 2
25. a. V(t) = 10t b. V( h) =
c. h(t ) = 15
1200 h 1500 if 2 h 3
2+ t 90
if 90 t 210

120
26. until his twenties the person gained weight and kept his weight constant until his forties, then he lost weight

because of a diet or an illness and after his mid-forties he started to gain weight again. 27. he left home at 8 a.m. and
1
during the day visited four places, probably for a sale, and at 6 p.m. left for home. 28. a. (, 4] b. [3, ) c. 0,
6

d. (, 1) (1, ) 29. 100 30. 0, 2, 1 31. 8 32. decreasing on , , increasing on 1 ,


5
5

2
13
2x
3
33. x 3x 34.
35. 12 36.
9
3x

EXERCISES

4 .3

1. a. x2 + x, x2 + x + 2, x3 + x2 x 1,
x5 +8 x4 +15 x3 ,
240

1
with domain \ {1, 1}
x 1

x2 + 3x
with domain \ {5, 0}
x +5

b.

x3 + 4 x2 +5 x, x3 + 2 x2 5 x,

c. x + 3+ x + 2, x + 3 x + 2, ( x + 3) x+ 2,

x+ 3
x+ 2

Answers to Exercises

d.

x 1+ x +1,

x 1 x +1,

2 x2 + 2 x
, x4 + 2 x3 + 2 x2 + x
x2 2 x +1

e. one-to-one
13. a.

8. x

2
2. a. 25 b. undefined c. 11 3. a. x, x 2 + x +1 ,
x +x 1
x +1
7
x
+6
11
x
5
x
+
3
10 x 3
b. x4 4x3 + 4x2, x2, x2 2x + 2, x + 2
c.
,
,
,
2 x + 3 x + 3 4x +9 13 3 x

x4
41
c. f(x) = x3 x + 4, g(x) = 2x2 x 5. a.
b. 1
5
11
9. a. not b. not c. one-to-one 10. a. one-to-one b. one-to-one c. not d. one-to-one

4. a. f(x) = x, g(x) = 2x 4
c. 4 d. 9 6. 2 7. 2

x 1

x 1 x +1,

b. f ( x) = x7 , g( x) =

11. a. one-to-one b. one-to-one c. not

y
(5, 4)

b. inverse function c.
does not exist

12. a.

6x
4

x+ 4
x3

b.

d.

y
(1, 4)

e.

(-2, 4)
(3, 0)

(0, 0)

(4, -5)

f. inverse function does not exist

14. a. 5 b. 0 c. 3 d. 14 e. 7 15. a.

y
(-5, 6)

(-3, -6)

(-3, -8)

x 9 +1
d.

x +5
4

(-3, 2)
(-1, -2)

(-5, 6)

(7, 5)

(-2, 2)

c.

(7, -6)

(3, -5)

2x 1
2x
22 x 1
2x 1
6x 5
3x 7
b.
c.
16. a.
b.
c. x d.
e. 2x 11
2x
3x 2
3x
x2
2 x +1
2

3x 7
x 1
g.
17. a. 1 b. 1 c. 1 d. 17 18. a. 1 b. 2 19. a. find two different x-values that give the same
x5
2x 4
16
y-value b. assume that for x1 x2, f(x1) f(x2) 20. 2 21. Compare the functions when the argument is x and x

f.

22. Compare the functions when the argument is x and x

23. a. x +9 3 b. x +11+ 4

24. Try to compose

each of the sides with g f

EXERCISES

5 .1

1. a.

b.

-1

3
2

-3
-4

2x2=3y
1

2. a. yes b. no c. yes d. no

Answers to Exercises

y=2x2

2
3

c.

1
x

y 4x2=0
2

y
0

-1

y=3x2

1
3

3y=x2

2y=3x2
y=3x2

y+4x2=0

-1
-

3. a.

1
1
7
25
b.
c.
d.
3
5
9
8

1
2

-2

x
2y+x2=0
y+2x2=0

241

4. a.

y
-4 -3 -2 -1 0 1 2 3 4
1
x
2
-2
-

5. a. V(0, 0), x = 0, ymin = 0 b. V(0, 0), x = 0, ymax = 0


c. V(0, 2), x = 0, ymin = 2 d. V(0, 1), x = 0, ymax = 1
6 6
6
6
5 25
5
25
e. V( ,
f. V( , ), x = , ymin =
), x = , ymax =
7 5
7
5
4 8
4
8

9
2

2 5
2
5
g. V( , ), x = , ymin =
3 3
3
3

1 5
1
5
h. V( , ), x = , ymax =
2 4
2
4

3 75
3
75
i. V( , ), x = , ymin =
j. V(4, 0), x = 4, ymax = 0
2
4
2
4
5
41
k. V(3, 5), x = 3, ymax = 5 l. V(1, 3), x = 1, ymin = 3 6.
7. 1 8.
9. a. (0, 0), (4, 0)
2
8
3
3
b. (3, 0), (0, 9) c. (0, 396) d. (22, 0), (22, 0), (0, 8) e. ( , 0), (1, 0), (0, 3) f. (0, 5) g. ( , 0), (0, 9)
2
4
h. (0, 4) 10. a. m (22, 22) b. m = 22 c. m (,
-8

22) (22, ) 11. 3


y

12. a.

b.
3 x

-3

c.

y
x

h.

-48

i.

1
2+2 x

3 x

-3
-4

5 x

-5
-5

-2

x
y

j.

13.

12

13

2 2
-3

f.

-1

98

-7

e.

-4
1

-1

d.

-9

g.

-4

f(A)= [9, 7)

-1 1
4

-2
3

-3

4 5

-5
-8
-9

14.

15. a. ymax = 12, ymin = 3

3
-1
1

16. ymax = 64

17. ymin =

9
4

1
18. a = , b =1
2

19. y = 3(x 5)2 20. 3(x 1)2 + 2 2; 3(x 1)2 0; 3(x 1)2 0 21. m [0,

16
]
9

-5

242

Answers to Exercises

22. a.

y=x2+4

b.

y=x2
y=x2 2

y=x2
4

-2

y
16

-25

y=x2 2

g.

y=(x 4)2

-1

y=x2

y=(x+1)2

3)2

b.

c.

d.

1 2 3

-1

-1
2 3 4

-1

24. a. y = (x 1)2 + 3 b. y =
f. y =

1
(x 3)(x + 3)
3

y=(x 2)2

y=x2

y=(x+1)2 2

23. a.

y=(x 2)2+3

x
-2

y=(x

h.
-1

-9
y=x2

y=x2

y=(x+5)2

y=x2

f.

-5

-2

-2

y=x2

d.

e.

c.

1
1
x(x 5) c. y = (x + 3)2 d. y = (x + 1)2 + 3 e. y = x2 + 4x 3
6
3

25. a. y = 3x2 + x + 4 b. y = x2 + 4x 4

c. y = (x + 1)2 4 d. y = x2 x 4 27. 12 28.


33. 10 square units 34. m (, 1) (0, )

18
5

29. 0 30.

1
2

26. a. y = x2 4 b. y = (x 2)2
31.

35. 64 square units 36.

25
4

32.

9
square units
4

49
37. 2 38. (3, 0) 39. 9 40. 1
20

41. 35 square units

Answers to Exercises

243

EXERCISES

6 .1

1. a. (2, ) b. (, 6] c. (, 7) d. [4, ) e. [2, 4) f. (1, 0] g. (5,


x

2. a.

6
f. x
;
a+ 3

1. a.
x2

5x+4

d.

1/4

c. (, 0) (2, )

1/2
x

14/3

+
0

b. x 0;

e. x > 2 ;
3

2/3
x

3
a4+4

3
g. x > 4
;
a +4

2. a. (4, 5) b. (0,

x 7
- +
2 3

6 .2

16x2+8x 1

c.

d. x 3;

6/(a+3)

EXERCISES

-5/3

1
3. a. x > ;
2

+
2

c. x > 2;

3x 5

6/3

3x 6

b.

4x + 1

d.

-1/4

1
3
) h. [0.5, ]
2
2

b.
2x2+x

3/2

-2

e.

2x2

+ + + + + +

3x+4

f.

x
4x2+10x 25

c.

12x2+43x+1

4
7
4
4
f. ( , 1) g. (, 4) (2, )
) c. ( , ] [0, ) d. e. x =
7
6
3
5

4. (, 23 1) (23 1, )

5. (3 22, 3 + 22)

3/6

3. a. (0, 2) b. 0, 2

6. {6, 5, 4, 3, 2, 1, 0 1, 2}

1
3
2
2
5
7. a. (0, ), x 1 b. ( , ) ( , 2) c. x 2, ( , 1) ( , 3) d. ( , ) (1, )
3
2
3
3
2
1
3
7
e. (4, 3) (2, 1) ( , 3) f. (1, 5) g. ( , ) ( , ) h. (8, 1) i. {2} j. (, 7] (1, 0) (0, 1] (3, )
2
2
3

k. ( ,

6
] [1, 0) [1,
2

6
4
1
11
] l. ( , 1) ( , 2) m. (, 2) {4, 2} n. [0, ] o. ( , 1) ( , )
3
3
5
2

1
p. (, 2) (2, 1) (1, 0) q. [ , 0] [1, )
2

3
8. a. (0, 1) b. (0, 1) c. d. [ , 2)
2

9. a. {7, 7}

b. (0, 16) (16, ) c. (, 2) [2, ) d. [2, 3] [1, 1] 10. a. (0, 3 1) (4, 3) (3 1, 2) (1, 2)
11. x 0, (2, 1)
244

Answers to Exercises

EXERCISES
1. a. (2,
c. (8,

6 .3

3
) b. (1, 3) c. (6, 5) d. (1, 1) e. (1, 2) f. [1, 2) g. (2, 4) h. (0, 1)
2

13
) (0, 5)
2

2. a. (0, 1) b. (1, 1) (3, 5)

3
5
3. a. ( , 1] [ , ) b. ( , ) (3, ) c. (, 5) (1, ) d. (, 4) (0, )
2
3

3
2 2
13
5 3
e. [ , 2) f. [ , ] g. (, 3) 4. a. (3, ) b. [ , 4) c. [4, ) d. (3, 1) e. [3, ) f. (9, ) 5. a. [3, ) b. [2,
]
2
7 3
5
10 2

c. [2, 3] d. [15,
7. [1, 5) (10, )

TEST
1.
2.
3.
4.
5.
6.
7.
8.
9.
10.

B
D
A
D
B
B
C
C
C
C

TEST
1.
2.
3.
4.
5.
6.
7.
8.

E
B
E
A
C
D
A
D

13 5
417
) e. [3,
)
64
2

TEST
B
B
C
B
C
D
C
D
A
B

6A
9.
10.
11.
12.
13.
14.
15.
16.

Answers to Exercises

5 1
,
2

13 + 3
11 1
11+1
] b. [1, 3] c. [ 6,
) (
,
2
2
2

5
8. a. (2, ) b. (1, ) c. ( , ) d. (, 2) (2, )
2

1A
11.
12.
13.
14.
15.
16.
17.
18.
19.
20.

6. a. [

1.
2.
3.
4.
5.
6.
7.
8.
9.
10.

C
D
B
C
B
D
C
B
D
D

TEST
E
B
D
D
C
B
A
B

1.
2.
3.
4.
5.
6.
7.
8.

D
B
A
C
E
B
E
C

1B
11.
12.
13.
14.
15.
16.
17.
18.
19.
20.

TEST
D
C
C
B
A
C
C
D
B
C

1.
2.
3.
4.
5.
6.
7.
8.

C
E
C
B
E
D
C
C

9. (5, 1)

3A
9.
10.
11.
12.
13.
14.
15.
16.

TEST
B
B
D
A
E
C
B
D

1.
2.
3.
4.
5.
6.
7.
8.

B
D
E
A
D
B
A
C

3B
9.
10.
11.
12.
13.
14.
15.

A
E
B
D
B
B
S

6B
9.
10.
11.
12.
13.
14.
15.
16.

C
D
A
B
E
C
D
D
245

6]

246

Answers to Exercises

Answers to Exercises

247

248

Answers to Exercises

Вам также может понравиться